Sie sind auf Seite 1von 48

B.

Labor Organizations ● Hanjin filed a supplemental petition, adding the alternative ground that Samahan
1. Policy committed a misrepresentation in connection with the list of members and/or voters
Articles 218(A)(b); (c); (d); and (g), Labor Code. who took part in the ratification of their constitution and by-laws in its application for
registration. Hanjin claimed that Samahan made it appear that its members were all
A) To promote and emphasize the primacy of free collective bargaining and negotiations, qualified to become members of the workers' association.
including voluntary arbitration, mediation and conciliation, as modes of settling labor or ● DOLE-Pampanga called for a conference, wherein Samahan requested for a 10-day
industrial disputes; period to file a responsive pleading. No pleading, however, was submitted. Instead,
B) To promote free trade unionism as an instrument for the enhancement of democracy Samahan filed a motion to dismiss on April 14, 2010.
and the promotion of social justice and development; ● DOLE Regional Director Ernesto Bihis ruled in favor of Hanjin
C) To foster the free and voluntary organization of a strong and united labor movement; ● Samahan failed to adduce evidence that the remaining 63 members were also
D) To promote the enlightenment of workers concerning their rights and obligations as employees of Hanjin
union members and as employees; ● Bureau of Labor Relations ruled in favour of Samahan
E) (G) To ensure the participation of workers in decision and policy-making processes ● In the MR of Hanjin decision is affirmed but with a DIRECTIVE for SAMAHAN to
affecting their rights, duties and welfare. remove "HANJIN SHIPYARD" from its name.
● CA rendered its decision that Samahan as a legitimate workers' association was
2. Definition contrary to the provisions of Article 243 of the Labor Code. It stressed that only 57 out
Article 219(g), Labor Code. of the 120 members were actually working in Hanjin.
G. "Labor organization" means any union or association of employees which exists in whole or in ● In the preamble of Samahan's Constitution and By-laws, "KAMI, ang mga
part for the purpose of collective bargaining or of dealing with employers concerning terms and Manggagawa sa Hanjin Shipyard" created an impression that all its members were
conditions of employment. employees of HHIC.
3. Categories ● Samahan could not register it as a legitimate worker's association because the place
Articles 219(g); (h); (i); and (j), Labor Code. where Hanjin's industry was located was not a rural area.
G. "Labor organization" means any union or association of employees which exists in whole or in
part for the purpose of collective bargaining or of dealing with employers concerning terms and ISSUE: CA erred in finding that samahan cannot form a workers' association of employees in
conditions of employment. hanjin and instead should have formed a union, hence their registration as a workers'
association should be cancelled.
H. "Legitimate labor organization" means any labor organization duly registered with the
Department of Labor and Employment, and includes any branch or local thereof. HELD: The petition is partly meritorious.

I. "Company union" means any labor organization whose formation, function or administration No misrepresentation on the part of Samahan to warrant cancellation of registration
has been assisted by any act defined as unfair labor practice by this Code.
In this case, Samahan's registration was cancelled not because its members were prohibited
Samahan ng Manggagawa sa Hanjin Shipyard v. Bureau of Labor Relations, G.R. No. from forming a workers' association but because they allegedly committed misrepresentation for
211145, October 14, 2015. using the phrase, "KAMI, ang mga Manggagawa sa HAN JIN Shipyard."

FACTS: Misrepresentation, as a ground for the cancellation of registration of a labor organization, is


● Samahan, through its authorized representative, Alfie F. Alipio, filed an application for committed "in connection with the adoption, or ratification of the constitution and by-laws or
registration of its name "Samahan ng Mga Manggagawa sa Hanjin Shipyard" with the amendments thereto, the minutes of ratification, the list of members who took part in the
DOLE. With a total of 120 members. ratification of the constitution and by-laws or amendments thereto, and those in connection with
● DOLE issued the corresponding certificate of registration. the election of officers, minutes of the election of officers, and the list of voters,."
● Then respondent Hanjin Heavy Industries (Hanjin), filed a petition with DOLE praying
for the cancellation of registration of Samahan's association on the ground that its The records of this case reveal no deliberate or malicious intent to commit misrepresentation on
members did not fall under any of the types of workers enumerated in the second the part of Samahan.
sentence of Article 243 (now 249).
● Hanjin opined that only ambulant, intermittent, itinerant, rural workers, self-employed, Removal of the word "Hanjin Shipyard" from the association's name, however, does not infringe
and those without definite employers may form a workers' association. on Samahan's right to self-organization
● It further posited that one third (1/3) of the members of the association had definite
employers and the continued existence and registration of the association would It would be misleading for the members of Samahan to use "Hanjin Shipyard" in its name as it
prejudice the company's goodwill. could give the wrong impression that all of its members are employed by Hanjin. in the directive

1
of the BLR removing the words "Hanjin Shipyard," no abridgement of Samahan's right to self- Article 249. Equity of the incumbent. All existing federations and national unions which meet the
organization was committed. qualifications of a legitimate labor organization and none of the grounds for cancellation shall
continue to maintain their existing affiliates regardless of the nature of the industry and the
Cebu Seamen’s Association, Inc. v. Ferrer-Calleja, G.R. No. 83190, August 4, 1992. location of the affiliates.

4. Rationale Takata (Philippines) Corporation v. Bureau of Labor Relations, G.R. No. 192676, June 4,
Guijarno v. Court of Industrial Relations, G.R. Nos. L-28791-93, August 27, 1973. 2014.

FACTS:
5. Regulation On July 7, 2009, Takata Corp filed with DOLE a petition for cancellation of Certificate of Union
Electromat Manufacturing and Recording Corporation v. Lagunzad, G.R. No. 172699, July Registration of Samahang Lakas Manggagawa ng Takata (SALAMAT) on the ground that was
guilty of misrepresentation, false statements, and fraud with respect to the number of those who
27, 2011.
participated in their organizational meeting, the adoption and ratification of its Constitution and
by-laws, and in the election of its officers. 

a. Registration Takata’s contention:


i. Requirements  In the May 1, 2009 organizational meeting of respondent, only 68 attendees signed
Articles 240; 241; 244; and 249, Labor Code. the attendance sheet, and which number comprised only 17% of the total number of
the 396 regular rank- and-file employees which respondent sought to represent, and
Article 240. Requirements of registration. Any applicant labor organization, association or group hence, respondent failed to comply with the 20% minimum membership requirement; 
of unions or workers shall acquire legal personality and shall be entitled to the rights and  The document “Pangalan ng mga Kasapi ng Unyon” bore no signatures of the alleged
privileges granted by law to legitimate labor organizations upon issuance of the certificate of 119 union members; 
 The employees were not given sufficient information on the documents they signed;
registration based on the following requirements.
 The document “Sama-Samang Pahayag ng Pagsapi” was not submitted at the time of
the filing of respondent's application for union registration; that the 119 union
Fifty pesos (P50.00) registration fee; members were actually only 117;
 The total number of petitioner's employees as of May 1, 2009 was 470, and not 396
The names of its officers, their addresses, the principal address of the labor organization, the as respondent claimed.
minutes of the organizational meetings and the list of the workers who participated in such
meetings; SALAMAT’s contention:
 Denied the charge and claimed that the 119 union members were more than the 20%
The names of all its members comprising at least twenty percent (20%) of all the employees in requirement for union registration.
the bargaining unit where it seeks to operate; (As amended by Executive Order No. 111,  The document “Sama-Samang Pahayag ng Pagsapi sa Unyon” which it presented in
December 24, 1986) its petition for certification election supported their claim of 119 members. 
 Takata was estopped from assailing its legal personality as it agreed to a certification
election and actively participated in the pre-election conference of the certification
If the applicant union has been in existence for one or more years, copies of its annual financial election proceedings.
reports; and  The union members were informed of the contents of the documents they signed and
that the 68 attendees to the organizational meeting constituted more than 50% of the
Four (4) copies of the constitution and by-laws of the applicant union, minutes of its adoption or total union membership, hence, a quorum existed for the conduct of the said meeting.
ratification, and the list of the members who participated in it. DOLE:
 Granted petition for cancellation of respondent's certificate of registration. Thus, the
Article 244. Additional requirements for federations or national unions. Subject to Article 238, if Union Certificate of Registration is revoked and/or cancelled and SALAMAT is delisted
from the roll of legitimate labor organization of this office.
the applicant for registration is a federation or a national union, it shall, in addition to the
 68 employees who attended the organizational meeting was obviously less than 20%
requirements of the preceding Articles, submit the following: of the total number of 396 regular rank-and-file employees which respondent sought
to represent, hence, short of the union registration requirement;
Proof of the affiliation of at least ten (10) locals or chapters, each of which must be a duly  attendance sheet which contained the signatures and names of the union members
recognized collective bargaining agent in the establishment or industry in which it operates, totalling to 68 contradicted the list of names stated in the document denominated as
supporting the registration of such applicant federation or national union; and “Pangalan ng mga Kasapi ng Unyon.” 
 The document “Sama-Samang Pahayag ng Pagsapi” was not attached to the
The names and addresses of the companies where the locals or chapters operate and the list of application for registration as it was only submitted in the petition for certification
election filed by respondent at a later date. 
all the members in each company involved.

2
 The proceedings in the cancellation of registration and certification elections are two San Miguel Corporation Employees Union – Philippine Transport and General Workers
different and entirely separate and independent proceedings which were not Organization (SMCEU-PTGWO) v. San Miguel Packaging Products Employees Union –
dependent on each other. Pambansang Diwa ng Manggagawang Pilipino (SMPEU-PDMP), G.R. No. 171153,
September 12, 2007.
Bureau of Labor Relations’s Ruling:
 Reversed the decision of DOLE and ordered that SALAMAT shall remain in the roster FACTS
of labor organizations.
Petitioner is the incumbent bargaining agent for the bargaining unit comprised of the regular
 Takata failed to prove that respondent deliberately and maliciously misrepresented the
monthly-paid rank and file employees of the three divisions of San Miguel Corporation (SMC),
number of rank-and-file employees. It pointed out petitioner's basis for the alleged
non-compliance with the minimum membership requirement for registration was the namely, the San Miguel Corporate Staff Unit (SMCSU), San Miguel Brewing Philippines (SMBP),
attendance of 68 members to the May 1, 2009 organizational meeting supposedly and the San Miguel Packaging Products (SMPP), in all offices and plants of SMC, including the
comprising only 17% of the total 396 regular rank-and-file employees.  Metal Closure and Lithography Plant in Laguna. It had been the certified bargaining agent for 20
 The list of employees who participated in the organizational meeting was a separate years – from 1987 to 1997.
and distinct requirement from the list of the names of members comprising at least
20% of the employees in the bargaining unit; and that there was no requirement for Respondent is registered as a chapter of Pambansang Diwa ng Manggagawang Pilipino
signatures opposite the names of the union members; and there was no evidence
(PDMP). PDMP issued Charter Certificate No. 112 to respondent on 15 June 1999. In
showing that the employees assailed their inclusion in the list of union members.
compliance with registration requirements, respondent submitted the requisite documents to the
BLR for the purpose of acquiring legal personality.6 Upon submission of its charter certificate
CA: Denied the petition and affirmed the decision of the BLR.
and other documents, respondent was issued Certificate of Creation of Local or Chapter PDMP-
ISSUE: 01 by the BLR on 6 July 1999.
Whether or not SALAMAT’s Certificate of Union Registration should be cancelled -NO. 
Respondent filed with the Med-Arbiter of the DOLE Regional Officer in the National Capital
HELD: Region (DOLE-NCR), three separate petitions for certification election to represent SMPP,
Petitioner's allegation of misrepresentation and fraud is based on its claim that during the SMCSU, and SMBP. All three petitions were dismissed, on the ground that the separate
organizational meeting on May 1, 2009, only 68 employees attended, while respondent claimed petitions fragmented a single bargaining unit.
that it has 119 members as shown in the document denominated as “Pangalan ng mga Kasapi
ng Unyon;” hence, respondent misrepresented on the 20% requirement of the law as to its Petitioner filed with the DOLE-NCR a petition seeking the cancellation of respondent's
membership.
registration and its dropping from the rolls of legitimate labor organizations. In its petition,
petitioner accused respondent of committing fraud and falsification, and non-compliance with
SC ruled that It does not appear in Article 234 (b) of the Labor Code that the attendees in the
registration requirements in obtaining its certificate of registration. It raised allegations that
organizational meeting must comprise 20% of the employees in the bargaining unit. In fact, even
the IRR of the Labor Code does not so provide. It is only under Article 234 (c) that requires the respondent violated Articles 239(a), (b) and (c)10 and 234(c)11 of the Labor Code. Moreover,
names of all its members comprising at least twenty percent (20%) of all the employees in the petitioner claimed that PDMP is not a legitimate labor organization, but a trade union center,
bargaining unit where it seeks to operate. Clearly, the 20% minimum requirement pertains to the hence, it cannot directly create a local or chapter.
employees’ membership in the union and not to the list of workers who participated in the
organizational meeting.
DOLE REGIONAL DIRECTOR
Indeed, Article 234 (b) and (c) provide for separate requirements, which must be submitted for issued an Order dismissing the allegations of fraud and misrepresentation, and irregularity in the
the union's registration, and which respondent did submit. Here, the total number of employees submission of documents by respondent. Regional Director Lim further ruled that respondent is
in the bargaining unit was 396, and 20% of which was about 79. Respondent submitted a
document entitled “Pangalan ng Mga Kasapi ng Unyon” showing the names of 119 employees allowed to directly create a local or chapter. However, he found that respondent did not comply
as union members, thus respondent sufficiently complied even beyond the 20% minimum with the 20% membership requirement and, thus, ordered the cancellation of its certificate of
membership requirement. Respondent also submitted the attendance sheet of the organizational registration and removal from the rolls of legitimate labor organizations.
meeting which contained the names and signatures of the 68 union members who attended the
meeting. Considering that there are 119 union members which are more than 20% of all the BLR
employees of the bargaining unit, and since the law does not provide for the required number of Respondent appealed to BLR. BLR granted appeal. Ruled that As a chartered local union,
members to attend the organizational meeting, the 68 attendees which comprised at least the
appellant is not required to submit the number of employees and names of all its members
majority of the 119 union members would already constitute a quorum for the meeting to
proceed and to validly ratify the Constitution and By-laws of the union. Thus, there’s, no basis for comprising at least 20% of the employees in the bargaining unit where it seeks to operate. Thus,
petitioner to contend that grounds exist for the cancellation of respondent's union registration. the revocation of its registration based on non-compliance with the 20% membership
For fraud and misrepresentation to be grounds for cancellation of union registration under Article requirement does not have any basis in the rules. Petitioner filed MR but BLR denied.
239 of the Labor Code, the nature of the fraud and misrepresentation must be grave and
compelling enough to vitiate the consent of a majority of union members. CA

3
Petitioner filed Rule 65 certiorari before CA. CA affirned BLR, ruled that a registered federation labor organization, particularly the right to file a petition for certification election, first and
or national union may directly create a local by submitting to the BLR copies of the charter foremost, depends on whether or not the labor organization has attained the status of a
certificate, the local's constitution and by-laws, the principal office address of the local, and the legitimate labor organization.
names of its officers and their addresses. Upon complying with the documentary requirements,
the local shall be issued a certificate and included in the roster of legitimate labor organizations. The applicable Implementing Rules enunciates a two-fold procedure for the creation of a chapter
The [herein respondent] is an affiliate of a registered federation PDMP, having been issued a or a local. The first involves the affiliation of an independent union with a federation or national
charter certificate. Under the rules we have reviewed, there is no need for SMPPEU to show a union or industry union. The second, finding application in the instant petition, involves the direct
membership of 20% of the employees of the bargaining unit in order to be recognized as a creation of a local or a chapter through the process of chartering.27
legitimate labor union.
A duly registered federation or national union may directly create a local or chapter by submitting
ISSUE: to the DOLE Regional Office or to the BLR two copies of the following:
Whether or not respondent met requirements to be a legitimate labor organization
(a) A charter certificate issued by the federation or national union indicating the creation or
RULING:NO ON ACCOUNT OF TRADE UNION NOT BEING ALLOWED TO CHARTER establishment of the local/chapter;

NOTE: Focus on registration requirements (b) The names of the local/chapter's officers, their addresses, and the principal office of the
local/chapter; and
Petitioner posits that respondent is required to submit a list of members comprising at least 20%
of the employees in the bargaining unit before it may acquire legitimacy, citing Article 234(c) of (c) The local/chapter's constitution and by-laws; Provided, That where the local/chapter's
the Labor Code which stipulates that any applicant labor organization, association or group of constitution and by-laws is the same as that of the federation or national union, this fact shall be
unions or workers shall acquire legal personality and shall be entitled to the rights and privileges indicated accordingly.
granted by law to legitimate labor organizations upon issuance of the certificate of registration
based on the following requirements: All the foregoing supporting requirements shall be certified under oath by the Secretary or the
Treasurer of the local/chapter and attested to by its President.28
a. Fifty pesos (P50.00) registration fee;
The Implementing Rules stipulate that a local or chapter may be directly created by a federation
b. The names of its officers, their addresses, the principal address of the labor organization, the or national union. A duly constituted local or chapter created in accordance with the foregoing
minutes of the organizational meetings and the list of the workers who participated in such shall acquire legal personality from the date of filing of the complete documents with the BLR.
meetings; The issuance of the certificate of registration by the BLR or the DOLE Regional Office is not the
operative act that vests legal personality upon a local or a chapter under Department Order No.
c. The names of all its members comprising at least twenty percent (20%) of all the employees 9. Such legal personality is acquired from the filing of the complete documentary requirements
in the bargaining unit where it seeks to operate; enumerated in Section 1, Rule VI.30

d. If the applicant union has been in existence for one or more years, copies of its annual However SC agreed with petitioners that respondent cannot be registered as it is a chapter of a
financial reports; and trade union center, not a legitimate labor organization.

e. Four (4) copies of the constitution and by-laws of the applicant union, minutes of its adoption Culling from its definition as provided by Department Order No. 9, a trade union center is any
or ratification and the list of the members who participated in it group of registered national unions or federations organized for the mutual aid and protection of
its members; for assisting such members in collective bargaining; or for participating in the
A legitimate labor organization19 is defined as "any labor organization duly registered with the formulation of social and employment policies, standards, and programs, and is duly registered
Department of Labor and Employment, and includes any branch or local thereof." The mandate with the DOLE in accordance with Rule III, Section 2 of the Implementing Rules.46 The same
of the Labor Code is to ensure strict compliance with the requirements on registration because a rule provides that the application for registration of an industry or trade union center shall be
legitimate labor organization is entitled to specific rights under the Labor Code, and are involved supported by the following:
in activities directly affecting matters of public interest. Registration requirements are intended to
afford a measure of protection to unsuspecting employees who may be lured into joining (a) The list of its member organizations and their respective presidents and, in the case of an
unscrupulous or fly-by-night unions whose sole purpose is to control union funds or use the industry union, the industry where the union seeks to operate;
labor organization for illegitimate ends. Legitimate labor organizations have exclusive rights
under the law which cannot be exercised by non-legitimate unions, one of which is the right to (b) The resolution of membership of each member organization, approved by the Board of
be certified as the exclusive representativeof all the employees in an appropriate collective Directors of such union;
bargaining unit for purposes of collective bargaining. The acquisition of rights by any union or
4
(c) The name and principal address of the applicant, the names of its officers and their
addresses, the minutes of its organizational meeting/s, and the list of member organizations and (a) The names of the chapter's officers, their addresses, and the principal office of the chapter;
their representatives who attended such meeting/s; and and

(d) A copy of its constitution and by-laws and minutes of its ratification by a majority of the (b) The chapter's constitution and by-laws: Provided, That where the chapter's constitution and
presidents of the member organizations, provided that where the ratification was done by-laws are the same as that of the federation or the national union, this fact shall be indicated
simultaneously with the organizational meeting, it shall be sufficient that the fact of ratification be accordingly.
included in the minutes of the organizational meeting.47
The additional supporting requirements shall be certified under oath by the secretary or
Evidently, while a "national union" or "federation" is a labor organization with at least ten locals treasurer of the chapter and attested by its president. (Emphasis ours.)
or chapters or affiliates, each of which must be a duly certified or recognized collective
bargaining agent;48 a trade union center, on the other hand, is composed of a group of Article 234 now includes the term trade union center, but interestingly, the provision indicating
registered national unions or federations.49 the procedure for chartering or creating a local or chapter, namely Article 234-A, still makes no
mention of a "trade union center."
The Implementing Rules, as amended by Department Order No. 9, provide that "a duly
registered federation or national union" may directly create a local or chapter. The provision Also worth emphasizing is that even in the most recent amendment of the implementing rules,54
reads: there was no mention of a trade union center as being among the labor organizations allowed to
charter.
Section 1. Chartering and creation of a local/chapter. – A duly registered federation or national
union may directly create a local/chapter by submitting to the Regional Office or to the Bureau In sum, although PDMP as a trade union center is a legitimate labor organization, it has no
two (2) copies of the following: power to directly create a local or chapter. Thus, SMPPEU-PDMP cannot be created under the
more lenient requirements for chartering, but must have complied with the more stringent rules
(a) A charter certificate issued by the federation or national union indicating the creation or for creation and registration of an independent union, including the 20% membership
establishment of the local/chapter; requirement.

(b) The names of the local/chapter's officers, their addresses, and the principal office of the Progressive Development Corporation v. The Honorable Secretary, Department of Labor
local/chapter; and and Employment, G.R. No. 96425, February 4, 1992.

(c) The local/chapter's constitution and by-laws; provided that where the local/chapter's FACTS
constitution and by-laws is the same as that of the federation or national union, this fact shall be Respondent Pambansang Kilusan ng Paggawa (KILUSAN) -TUCP (hereinafter referred to as
indicated accordingly. Kilusan) filed with the Department of Labor and Employment (DOLE) a petition for certification
election among the rank-and-file employees of the petitioner alleging that it is a legitimate labor
All the foregoing supporting requirements shall be certified under oath by the Secretary or the federation and its local chapter, Progressive Development Employees Union, was issued a
Treasurer of the local/chapter and attested to by its President.50 charter certificate. Kilusan claimed that there was no existing collective bargaining agreement
and that no other legitimate labor organization existed in the bargaining unit.
Department Order No. 9 mentions two labor organizations either of which is allowed to directly
create a local or chapter through chartering – a duly registered federation or a national union. Petitioner PDC filed its motion to dismiss dated July 11, 1990 contending that the local union
Department Order No. 9 defines a "chartered local" as a labor organization in the private sector failed to comply with Rule II Section 3, Book V of the Rules Implementing the Labor Code, as
operating at the enterprise level that acquired legal personality through a charter certificate, amended, which requires the submission of: (a) the constitution and by-laws; (b) names,
issued by a duly registered federation or national union and reported to the Regional Office in addresses and list of officers and/or members; and (c) books of accounts.
accordance with Rule III, Section 2-E of these Rules
Respondent Kilusan submitted a rejoinder to PDC's motion to dismiss claiming that it had
submitted the necessary documentary requirements for registration, such as the constitution and
ART. 234-A. Chartering and Creation of a Local Chapter. — A duly registered federation or by-laws of the local union, and the list of officers/members with their addresses. Kilusan further
national union may directly create a local chapter by issuing a charter certificate indicating the averred that no books of accounts could be submitted as the local union was only recently
establishment of the local chapter. The chapter shall acquire legal personality only for purposes organized
of filing a petition for certification election from the date it was issued a charter certificate.
Petitioner insisted that upon verification with the Bureau of Labor Relations (BLR), it found that
The chapter shall be entitled to all other rights and privileges of a legitimate labor organization the alleged minutes of the organizational meeting was unauthenticated, the list of members did
only upon the submission of the following documents in addition to its charter certificate: not bear the corresponding signatures of the purported members, and the constitution and by-
5
laws did not bear the signature of the members and was not duly subscribed. It argued that the organizations upon issuance of the certificate of registration based on the following
private respondent therefore failed to substantially comply with the registration requirements requirements:
provided by the rules.
(a) Fifty-pesos (P50.00) registration fee;
MED-ARBITER
Held that there was substantial compliance with the requirements for the formation of the (b) The names of its officers, their addresses, the principal address of the labor organization, the
chapter. He further stated that mere issuance of the charter certificate by the federation was minutes of the organizational meeting and the list of the workers who participated in such
sufficient compliance with the rules. Considering that the establishment was unorganized, he meetings;
maintained that a certification election should be conducted to resolve the question of
representation (c) The names of all its members comprising at least twenty 20% percent of all the employees in
the bargaining unit where it seek to operate;
LABOR USEC
Treating the motion for reconsideration filed by the PDC as an appeal to the Office of the (d) If the applicant has been in existence for one or more years, copies , of its annual financial
Secretary, Undersecretary Laguesma held that the same was merely a "reiteration of the issues reports; and
already ventilated in the proceedings before the Med-Arbiter, specifically, the matter involving
the formal organization of the chapter. Also denied MR (e) Four copies of the constitution and by-laws of the applicant union, the minutes of its adoption
or ratification and the list of the members who participated in it.
ISSUE:
Whether or not private respondent complied with requirements for registration: And under Article 235 (Action on Application)

RULING: The Bureau shall act on all applications for registration within thirty (30) days from filing.
NO
All requisite documents and papers shall be certified under oath by the secretary or the treasurer
Petitioner's contention is that a labor organization may not validly invest the status of legitimacy of the organization, as the case may be, and attested to by its president.
upon a local or chapter through the mere expedient of issuing a charter certificate and
submitting such certificate to the BLR Petitioner PDC posits that such local or chapter must at Moreover, section 4 of Rule II, Book V of the Implementing Rules requires that the application
the same time comply with the requirement of submission of duly subscribed constitution and should be signed by at least twenty percent (20%) of the employees in the appropriate
by-laws, list of officers and books of accounts. PDC points out that the constitution and by-laws bargaining unit and be accompanied by a sworn statement of the applicant union that there is no
and list of officers submitted were not duly subscribed. Likewise, the petitioner claims that the certified bargaining agent or, where there is an existing collective bargaining agreement duly
mere filing of the aforementioned documents is insufficient; that there must be due recognition or submitted to the DOLE, that the application is filed during the last sixty (60) days of the
acknowledgment accorded to the local or chapter by BLR through a certificate of registration or agreement.
any communication emanating from it. The Solicitor General, in behalf of the public respondent,
avers that there was a substantial compliance with the requirements for the formation of a But when an unregistered union becomes a branch, local or chapter of a federation, some of the
chapter. Moreover, he invokes Article 257 of the Labor Code which mandates the automatic aforementioned requirements for registration are no longer required. The provisions governing
conduct by the Med-Arbiter of a certification election in any establishment where there is no union affiliation are found in Rule II, Section 3, Book V of the Implementing Rules, the relevant
certified bargaining agreement. portions of which are cited below:

While Article 257 cited by the Solicitor General directs the automatic conduct of a certification Sec. 3. Union affiliation; direct membership with national union. — An affiliate of a labor
election in an unorganized establishment, it also requires that the petition for certification federation or national union may be a local or chapter thereof or an independently registered
election must be filed by a legitimate labor organization. Article 242 enumerates the exclusive union.
rights of a legitimate labor organization among which is the right to be certified as the exclusive
representative of all the employees in an appropriate collective bargaining unit for purposes of a) The labor federation or national union concerned shall issue a charter certificate indicating the
collective bargaining creation or establishment of a local or chapter, copy of which shall be submitted to the Bureau of
Labor Relations within thirty (30) days from issuance of such charter certificate.
Ordinarily, a labor organization acquires legitimacy only upon registration with the BLR. Under
Article 234 (Requirements of Registration): b) An independently registered union shall be considered an affiliate of a labor federation or
national union after submission to the Bureau of the contract or agreement of affiliation within
Any applicant labor organization, association or group of unions or workers shall acquire legal thirty (30) days after its execution.
personality and shall be entitled to the rights and privileges granted by law to legitimate labor e) The local or chapter of a labor federation or national union shall have and maintain a
constitution and by laws, set of officers and books and accounts. For reporting purposes, the
6
procedure governing the reporting of independently registered unions, federations or national
unions shall be observed. ART. 242. [235] Action on Application. The Bureau shall act on all applications for registration
within thirty (30) days from filing.
Paragraph (a) refers to the local or chapter of a federation which did not undergo the rudiments
of registration while paragraph (b) refers to an independently registered union which affiliated All requisite documents and papers shall be certified under oath by the secretary or the treasurer
with a federation. Implicit in the foregoing differentiation is the fact that a local or chapter need of the organization, as the case may be, and attested to by its president.
not be independently registered. By force of law (in this case, Article 212[h]); such local or
chapter becomes a legitimate labor organization upon compliance with the aforementioned ART. 243. [236] Denial of Registration; Appeal. The decision of the Labor Relations Division in
provisions of Section 3. the regional office denying registration may be appealed by the applicant union to the Bureau
within ten (10) days from receipt of notice thereof.
Thus, several requirements that are otherwise required for union registration are omitted, to wit:
Progressive Development Corporation – Pizza Hut v. Laguesma, G.R. No. 115077, April
(1) The requirement that the application for registration must be signed by at least 20% of the 18, 1997.
employees in the appropriate bargaining unit;
Nagkakaisang Lakas ng Manggagawa (NLM)-Katipunan (respondent Union) filed a petition for
2) The submission of officers' addresses, principal address of the labor organization, the minutes certification election with the Department of Labor (National Capital Region) in behalf of the rank
of organizational meetings and the list of the workers who participated in such meetings; and file employees of the Progressive Development Corporation (Pizza Hut).

3) The submission of the minutes of the adoption or ratification of the constitution and by the Petitioner filed verified Motion to Dismiss the petition alleging fraud, falsification and
laws and the list of the members who participated in it. misrepresentation in the respondent. Union's registration making it void and invalid. The motion
specifically alleged that: a) respondent Union's registration was tainted with false, forged,
In the case at bar, the constitution and by-laws and list of officers submitted in the BLR, while double or multiple signatures of those who allegedly took part in the ratification of the
attested to by the chapter's president, were not certified under oath by the secretary. respondent Union's constitution and by-laws and in the election of its officers that there
were two sets of supposed attendees to the alleged organizational meeting that was
A local or chapter therefore becomes a legitimate labor organization only upon submission of the alleged to have taken place on June 26, 1993; that the alleged chapter is claimed to have
following to the BLR: been supported by 318 members when in fact the persons who actually signed their
names were much less; and b) while the application for registration of the charter was
1) A charter certificate, within 30 days from its issuance by the labor federation or national union, supposed to have been approved in the organizational meeting held on June 27, 1993, the
and charter certification issued by the federation KATIPUNAN was dated June 26, 1993 or one
(1) day prior to the formation of the chapter, thus, there were serious falsities in the dates of
2) The constitution and by-laws, a statement on the set of officers, and the books of accounts all the issuance of the charter certification and the organization meeting of the alleged chapter.
of which are certified under oath by the secretary or treasurer, as the case may be, of such local
or chapter, and attested to by its president. Other violations alleged in supplement to MTD included:
1.)Election of its officers was held on June 27, 1993; however, it appears from the documents
Absent compliance with these mandatory requirements, the local or chapter does not become a submitted by respondent union to the BIR-DOLE that the Union's constitution and by-laws were
legitimate labor organization. adopted only on July 7, 1993, hence, there was no bases for the supposed election of officers
In the case at bar, the failure of the secretary of PDEU-Kilusan to certify the required documents on June 27, 1993 because as of this date, there existed no positions to which the officers could
under oath is fatal to its acquisition of a legitimate status. be validly elected;

On June 19, 1990, or just three days after the organizational meeting, Kilusan filed a petition for 2) Voting was not conducted by secret ballot in violation of Article 241, section (c) of the Labor
certification election (Records, pages 2 and 3) accompanied by a copy each of the charter Code;
certificate, constitution and by-laws and minutes of the organizational meeting. Had the local
union filed an application for registration, the petition for certification election could not have 3) The Constitution and by Laws submitted in support of its petition were not properly
been immediately filed. The applicant union must firstly comply with the "20% signature" acknowledged and notarized.
requirement and all the other requisites enumerated in Article 234. Moreover, since under Article
235 the BLR shall act on any application for registration within thirty (30) days from its filing, the Petitioner filed a Petition seeking the cancellation of the Union's registration on the grounds of
likelihood is remote that, assuming the union complied with all the requirements, the application fraud and falsification, Motion was likewise filed by petitioner with the Med-Arbiter requesting
would be approved on the same day it was filed. suspension of proceedings in the certification election case until after the prejudicial question of
the Union's legal personality is determined in the proceedings for cancellation of registration.
ii. Action on Application and Denial of Registration
7
However, in an Order. Med-Arbiter Rasidali C. Abdullah directed the holding of a certification
election among petitioner's rank and file employees. A more than cursory reading of the aforecited provisions clearly indicates that the requirements
embodied therein are intended as preventive measures against the commission of fraud. After a
MED-ARBITER labor organization has filed the necessary papers and documents for registration, it becomes
The Order explained Sumasaklaw sa Manggagawa ng Pizza Hut is a legitimate labor mandatory for the Bureau of Labor Relations to check if the requirements under Article 234 have
organization in contemplation of law and shall remain as such until its very charter certificate is been sedulously complied with. If its application for registration is vitiated by falsification and
canceled or otherwise revoked by competent authority. The alleged misrepresentation, fraud serious irregularities, especially those appearing on the face of the application and the
and false statement in connection with the issuance of the charter certificate are collateral issues supporting documents, a labor organization should be denied recognition as a legitimate labor
which could be properly ventilated in the cancellation proceedings. organization. And if a certificate of recognition has been issued, the propriety of the labor
organization's registration could be assailed directly through cancellation of registration
DOLE USEC proceedings in accordance with Articles 238 and 239 of the Labor Code, or indirectly, by
Denied appeal challenging its petition for the issuance of an order for certification election.

ISSUE: These measures are necessary — and may be undertaken simultaneously — if the spirit behind
Whether or not DOLE USEC committed GAD in affirming Med-Arbiter decision the Labor Code's requirements for registration are to be given flesh and blood. Registration
considering that: (1) respondent Union's legal personality was squarely put in issue; (2) requirements specifically afford a measure of protection to unsuspecting employees who may be
allegations of fraud and falsification, supported by documentary evidence were made; lured into joining unscrupulous or fly-by-night unions whose sole purpose is to control union
and (3) a petition to cancel respondent Union's registration is pending with the regional funds or use the labor organization for illegitimate ends. 12 Such requirements are a valid
office of the Department of Labor and Employment exercise of the police power, because the activities in which labor organizations, associations
and unions of workers are engaged directly affect the public interest and should be protected.
HELD:
YES. Furthermore, the Labor Code itself grants the Bureau of Labor Relations a period of thirty (30)
In the public respondent's assailed Resolution dated December 29, 1993, the suggestion is days within which to review all applications for registration. Article 235 provides:
made that once a labor organization has filed the necessary documents and papers and the
same have been certified under oath and attested to, said organization necessarily becomes Art. 235. Action on application. — The Bureau shall act on all applications for registration within
clothed with the character of a legitimate labor organization. thirty (30) days from filing.

In the first place, the public respondent's views as expressed in his December 29, 1993 All requisite documents and papers shall be certified under oath by the secretary or the treasurer
Resolution miss the entire point behind the nature and purpose of proceedings leading to the of the organization, as the case may be, and attested to by its president.
recognition of unions as legitimate labor organizations. Article 234 of the Labor Code provides:
The thirty-day period in the aforecited provision ensures that any action taken by the Bureau of
Art. 234. Requirements of registration. — Any applicant labor organization, association or group Labor Relations is made in consonance with the mandate of the Labor Code, which, it bears
of unions or workers shall acquire legal personality and shall be entitled to the rights and emphasis, specifically requires that the basis for the issuance of a certificate of registration
privileges granted by law to legitimate labor organizations upon issuance of the certificate of should be compliance with the requirements for recognition under Article 234. Since, obviously,
registration based on the following requirements: recognition of a labor union or labor organization is not merely a ministerial function, the
question now arises as to whether or not the public respondent committed grave abuse of
(a) Fifty pesos (P50.00) registration fee; discretion in affirming the Med-Arbiter's order in spite of the fact that the question of the Union's
legitimacy was squarely put in issue and that the allegations of fraud and falsification were
(b) The names of its officers, their addresses, the principal address of the labor organization, the adequately supported by documentary evidence.
minutes of the organizational meetings and the list of the workers who participated in such
meetings; It cannot be denied that the grounds invoked by petitioner for the cancellation of respondent
Union's registration fall under paragraph (a) and (c) of Article 239 of the Labor Code, to wit:
(c) The names of all its members comprising at least twenty percent (20%) of all the employees
in the bargaining unit where it seeks to operate; (a) Misrepresentation, false statement or fraud in connection with the adoption or ratification of
the constitution and by-laws or amendments thereto, the minutes of ratification, the list of
(d) If the applicant union has been in existence for one or more years, copies of its annual members who took part in the ratification of the constitution and by-laws or amendments thereto,
financial reports; and the minutes of ratification, the list of members who took part in the ratification;

(e) Four (4) copies of the constitution and by-laws of the applicant union, minutes of its adoption xxx xxx xxx
or ratification, and the list of the members who participated in it.
8
(c) Misrepresentation, false statements or fraud in connection with the election of officers, (a) The names of the chapter's officers, their addresses, and the principal office of the chapter;
minutes of the election of officers, the list of voters, or failure to submit these documents and
together with the list of the newly elected-appointed officers and their postal addresses within (b) The chapter's constitution and by-laws: Provided, That where the chapter's constitution and
thirty (30) days from election. by-laws are the same as that of the federation or the national union, this fact shall be indicated
accordingly.
The grounds ventilated in cancellation proceedings in accordance with Article 239 of the Labor The additional supporting requirements shall be certified under oath by the secretary or
Code constitute a grave challenge to the right of respondent Union to ask for certification treasurer of the chapter and attested by its president.
election. The Med-Arbiter should have looked into the merits of the petition for cancellation
before issuing an order calling for certification election. Registration based on false and San Miguel Corporation (Mandaue Packaging Products Plants) v. Mandaue Packing
fraudulent statements and documents confer no legitimacy upon a labor organization irregularly Products Plants – San Miguel Packaging Products – San Miguel Corporation Monthlies
recognized, which, at best, holds on to a mere scrap of paper. Under such circumstances, the Rank-and-File Union – FFW (MPPP-SMPP-SMAMRFU-FFW), G.R. No. 152356, August 16,
labor organization, not being a legitimate labor organization, acquires no rights, particularly the 2005.
right to ask for certification election in a bargaining unit.
FACTS
it would have been more prudent for the Med-Arbiter and public respondent to have granted Respondent, identifying itself as an affiliate of Federation of Free Workers (FFW), filed a petition
petitioner's request for the suspension of proceedings in the certification election case, until the for certification election with the DOLE Regional Office No. VII. In the petition, respondent stated
issue of the legality of the Union's registration shall have been resolved. Failure of the Med- that it sought to be certified and to represent the permanent rank-and-file monthly paid
Arbiter and public respondent to heed the request constituted a grave abuse of discretion. employees of the petitioner.2 The following documents were attached to the petition: (1) a
NOTE: case was REMANDED to the Med-Arbiter to resolve with reasonable dispatch Charter Certificate issued by FFW on 5 June 1998 certifying that respondent as of that date was
petitioner's petition for cancellation of respondent Union's registration duly certified as a local or chapter of FFW; (2) a copy of the constitution of respondent prepared
by its Secretary, Noel T. Bathan and attested by its President, Wilfred V. Sagun; (3) a list of
iii. Effect of Registration, Non-registration, and respondent’s officers and their respective addresses, again prepared by Bathan and attested by
Acquisition of Legal Personality Sagun; (4) a certification signifying that respondent had just been organized and no amount had
yet been collected from its members, signed by respondent’s treasurer Chita D. Rodriguez and
ART. 240. [234] Requirements of Registration.183 A federation, national union or industry or attested by Sagun; and (5) a list of all the rank-and-file monthly paid employees of the Mandaue
trade union center or an independent union shall acquire legal personality and shall be entitled Packaging Products Plants and Mandaue Glass Plant prepared by Bathan and attested by
to the rights and privileges granted by law to legitimate labor organizations upon issuance of the Sagun.
certificate of registration based on the following requirements:
(a) Fifty pesos (P50.00) registration fee; Petitioner filed a motion to dismiss the petition for certification election on the sole ground that
(b) The names of its officers, their addresses, the principal address of the labor organization, the herein respondent is not listed or included in the roster of legitimate labor organizations based
minutes of the organizational meetings and the list of the workers who participated in such on the certification issued by the Officer-In-Charge, Regional Director of the DOLE Regional
meetings; Office No. VII.
(c) In case the applicant is an independent union, the names of all its members comprising at
least twenty percent (20%) of all the employees in the bargaining unit where it seeks to operate; Respondent submitted to the Bureau of Labor Relations the same documents earlier attached to
(d) If the applicant union has been in existence for one or more years, copies of its annual its petition for certification. The accompanying letter, signed by respondent’s president Sagun,
financial reports; and stated that such documents were submitted in compliance with the requirements for the creation
(e) Four copies of the constitution and by-laws of the applicant union, minutes of its adoption or of a local/chapter pursuant to the Labor Code and its Implementing Rules; and it was hoped that
ratification, and the list of the members who participated in it. the submissions would facilitate the listing of respondent under the roster of legitimate labor
organizations. the Chief of Labor Relations Division of DOLE Regional Office No. VII issued a
ART. 241. [234-A] Chartering and Creation of a Local Chapter.184 A duly registered federation Certificate of Creation of Local/Chapter certifying that from 30 July 1998, respondent has
or national union may directly create a local chapter by issuing a charter certificate indicating the acquired legal personality as a labor organization/worker’s association, it having submitted all
establishment of the local chapter. The chapter shall acquire legal personality only for purposes the required documents.
of filing a petition for certification election from the date it was issued a charter certificate.
182 Renumbered as Arts. 264, 265, and 268, respectively. Respondent instead filed a Position Paper wherein it asserted that it had complied with all the
necessary requirements for the conduct of a certification election, and that the ground relied
183 As amended by Sec. 1 of R.A. No. 9481 (2007), An Act Strengthening the Workers' upon in the Motion to Dismiss was a mere technicality
Constitutional Right to Self-Organization. 184 Inserted as a new provision pursuant to Sec. 2 of
R.A. No. 9481 (2007). Petitioner filed a Comment, wherein it reiterated that respondent was not a legitimate labor
The chapter shall be entitled to all other rights and privileges of a legitimate labor organization organization at the time of the filing of the petition. Petitioner also propounded that contrary to
only upon the submission of the following documents in addition to its charter certificate: respondent’s objectives of establishing an organization representing rank-and-file employees,
9
two of respondent’s officers, namely Vice-President Emannuel L. Rosell and Secretary Bathan, Treasurer of the local/chapter and attested by the President, which vests legal personality in the
were actually supervisory employees. In support of this allegation, petitioner attached various local/chapter, which is then free to file on its own a petition for certification election.
documents evidencing the designation of these two officers in supervisory roles, as well as their
exercise of various supervisory functions.9 Petitioner cited Article 245 of the Labor Code, which In this case, the federation in question, the FFW, did not submit any of these documentary
provides that supervisory employees shall not be eligible for membership in a labor organization requirements to the Regional Office or Bureau. It did however issue a charter certificate to the
of the rank-and-file employees. putative local/chapter (herein respondent). Respondent then submitted the charter certificate
along with the other documentary requirements to the Regional Office, but not for the specific
Petitioner’s petition to cancel union registration was denied and CA denied their appeal. The purpose of creating the local/chapter, but for filing the petition for certification election.
decision became final and executory. Despite this Med-Arbiter Manit issued an Order dismissing
respondent’s petition for certification election. The sole ground relied upon for the dismissal was It could be properly said that at the exact moment respondent was filing the petition for
the Med-Arbiter’s Opinion that as of the date of filing of the petition on 15 June 1998, respondent certification, it did not yet possess any legal personality, since the requisites for acquisition of
did not have the legal personality to file the said petition for certification election. legal personality under Section 3, Rule VI of Department Order No. 9 had not yet been complied
with. It could also be discerned that the intention of the Labor Code and its Implementing Rules
DOLE USEC that only those labor organizations that have acquired legal personality are capacitated to file
Respondent appealed to DOLE. DOLE USEC Rosalinda Dimapilis-Baldoz reversed ruling that petitions for certification elections. Such is the general rule.
respondent acquired legal personality as early as 15 June 1998, the date it submitted the
required documents, citing Section 3, Rule VI of the New Rules Implementing the Labor Code Yet there are peculiar circumstances in this case that allow the Court to rule that respondent
(Implementing Rules) which deems that a local/chapter acquires legal personality from the date acquired the requisite legal personality at the same time it filed the petition for certification
of filing of the complete documentary requirements as mandated in the Implementing Rules. The election. In doing so, the Court acknowledges that the strict letter of the procedural rule was not
DOLE also ruled that the contention that two of respondent’s officers were actually supervisors complied with. However, labor laws are generally construed liberally in favor of labor, especially
can be threshed out in the pre-election conferences where the list of qualified voters is to be if doing so affirms the constitutionally guaranteed right to self-organization.
determined.
True enough, there was no attempt made by the national federation, or the local/chapter for that
ISSUE: matter, to submit the enumerated documentary requirements to the Regional Office or Bureau
Whether or not the union acquired legal personality for the specific purpose of creating the local/chapter. However, these same documents were
submitted by the local/chapter to the Regional Office as attachments to its petition for
RULING: certification election. Under Section 3, Rule VI of Department Order No. 9, it is the submission of
YES these same documents to the Regional Office or Bureau that operates to vest legal personality
At the onset, the arguments raised by petitioner on this point are plainly erroneous. Petitioner on the local/chapter.
cites the case of Toyota Motor Philippines v. Toyota Motor Philippines Corporation Labor
Union,38 and the purported holding therein that "[if] it is true that at the time of the filing of the Thus, in order to ascertain when respondent acquired legal personality, we only need to
petition, the said registration certificate has not been approved yet, then, petitioner lacks the determine on what date the Regional Office or Bureau received the complete documentary
legal personality to file the petition." However, an examination of the case actually reveals that requirements enumerated under Section 1, Rule VI of Department Order No. 9. There is no
the cited portion was lifted from one of the antecedent rulings of the Med-Arbiter in that case doubt that on 15 June 1998, or the date respondent filed its petition for certification election,
which had not even been affirmed or reinstated by the Court on review.40 Moreover, such attached thereto were respondent’s constitution, the names and addresses of its officers, and
pronouncement made prior to the enactment of Department Order No. 9 squarely contradicts the charter certificate issued by the national union FFW. The first two of these documents were
Section 3, Rule VI thereof, which provides that legal personality of the local/chapter is vested duly certified under oath by respondent’s secretary Bathan and attested to by president Sagun
upon the submission of the complete documentary requirements.
It may be noted though that respondent never submitted a separate by-laws, nor does it appear
It is also worth noting that petitioner union in Toyota was an independent labor union, and not a that respondent ever intended to prepare a set thereof. Section 1(c), Rule VI, Book V of
local/chapter, and under Department Order No. 9, independent labor unions, unlike Department Order No. 9 provides that the submission of both a constitution and a set of by-laws
local/chapters, acquire legal personality only upon issuance of the certificate of registration by is required, or at least an indication that the local/chapter is adopting the constitution and by-
the Bureau or Regional Office. laws of the federation or national union.

In regular order, it is the federation or national union, already in possession of legal personality, An examination of respondent’s constitution reveals it sufficiently comprehensive in establishing
which initiates the creation of the local/chapter. It issues a charter certificate indicating the the necessary rules for its operation.
creation or establishment of the local/chapter. It then submits this charter certificate, along with Article IV establishes the requisites for membership in the local/chapter.
the names of the local/chapter’s officers, constitution and by-laws to the Regional Office or Articles V and VI name the various officers and what their respective functions are. The
Bureau. It is the submission of these documents, certified under oath by the Secretary or procedure for election of these officers, including the necessary vote requirements, is provided
for in Article IX, while Article XV delineates the procedure for the impeachment of these officers.
10
Article VII establishes the standing committees of the local/chapter and how their members are
appointed. Sec. 1 —. . . Employees who are already members of the UNION at the time of the signing of
Article VIII lays down the rules for meetings of the union, including the notice and quorum this Agreement or who become so thereafter shall be required to maintain their membership
requirements thereof. therein as a condition of continued employment.
Article X enumerates with particularity the rules for union dues, special assessments, fines, and
other payments. Sec. 3—Any employee who is expelled from the UNION for joining another federation or forming
Article XII provides the general rule for quorum in meetings of the Board of Directors and of the another union, or who fails or refuses to maintain his membership therein as required, . . . shall,
members of the local/chapter, and cites the applicability of the Robert’s Rules of Order43 in its upon written request of the UNION be discharged by the COMPANY. (Rollo, pp. 667-670)
meetings.
Article XVI governs and institutes the requisites for the amendment of the constitution. And attached to the Agreement as Appendix "A" is a check-off Authorization Form, the terms of
which are as follows:
The Court likewise sees no impediment in deeming respondent as having acquired legal
personality as of 15 June 1998, the fact that it was the local/chapter itself, and not the FFW, We, the undersigned, hereby designate the NATIONAL Association of Trade Unions, of which
which submitted the documents required under Section 1, Rule VI of Department Order No. 9.In the TROPICAL HUT EMPLOYEES UNION is an affiliate as sole collective bargaining agent in all
the ordinary course, it should have been FFW, and not respondent, which should have matters relating to salary rates, hours of work and other terms and conditions of employment in
submitted the subject documents to the Regional Office. Nonetheless, there is no good reason the Tropical Hut Food Market, Inc. and we hereby authorize the said company to deduct the
to deny legal personality or defer its conferral to the local/chapter if it is evident at the onset that amount of Four (P 4.00) Pesos each every month as our monthly dues and to deliver the
the federation or national union itself has already through its own means established the amount to the Treasurer of the Union or his duly authorized representatives.
local/chapter.
On May 21, 1971, respondent company and THEU-NATU entered into a new Collective
Tropical Hut Employees’ Union-CGW v. Tropical Hut Food Market, Inc., G.R. Nos. L-43495- Bargaining Agreement which ended on March 31, 1974. This new CBA incorporated the
99, January 20, 1990 previous union-shop security clause and the attached check-off authorization form.

FACTS Sometime in July, 1973, Arturo Dilag, incumbent President of THEU-NATU, was appointed by
On January 2, 1968, the rank and file workers of the Tropical Hut Food Market Incorporated, the respondent company as Assistant Unit Manager. On July 24, 1973, he wrote the general
organized a local union called the Tropical Hut Employees Union(THEU), elected their officers, membership of his union that for reason of his present position, he was resigning as President of
adopted their constitution and by-laws and immediately sought affiliation with the National the THEU-NATU effective that date. As a consequence thereof, his Vice-President, Jose
Association of Trade Unions (NATU). Encinas, assumed and discharged the duties of the presidency of the THEU-NATU.
The NATU accepted the THEU application for affiliation. Following such affiliation with NATU,
Registration Certificate was issued by the Department of Labor in the name of the Tropical Hut On December 19,1973, NATU received a letter dated December 15, 1973, jointly signed by the
Employees Union — NATU. It appears, however, that NATU itself as a labor federation, was not incumbent officers of the local union informing the NATU that THEU was disaffiliating from the
registered with the Department of Labor NATU federation. On December 20, 1973, the Secretary of the THEU, Nemesio Barro, made an
announcement in an open letter to the general membership of the THEU, concerning the latter's
After several negotiations were conducted between THEU-NATU, represented by its local disaffiliation from the NATU and its affiliation with the Confederation of General Workers (CGW)
president and the national officers of the NATU, particularly Ignacio Lacsina, President, Pacifico
Rosal, Executive Vice-President and Marcelino Lontok, Jr., Vice President, and respondent On January 9, 1974, Pacifico Rosal, President of the Confederation of General Workers (CGW),
Tropical Hut Food Market, Incorporated, thru its President and General Manager, Cesar Azcona, wrote a letter in behalf of complainant THEU-CGW to the respondent company demanding the
Sr., a Collective Bargaining Agreement was concluded between the parties on April 1, 1968, the remittance of the union dues collected by the Tropical Hut Food Mart, Incorporated to the THEU-
term of which expired on March 31, 1971. CGW, but this was refused by the respondent company.

The CBA included the following stipulations: The then so-called THEU-CGW held its annual election of officers, with Jose Encinas elected as
President. On January 3, 1974, Encinas, in his capacity as THEU-CGW President, informed the
Article I respondent company of the result of the elections. On January 9, 1974, Pacifico Rosal,
Coverage and Effectivity President of the Confederation of General Workers (CGW), wrote a letter in behalf of
Sec. 1. The COMPANY recognizes the UNION as the sole and exclusive collective bargaining complainant THEU-CGWto the respondent company demanding the remittance of the union
agent for all its workers and employees in all matters concerning wages, hours of work, and dues collected by the Tropical Hut Food Mart, Incorporated to the THEU-CGW, but this was
other terms and conditions of employment. refused by the respondent company.
A request made by the NATU federation to the respondent company to dismiss him (Encinas)
Article III in view of his violation of Section 3 of Article III of the Collective Bargaining Agreement.
Union Membership and Union Check-off
11
The respondent company applied for clearance with the Secretary of Labor to dismiss the other embodied in the agreements cannot be used to justify the dismissals meted to petitioners since
officers and members of THEU-CGW. The company also suspended them effective that day. it is not applicable to the circumstances obtaining in this case. The CBA imposes dismissal only
NATU received a letter jointly signed by the incumbent officers of the local union informing the in case an employee is expelled from the union for joining another federation or for forming
NATU that THEU was disaffiliating from the NATU federation. The employees were dismissed another union or who fails or refuses to maintain membership therein. The case at bar does not
because, as respondent company contended, they violated the union security clause. involve the withdrawal of merely some employees from the union but of the whole THEU itself
from its federation. Clearly, since there is no violation of the union security provision in the CBA,
Not properly discussed in full text of the case but certain employees were dismissed due to the there was no sufficient ground to terminate the employment of petitioners. Petition granted
disaffiliation from NATU supposedly due to CBA violation
iv. Rights of Legitimate Labor Organization
LABOR ARBITER ART. 251. [242] Rights of Legitimate Labor Organizations.193 A legitimate labor organization
Ordered reinstatement of dismissed employees shall have the right:
(a) To act as the representative of its members for the purpose of collective bargaining;
NLRC (b) To be certified as the exclusive representative of all the employees in an appropriate
Granted appeal of private respondent, ordered the respondent company, under pain of being bargaining unit for purposes of collective bargaining;
cited for contempt for failure to do so, to give the individual complainants a second chance by (c) To be furnished by the employer, upon written request, with its annual audited financial
reemploying them upon their voluntary reaffirmation of membership and loyalty to the Tropical statements, including the balance sheet and the profit and loss statement, within thirty (30)
Hut Employees Union-NATU and the National Association of Trade Unions in the event it hires calendar days from the date of receipt of the request, after the union has been duly recognized
additional personnel. by the employer or certified as the sole and exclusive bargaining representative of the
employees in the bargaining unit, or within sixty (60) calendar days before the expiration of the
DOLE SEC: Affirmed NLRC existing collective bargaining agreement, or during the collective bargaining negotiation;
(d) To own property, real or personal, for the use and benefit of the labor organization and its
ISSUES: members;
Whether THEU-CGW’s disaffiliation was valid and whether they lost legal personality by (e) To sue and be sued in its registered name; and
disaffiliating with NATU (f) To undertake all other activities designed to benefit the organization and its members,
including cooperative, housing, welfare and other projects not contrary to law.
HELD: Notwithstanding any provision of a general or special law to the contrary, the income and the
YES. The right of a local union to disaffiliate from its mother federation is well-settled. A local properties of legitimate labor organizations, including grants, endowments, gifts, donations and
union, being a separate and voluntary association, is free to serve the interest of all its members contributions they may receive from fraternal and similar organizations, local or foreign, which
including the freedom to disaffiliate when circumstances warrant. This right is consistent with the are actually, directly and exclusively used for their lawful purposes, shall be free from taxes,
constitutional guarantee of freedom of association. duties and other assessments. The exemptions provided herein may be withdrawn only by a
special law expressly repealing this provision.
All employees enjoy the right to self organization and to form and join labor organizations of their
own choosing for the purpose of collective bargaining and to engage in concerted activities for Baptista v. Villanueva, G.R. No. 194709, July 31, 2013.
their mutual aid or protection. This is a fundamental right of labor that derives its existence from
the Constitution. In interpreting the protection to labor and social justice provisions of the FACTS
Constitution and the labor laws or rules or regulations, We have always adopted the liberal Petitioners were former union members of Radio Philippines Network Employees Union
approach which favors the exercise of labor rights. (RPNEU), a legitimate labor organization and the sole and exclusive bargaining agent of the
rank and file employees of Radio Philippines Network (RPN), a government-sequestered
Further, there is no merit in the contention of the respondents that the act of disaffiliation violated corporation involved in commercial radio and television broadcasting affairs
the union security clause of the CBA and that their dismissal as a consequence thereof is valid.
A perusal of the collective bargaining agreements shows that the THEU-NATU, and not the Respondents were the union’s elected officers and members
NATU federation, was recognized as the sole and exclusive collective bargaining agent for all its
workers and employees in all matters concerning wages, hours of work and other terms and On suspicion of union mismanagement, petitioners, together with some other union members,
conditions of employment Although NATU was designated as the sole bargaining agent in the filed a complaint for impeachment of their union president, Reynato Siozon, before the executive
check-off authorization form attached to the CBA, this simply means it was acting only for and in board of RPN, which was eventually abandoned. They later re-lodged the impeachment
behalf of its affiliate. The NATU possessed the status of an agent while the local union remained complaint, this time, against all the union officers and members of RPNEU before the
the basic principal union which entered into contract with the respondent company. Department of Labor and Employment (DOLE). They likewise filed various petitions for audit
covering the period from 2000 to 2004
When the THEU disaffiliated from its mother federation, the former did not lose its legal
personality as the bargaining union under the CBA. Moreover, the union security clause
12
Two written complaints were filed against petitioners and several others for alleged violation of ISSUE:
the union’s Constitution and By-Laws.Months later a different group of union members filed a Whether the union members are guilty of ULP
third complaint against petitioners and 12 others, before the Chairman of RPNEU’s Committee
on Grievance and Investigation citing as grounds the "commission of an act which violates RULING:
RPNEU Constitution and By-Laws, specifically: NO
Article IX, Section 2.2 for joining or forming a union outside the sixty (60) days period The primary concept of ULP is embodied in Article 247 of the Labor Code, which provides:
Article IX, Section 2.5 for urging or advocating that a member start an action in any court of
justice or external investigative body against the Union or its officer without first exhausting all Article 247. Concept of unfair labor practice and procedure for prosecution thereof.––Unfair
internal remedies open to him or available in accordance with the CBL labor practices violate the constitutional right of workers and employees to self-organization, are
inimical to the legitimate interests of both labor and management, including their right to bargain
Petitioners and their group, through an exchange of communications with the Committee, denied collectively and otherwise deal with each other in an atmosphere of freedom and mutual respect,
the charges imputed against them and contested the procedure adopted by the Committee in its disrupt industrial peace and hinder the promotion of healthy and stable labor-management
investigation. The Committee submitted their recommendation of expulsion from the union to relations.
RPNEU’s Board of Directors.RPNEU’s Board of Directors affirmed the recommendation of
expulsion of petitioners and the 12 others from union membership in a Board Resolution. In essence, ULP relates to the commission of acts that transgress the workers’ right to organize.
As specified in Articles 248 and 249 of the Labor Code, the prohibited acts must necessarily
Petitioners with the 12 others wrote to RPNEU’s President and Board of Directors that their relate to the workers' right to self-organization and to the observance of a CBA. Absent the said
expulsion from the union was an ultra vires act because the Committee failed to observe the vital elements, the acts complained, although seemingly unjust, would not constitute ULP
basic elements of due process because they were not given the chance to physically confront
and examine their complainants. In a letter, RPNEU’s officers informed their company of the ART. 249. UNFAIR LABOR PRACTICES OF LABOR ORGANIZATIONS.- It shall be unfair labor
expulsion of petitioners and the 12 others from the union and requested the management to practice for a labor organization, its officers, agents or representatives:
serve them notices of termination from employment in compliance with their CBA’s union
security clause. RPN HRD Manager, Lourdes Angeles, informed petitioners and the 12 others of (a) To restrain or coerce employees in the exercise of their rights to self-organization. However,
the termination of their employment effective March 20, 2006, enforcing Article II, Section 216 a labor organization shall have the right to prescribe its own rules with respect to the acquisition
also known as the union security clause of their current CBA. or retention of membership:

Petitioners filed three complaints for Unfair Labor Practice questioning legality of their expulsion (b) To cause or attempt to cause an employer to discriminate against an employee, including
discrimination against an employee with respect to whom membership in such organization has
LABOR ARBITER been denied or to terminate an employee on any ground other than the usual terms and
Favored petitioners, adjudged the respondents guilty of ULP pursuant to Article 249 (a) and (b) conditions under which membership or continuation of membership is made available to other
of the Labor Code. The LA clarified that only the union officers of RPNEU could be held members;
responsible for ULP, so they exonerated six (6) of the original defendants who were mere union
members. The LA also ordered the reinstatement of petitioners as bonafide members of Based on RPNEU’s Constitution and By-Laws, the charges against petitioners were not mere
RPNEU. internal squabbles, but violations that demand proper investigation because, if proven, would
constitute grounds for their expulsion from the union. As such, Article X, Investigation
NLRC Procedures and Appeal Process of RPNEU’s Constitution and By-Laws, which reads –
Set aside LA decision. NLRC found that petitioners filed a suit calling for the impeachment of the
officers and members of the Executive Board of RPNEU without first resorting to internal SECTION 1. Charge against any member or officer of the Union shall be submitted to the Board
remedies available under its own Constitution and By-Laws. The NLRC likewise decreed that the of Directors (BOD) in writing, which shall refer the same, if necessary, to the committee on
LA’s order of reinstatement was improper because the legality of the membership expulsion was Grievance and Investigation. The Committee shall hear any charge and subsequently, forward
not raised in the proceedings and, hence, beyond the jurisdiction of the LA its finding and recommendation to the BOD. The BOD has the power to approve or nullify the
recommendation of the Committee on Grievance and Investigation based on the merit of the
CA appeal.
Affirmed NLRC. CA stated that the termination of employment by virtue of a union security was correctly applied under the circumstances.
clause was recognized in our jurisdiction. It explained that the said practice fortified the union
and averted disunity in the bargaining unit within the duration of the CBA. The CA declared that It is well-settled that workers’ and employers’ organizations shall have the right to draw up their
petitioners were accorded due process before they were removed from office. In fact, petitioners constitutions and rules to elect their representatives in full freedom, to organize their
were given the opportunity to explain their case and they actually availed of said opportunity by administration and activities and to formulate their programs.In this case, RPNEU’s Constitution
submitting letters containing their arguments and By-Laws expressly mandate that before a party is allowed to seek the intervention of the
court, it is a pre-condition that he should have availed of all the internal remedies within the
13
organization. Petitioners were found to have violated the provisions of the union’s Constitution ● ICTS is ordered to pay the employees concerned the differentials representing the
and By-Laws when they filed petitions for impeachment against their union officers and for audit underpayment of said salaries and other benefits.
before the DOLE without first exhausting all internal remedies available within their organization. ● Denied petitioners-appellants’ Complaint-in-Intervention with Motion for Intervention
This act is a ground for expulsion from union membership. Thus, petitioners’ expulsion from the upon a finding that they are already well represented by APCWU
union was not a deliberate attempt to curtail or restrict their right to organize, but was triggered
by the commission of an act, expressly sanctioned by Section 2.5 of Article IX of the union’s NLRC’s Ruling:
Constitution and By-Laws. ● Reversed the decision of the LA and dismissed APCWU’s complaint.

Petitioners failed to discharge the burden required to prove the charge of ULP against the CA’s Ruling:
respondents. Aside from their self-serving allegations, petitioners were not able to establish how ● dismissed APCWU’s petition on the following grounds: failure to allege when its
they were restrained or coerced by their union in a way that curtailed their right to self- motion for reconsideration of the NLRC decision was filed, failure to attach the
organization.petitioners had the burden of adducing substantial evidence to support its necessary appendices to the petition, and failure to file its motion for extension to file
allegations of ULP, which burden they failed to discharge. In fact, both the NLRC and the CA its petition within the reglementary period.
found that petitioners were unable to prove their charge of ULP against the respondents ● Records shows that petitioners, claiming to be employees of ICTSI, are already well
represented by its employees union, APCWU.
Acedera v. International Container Terminal Services, Inc. (ICTSI), G.R. No. 146073,
January 13, 2003. ISSUE:
FACTS: Whether or not Acedera et al. have no legal right to intervene in the case as their intervention
Jerry Acedera, Et. Al. are employees of herein private respondent International Container was a superfluity -YES.
Terminal Services, Inc. (ICTSI) and are officers/members of Associated Port Checkers &
Workers Union-International Container Terminal Services, Inc. Local Chapter (APCWU-ICTSI), a HELD:
labor organization duly registered as a local affiliate of the Associated Port Checkers & Workers Petitioner’s contention is based on Rule 19 of the 1997 Rules of Civil Procedure, Section 1.
Union (APCWU). They stress that they have complied with the requisites for intervention because (1) they are the
ones who stand to gain or lose by the direct legal operation and effect of any judgment that may
On September 28, 1990, ICTSI entered into its first Collective Bargaining Agreement (CBA) with be rendered in this case, (2) no undue delay or prejudice would result from their intervention
APCWU with a term of five years effective until September 28, 1995. The CBA was renegotiated since their Complaint-in-Intervention with Motion for Intervention was filed while the Labor
and thereafter renewed through a second CBA that took effect on September 29, 1995, effective Arbiter was still hearing the case and before any decision thereon was rendered, and (3) it was
for another five years. Thus, in accordance with the above-quoted provision of the CBA, the not possible for them to file a separate case as they would be guilty of forum shopping because
employees’ work week was reduced to five days or a total of 250 days a year. ICTSI, however, the only forum available for them was the Labor Arbiter.
continued using the 304-day divisor in computing the wages of the employees.
However, SC ruled that Petitioners failed to consider, in addition to the rule on intervention, the
Regional Tripartite Wage and Productivity Board (RTWPB) in the NCR decreed a P17.00 daily rule on representation, thusly:
wage increase for all workers and employees receiving P125.00 per day or lower in the National Sec. 3. Representatives as parties. — Where the action is allowed to be prosecuted or defended
Capital Region. 7 The then president of APCWU, together with some union members, thus by a representative or someone acting in a fiduciary capacity, the beneficiary shall be included
requested the ICTSI’s Human Resource Department/Personnel Manager to compute the actual in the title of the case and shall be deemed to be the real party in interest. A representative may
monthly increase in the employees’ wages by multiplying the RTWPB mandated increase by 365 be a trustee of an express trust, a guardian, an executor or administrator, or a party authorized
days and dividing the product by 12 months. ICTSI stopped using 304 days as divisor and by law or these Rules.
started using 365 days to determine the daily wage. A labor union is one such party authorized to represent its members under Article 242(a) of the
Labor Code which provides that a union may act as the representative of its members for the
Later on, ICTSI went on a retrenchment program and laid off its on-call employees. This purpose of collective bargaining. This authority includes the power to represent its members for
prompted the APCWU-ICTSI to file a notice of strike which included as cause of action not only the purpose of enforcing the provisions of the CBA. That APCWU acted in a representative
the retrenchment of the employees but also ICTSI’s use of 365 days as divisor in the capacity "for and on behalf of its Union members and other employees similarly situated," the
computation of wages. The dispute respecting the retrenchment was resolved by a compromise title of the case filed by it at the Labor Arbiter’s Office so expressly states.
settlement while that respecting the computation of wages was referred to the Labor Arbiter.
Acedera et al. filed a Motion to Intervene. While a party acting in a representative capacity, such as a union, may be permitted to
intervene in a case, ordinarily, a person whose interests are already represented will not
LA’s Ruling:The correct divisor in computing the daily wage and other labor standard benefits be permitted to do the same except when there is a suggestion of fraud or collusion or
of the employees of ICTSI who are members of the complainant Union as well as the other that the representative will not act in good faith for the protection of all interests
employees similarly situated is 250 days. represented by him.

14
To reiterate, for a member of a class to be permitted to intervene in a representative action,
fraud or collusion or lack of good faith on the part of the representative must be proven. It must ISSUE:
be based on facts borne on record. Mere assertions, as what petitioners-appellants proffer, do Whether or not SMFI is not a legitimate labor organization on the basis of Article 234 of the
not suffice. Labor Code, any labor organization shall acquire legal personality only upon the issuance of the
Certificate of Registration by the Bureau of Labor Relations - NO.
San Miguel Foods, Inc. – Cebu B-MEG Feed Plant v. Lauesma, G.R. No. 116172, October
10, 1996. HELD:
Article 212(h) of the Labor Code defines a legitimate labor organization as "any labor
FACTS: organization duly registered with the Department of Labor and Employment, and includes any
On September 24, 1993, a petition for certification election among the monthly-paid employees branch or local thereof ."
of the San Miguel Food, Inc.-Cebu B-Meg Feeds Plant was filed by labor federation Ilaw at
Buklod ng Mangagawa (IBM) before Med-Arbiter Achilles V. Manit, alleging, that it is a legitimate It is important to determine whether or not a particular labor organization is legitimate since
labor organization duly registered with DOLE. SMFI-Cebu B-Meg Feeds Plant (SMFI), is a legitimate labor organizations have exclusive rights under the law which cannot be exercised by
business entity duly organized and existing under the laws of the Philippines which employs non-legitimate unions, one of which is the right to be certified as the exclusive representative of
roughly 75 monthly paid employees, almost all of whom support the present petition. It was all the employees in an appropriate collective bargaining unit for purposes of collective
submitted in the petition that there has been no certification election conducted in SMFI to bargaining. These rights are found under Article 242.
determine the sole and exclusive bargaining agent thereat for the past 2 years and that the
proposed bargaining unit, which is SMFI's monthly paid employees, is an unorganized one. It Moreover,, a labor organizations attains the status of legitimacy only upon the issuance in its
was also stated therein that IBM has already complied with the mandatory requirements for the name of a Certificate of Registration by the Bureau of Labor Relations pursuant to Articles 234
creation of its local or affiliate in SMFI's establishment. and 235 of the Labor Code. The procedure is not the only way by which a labor union may
become legitimate, however. When an unregistered union becomes a branch, local or chapter of
SMFI filed a Motion to Dismiss on the ground that a similar petition remains pending between a federation, some of the aforementioned requirements for registration are no longer required.
the same parties for the same cause of action before Med-Arbiter Achilles V. Manit. The earlier Section 3, Rule II, Book V of the Implementing Rules of the Labor Code governs the procedure
petition, pertains to the the issuance of an order by the Med-Arbiter allowing the conduct for a for union affiliation.
certification election in SMFI's establishment.
In upholding the legitimate status of IBM at SMFI as mentioned by the SolGen, the contention of
IBM filed its Opposition to SMFI's Motion to Dismiss contending, case referred to by SMFI had the respondent that unless and until the issue on who is the legitimate national president, of the
already been resolved by Med-Arbiter Manit in his Resolution and Order wherein IBM's first Ilaw at Buklod ng Mangagawa is resolved, the petitioner cannot claim that is has a valid charter
petition for certification election was denied mainly due to IBM's failure to comply with certain certificate necessary for it to acquire legal personality is untenable. We wish to stress that the
mandatory requirements of the law. Thus, IBM argues that there having been no similar petition resolution of the said issue will not in any way affect the validity of the charter certificate issued
pending before Med-Arbiter Manit, another petition for certification election may be refiled as by the IBM in favor of the local union. It must be borne in mind that the said charter certificate
soon as the said requirements are met. was issued by the IBM in its capacity as a labor organization, a juridical entity which has a
separate and distinct legal personality from its members. When as in this case, there is no
Med-Arbiter Manit’s Ruling: showing that the Federation acting as a separate entity is questioning the legality of the
● Usec Laguesma, granted the second petition for certification election of private issuance of the said charter certificate, the legality of the issuance of the same in favor of the
respondent IBM. local union is presumed. This, notwithstanding the alleged controversy on the leadership of the
federation.
SMFI’s contention upon appeal:
Med-Arbiter erred in directing the conduct of certification election considering that the local or In affirming the comment of the SolGen, SC ruled that respondent's Comment to the petition
chapter of IBM at SMFI is still not a legitimate labor organization with a right to be certified as the indicates that in the election of officers held to determine the representatives of IBM, the faction
exclusive bargaining agent in petitioner's establishment based on two grounds: (1) the of Mr. Meron lost to the group of Mr. Edilberto Galvez, and the latter was acknowledged as the
authenticity and due execution of the Charter Certificate submitted by IBM in favor of its local at duly elected IBM National President. Thus, the authority of Mr. Galvez to sign the charter
SMFI cannot yet be ascertained as it is still not known who is the legitimate and authorized certificate of IBM at SMFI, as President of the IBM Federation, can no longer be successfully
representative of the IBM Federation who may validly issue said Charter Certificate; and (2) a questioned. A punctilious examination of the records presents no evidence to the contrary and
group of workers or a local union shall acquire legal personality only upon the issuance of a petitioner, instead of squarely refuting this point, skirted the issue by insisting that the mere
Certificate of Registration by the Bureau of Labor Relations under Article 234 of the Labor Code, presence of two contending factions in the IBM prevents the issuance of a valid and authentic
which IBM at SMFI did not possess. charter certificate in favor of IBM at SMFI.

Secretary of Labor and Employment’s Ruling: Lastly,, the certification election sought to be stopped by petitioner is fait accompli. The monthly
● Denied the appeal paid rank-and-file employees of SMFI have already articulated their choice as to who their
15
collective bargaining agent should be. In the certification election held on August 20, 1994, the (b) Its list of officers, minutes of the election of officers, and list of voters within thirty (30) days
SMFI workers chose IBM at SMFI to be their sole and exclusive bargaining agent. This from election;
democratic decision deserve utmost respect. Again, it bears stressing that labor legislation
seeks in the main to protect the interest of the members of the working class. It should never be (c) Its annual financial report within thirty (30) days after the close of every fiscal year; and
used to subvert their will.
(d) Its list of members at least once a year or whenever required by the Bureau.
b. Cancellation of Union Certificate of Registration
• Articles 245; 246; 247; 248; 250(j), last paragraph; 252; and 256, Labor Code.
Failure to comply with the above requirements shall not be a ground for cancellation of union
registration but shall subject the erring officers or members to suspension, expulsion from
Article 245: [238] Cancellation of Registration.186 The certificate of registration of any
membership, or any appropriate penalty.
legitimate labor organization, whether national or local, may be cancelled by the Bureau, after
due hearing, only on the grounds specified in Article 239 hereof.
Article 256: [245-A] Effect of Inclusion as Members of Employees Outside the Bargaining
Unit.198 The inclusion as union members of employees outside the bargaining unit shall not be
Article 246: [238-A] Effect of a Petition for Cancellation of Registration.188 A petition for
a ground for the cancellation of the registration of the union. Said employees are automatically
cancellation of union registration shall not suspend the proceedings for certification election nor
deemed removed from the list of membership of said union.
shall it prevent the filing of a petition for certification election.

Takata (Philippines) Corporation v. Bureau of Labor Relations, G.R. No. 192676, June 4,
In case of cancellation, nothing herein shall restrict the right of the union to seek just and
2014.
equitable remedies in the appropriate courts.
FACTS:
Article 247: [239] Grounds for Cancellation of Union Registration.189 The following may On July 7, 2009, Takata Corp filed with DOLE a petition for cancellation of Certificate of Union
constitute grounds for cancellation of union registration: Registration of Samahang Lakas Manggagawa ng Takata (SALAMAT) on the ground that was
guilty of misrepresentation, false statements, and fraud with respect to the number of those who
(a) Misrepresentation, false statement or fraud in connection with the adoption or ratification of participated in their organizational meeting, the adoption and ratification of its Constitution and
by-laws, and in the election of its officers.
the constitution and by-laws or amendments thereto, the minutes of ratification, and the list of
members who took part in the ratification; Takata’s contention:
- In the May 1, 2009 organizational meeting of respondent, only 68 attendees signed
(b) Misrepresentation, false statements or fraud in connection with the election of officers, the attendance sheet, and which number comprised only 17% of the total number of
minutes of the election of officers, and the list of voters; the 396 regular rank- and-file employees which respondent sought to represent, and
hence, respondent failed to comply with the 20% minimum membership requirement;
- The document “Pangalan ng mga Kasapi ng Unyon” bore no signatures of the alleged
(c) Voluntary dissolution by the members. 119 union members;
- The employees were not given sufficient information on the documents they signed;
Article 248: [239-A] Voluntary Cancellation of Registration.190 The registration of a legitimate - The document “Sama-Samang Pahayag ng Pagsapi” was not submitted at the time of
labor organization may be cancelled by the organization itself: Provided, That at least two-thirds the filing of respondent's application for union registration; that the 119 union
of its general membership votes, in a meeting duly called for that purpose to dissolve the members were actually only 117;
- The total number of petitioner's employees as of May 1, 2009 was 470, and not 396
organization: Provided, further, That an application to cancel registration is thereafter submitted
as respondent claimed.
by the board of the organization, attested to by the president thereof.
SALAMAT’s contention:
Article 252: [242-A] Reportorial Requirements.194 The following are documents required to be - Denied the charge and claimed that the 119 union members were more than the 20%
submitted to the Bureau by the legitimate labor organization concerned: requirement for union registration.
- The document “Sama-Samang Pahayag ng Pagsapi sa Unyon” which it presented in
its petition for certification election supported their claim of 119 members.
193 As amended by Sec. 17 of R.A. No. 6715 (1989).194 As amended by Sec. 7 of R.A. No. - Takata was estopped from assailing its legal personality as it agreed to a certification
9481 (2007). election and actively participated in the pre-election conference of the certification
election proceedings.
(a) Its constitution and by-laws, or amendments thereto, the minutes of ratification, and the list of - The union members were informed of the contents of the documents they signed and
that the 68 attendees to the organizational meeting constituted more than 50% of the
members who took part in the ratification of the constitution and by-laws within thirty (30) days
total union membership, hence, a quorum existed for the conduct of the said meeting.
from adoption or ratification of the constitution and by-laws or amendments thereto; DOLE:

16
● Granted petition for cancellation of respondent's certificate of registration. Thus, the as union members, thus respondent sufficiently complied even beyond the 20% minimum
Union Certificate of Registration is revoked and/or cancelled and SALAMAT is delisted membership requirement. Respondent also submitted the attendance sheet of the organizational
from the roll of legitimate labor organization of this office. meeting which contained the names and signatures of the 68 union members who attended the
● 68 employees who attended the organizational meeting was obviously less than 20%
meeting. Considering that there are 119 union members which are more than 20% of all the
of the total number of 396 regular rank-and-file employees which respondent sought
to represent, hence, short of the union registration requirement; employees of the bargaining unit, and since the law does not provide for the required number of
● attendance sheet which contained the signatures and names of the union members members to attend the organizational meeting, the 68 attendees which comprised at least the
totalling to 68 contradicted the list of names stated in the document denominated as majority of the 119 union members would already constitute a quorum for the meeting to
“Pangalan ng mga Kasapi ng Unyon.” proceed and to validly ratify the Constitution and By-laws of the union. Thus, there’s, no basis for
● The document “Sama-Samang Pahayag ng Pagsapi” was not attached to the petitioner to contend that grounds exist for the cancellation of respondent's union registration.
application for registration as it was only submitted in the petition for certification For fraud and misrepresentation to be grounds for cancellation of union registration under Article
election filed by respondent at a later date.
239 of the Labor Code, the nature of the fraud and misrepresentation must be grave and
● The proceedings in the cancellation of registration and certification elections are two
different and entirely separate and independent proceedings which were not compelling enough to vitiate the consent of a majority of union members.
dependent on each other.
The Heritage Hotel Manila v. National Union of Workers in the Hotel, Restaurant and Allied
Bureau of Labor Relations’s Ruling: Industries – Hertiage Hotel Manila Supervisors Chapter (NUWHRAINHHMSC), G.R. No.
● Reversed the decision of DOLE and ordered that SALAMAT shall remain in the roster 178296, January 12, 2011.
of labor organizations.
● Takata failed to prove that respondent deliberately and maliciously misrepresented the
FACTS:
number of rank-and-file employees. It pointed out petitioner's basis for the alleged
non-compliance with the minimum membership requirement for registration was the On October 11, 1995, respondent filed with the DOLE a petition for certification election. The
attendance of 68 members to the May 1, 2009 organizational meeting supposedly Med-Arbiter granted the petition on February 14, 1996 and ordered the holding of a certification
comprising only 17% of the total 396 regular rank-and-file employees. election. On appeal, the DOLE Secretary, in a Resolution dated August 15, 1996, affirmed the
● The list of employees who participated in the organizational meeting was a separate Med-Arbiter’s order and remanded the case to the Med-Arbiter for the holding of a preelection
and distinct requirement from the list of the names of members comprising at least conference on February 26, 1997. Petitioner filed a motion for reconsideration, but it was denied
20% of the employees in the bargaining unit; and that there was no requirement for on September 23, 1996.
signatures opposite the names of the union members; and there was no evidence
showing that the employees assailed their inclusion in the list of union members.
The preelection conference was not held as initially scheduled; Petitioner moved for the
CA: Denied the petition and affirmed the decision of the BLR. dismissal on the petition due to alleged repeated non-appearance of respondent. The latter
agreed to suspend proceedings until further notice. The preelection conference resumed on
ISSUE: January 29, 2000. Later on, Petitioner discovered that respondent had failed to submit to the
Whether or not SALAMAT’s Certificate of Union Registration should be cancelled -NO. Bureau of Labor Relations (BLR) its annual financial report for several years and the list of its
members since it filed its registration papers in 1995. Thus, petitioner filed a Petition for
HELD: Cancellation of Registration of respondent, on the ground of the non-submission of the said
Petitioner's allegation of misrepresentation and fraud is based on its claim that during the documents. Petitioner prayed that respondent’s Certificate of Creation of Local/Chapter be
organizational meeting on May 1, 2009, only 68 employees attended, while respondent claimed cancelled and its name be deleted from the list of legitimate labor organizations. It further
that it has 119 members as shown in the document denominated as “Pangalan ng mga Kasapi requested the suspension of the certification election proceedings.
ng Unyon;” hence, respondent misrepresented on the 20% requirement of the law as to its
membership. Nevertheless, the certification election pushed through on June 23, 2000. Respondent emerged
as the winner. Thus, petitioner filed a Protest with Motion to Defer Certification of Election
SC ruled that It does not appear in Article 234 (b) of the Labor Code that the attendees in the Results and Winner.
organizational meeting must comprise 20% of the employees in the bargaining unit. In fact, even
the IRR of the Labor Code does not so provide. It is only under Article 234 (c) that requires the Petitioner’s contention:
names of all its members comprising at least twenty percent (20%) of all the employees in the - The certification election held on June 23, 2000 was an exercise in futility because,
bargaining unit where it seeks to operate. Clearly, the 20% minimum requirement pertains to the once respondent’s registration is cancelled, it would no longer be entitled to be
employees’ membership in the union and not to the list of workers who participated in the certified as the exclusive bargaining agent of the supervisory employees.
organizational meeting. - Some of respondent’s members were not qualified to join the union because they
were either confidential employees or managerial employees.
Indeed, Article 234 (b) and (c) provide for separate requirements, which must be submitted for - Prayed that the certification of the election results and winner be deferred until the
the union's registration, and which respondent did submit. Here, the total number of employees petition for cancellation shall have been resolved, and that respondent’s members
in the bargaining unit was 396, and 20% of which was about 79. Respondent submitted a who held confidential or managerial positions be excluded from the supervisors’
document entitled “Pangalan ng Mga Kasapi ng Unyon” showing the names of 119 employees bargaining unit.
17
of the word "shall." Petitioner points out that the Regional Director has admitted in its decision
Respondent’s contention: that respondent failed to submit the required documents for a number of years; therefore,
- The petition was filed primarily to delay the conduct of the certification election, the cancellation of its registration should have followed as a matter of course.
respondent’s certification as the exclusive bargaining representative of the supervisory
employees, and the commencement of bargaining negotiations. SC ruled that the non-compliance should not be a ground for the cancellation. Articles 238 and
- Prayed for the dismissal of the petition for the following reasons: (a) petitioner is 239 of the Labor Code provide that failure to file financial reports and the list of its members are
estopped from questioning respondent’s status as a legitimate labor organization as it grounds for the cancellation of Union Organization. However, consideration must be taken of the
had already recognized respondent as such during the preelection conferences; (b) fundamental rights guaranteed by Article XIII, Section 3 of the Constitution, i.e., the rights of all
petitioner is not the party-in-interest, as the union members are the ones who would workers to self-organization, collective bargaining and negotiations, and peaceful concerted
be disadvantaged by the non-submission of financial reports; (c) it has already activities. Labor authorities should bear in mind that registration confers upon a union the status
complied with the reportorial requirements, having submitted its financial statements of legitimacy and the concomitant right and privileges granted by law to a legitimate labor
for 1996, 1997, 1998, and 1999, its updated list of officers, and its list of members for organization, particularly the right to participate in or ask for certification election in a bargaining
the years 1995, 1996, 1997, 1998, and 1999; (d) the petition is already moot and unit. Thus, the cancellation of a certificate of registration is the equivalent of snuffing out the life
academic, considering that the certification election had already been held, and the of a labor organization.
members had manifested their will to be represented by respondent.
- Citing National Union of Bank Employees v. Minister of Labor, et al. and Samahan ng Moreover, submission of the required documents is the duty of the officers of the union. It would
Manggagawa sa Pacific Plastic v. Hon. Laguesma, the Med-Arbiter held that the be unreasonable for this Office to order the cancellation of the union and penalize the entire
pendency of a petition for cancellation of registration is not a bar to the holding of a union membership on the basis of the negligence of its officers.As aptly ruled by respondent
certification election. Bureau of Labor Relations Director Noriel: "The rights of workers to self-organization finds
general and specific constitutional guarantees. x x x Such constitutional guarantees should not
DOLE: Dismissed the appeal and the Regional Director’s reasoning: be lightly taken much less nullified. A healthy respect for the freedom of association demands
● While finding that respondent had indeed failed to file financial reports and the list of that acts imputable to officers or members be not easily visited with capital punishments against
its members for several years, he, nonetheless, denied the petition, ratiocinating that the association itself."
freedom of association and the employees’ right to self-organization are more
substantive considerations. At any rate, we note that on 19 May 2000, appellee had submitted its financial statement for the
● The fact that respondent won the certification election and that it had already been years 1996-1999. With this submission, appellee has substantially complied with its duty to
certified as the exclusive bargaining agent of the supervisory employees. In view of submit its financial report for the said period. To rule differently would be to preclude the union,
the foregoing, Regional Director Maraan—while emphasizing that the non-compliance after having failed to meet its periodic obligations promptly, from taking appropriate measures to
with the law is not viewed with favor—considered the belated submission of the correct its omissions. For the record, we do not view with favor appellee’s late submission.
annual financial reports and the list of members as sufficient compliance thereof and Punctuality on the part of the union and its officers could have prevented this petition.
considered them as having been submitted on time.
Mariwasa Siam Ceramics, Inc. v. The Secretary of the Department of Labor and
Bureau of Labor Relations: Director Cacdac’s inhibited himself from the case being the Employment, G.R. No. 183317, December 21, 2009.
previous counsel of respondent. Thus, DOLE Secretary Sto. Tomas took cognizance of the
appeal by dismissing the same holding that the constitutionally guaranteed freedom of FACTS:
association and right of workers to self-organization outweighed respondent’s noncompliance On May 4, 2005, respondent Samahan Ng Mga Manggagawa Sa Mariwasa Siam Ceramics, Inc.
with the statutory requirements to maintain its status as a legitimate labor organization. (SMMSC-Independent) was issued a Certificate of Registration as a legitimate labor
organization by DOLE.
CA’s Ruling: Denied the petition.
Mariwasa Siam Ceramics, Inc. filed a Petition for Cancellation of Union Registration against
respondent, claiming that the latter violated Article 234 of the Labor Code for not complying with
ISSUE:
the 20% requirement, and that it committed massive fraud and misrepresentation in violation of
Whether or not failure to comply with the statutory requirement, filing financial reports and the list Article 239 of the same code. The petitioner insists that respondent failed to comply with the
of its members, sufficient ground for the cancellation of registration of the respondent as a labor 20% union membership requirement for its registration as a legitimate labor organization
union -NO. because of the disaffiliation from the total number of union members of 102 employees who
executed affidavits recanting their union membership.
HELD:
Petitioner also insists that respondent’s registration as a legitimate labor union should be Regional Director of DOLE: issued an Order granting the petition, revoking the registration of
cancelled. Petitioner posits that once it is determined that a ground enumerated in Article 239 of respondent, and delisting it from the roster of active labor unions.
the Labor Code is present, cancellation of registration should follow; it becomes the ministerial Bureau of Labor Relations: granted the appeal. Reversed decision of Regional Director,
SMMSC-Independent remains in the roster of legitimate labor organizations.
duty of the Regional Director to cancel the registration of the labor organization, hence, the use CA: denied the petition.
18
certification election which petitioner opposed on the ground that HHE misrepresented itself to
ISSUES: be an independent union, when in fact it was a local chapter of the National Union of Workers in
1. Whether or not SMMSC-Independent failed to comply with 20% requirement -NO.  Hotel and Restaurant and Allied Industries (NUWHRAIN). It was also alleged that such omitted
2. whether or not the withdrawal of 31 union members from NATU affected the petition disclosure was intentional because petitioner’s supervisors union was already affiliated with it.
for certification election insofar as the 30% requirement is concerned -NO. Petitioner also filed a petition to cancel the union’s registration certificate. The Med-Arbiter
nevertheless granted HHE’s petition for certification election. 
 
HELD:
Petitioner appealed to the Secretary of Labor but it was denied as well as its motion for
1. Even assuming the veracity of the affidavits of recantation, the legitimacy of
reconsideration, prompting petitioner to file a petition for certiorari with the Court of Appeals. The
respondent as a labor organization must be affirmed. While it is true that the
CA issued a writ of injunction against the holding of HHE’s certification election until the petition
withdrawal of support may be considered as a resignation from the union, the fact
for cancellation of its registration shall have been resolved with finality. 
remains that at the time of the union's application for registration, the affiants were
 
members of respondent and they comprised more than the required 20% membership
In 2003, another union, herein respondent Pinag-Isang Galing at Lakas ng mga Manggagawa sa
for purposes of registration as a labor union. Article 234 of the Labor Code merely
Heritage Manila (PIGLAS), was formed by certain rank and file employees of petitioner at a
requires a 20% minimum membership during the application for union registration. It
meeting. PIGLAS was issued its registration certificate by the DOLE-NCR in 2004. HHE later on
does not mandate that a union must maintain the 20% minimum membership
adopted a resolution for its dissolution and then filed a petition for cancellation of its union
requirement all throughout its existence.
registration. 
 
2. It appears undisputably that the 31 union members had withdrawn their support to the PIGLAS filed a petition for certification election which petitioner opposed alleging that the new
petition before the filing of said petition. It would be otherwise if the withdrawal was union’s officers and members were also those who comprised the old union. Petitioner likewise
made after the filing of the petition for it would then be presumed that the withdrawal alleged that PIGLAS was formed to circumvent the CA’s injunction earlier issued. The Med-
was not free and voluntary. The presumption would arise that the withdrawal was Arbiter nevertheless granted the petition for certification election. 
procured through duress, coercion or for valuable consideration. In other words, the  
distinction must be that withdrawals made before the filing of the petition are Petitioner filed a petition to cancel PIGLAS’ registration, claiming that the documents submitted
presumed voluntary unless there is convincing proof to the contrary, whereas with the union’s application for registration bore the following false information:
withdrawals made after the filing of the petition are deemed involuntary.  The List of Members showed that the PIGLAS union had 100 union members; 
  The Organizational Minutes said that 90 employees attended the meeting on
The reason for such distinction is that if the withdrawal or retraction is made before the December 10, 2003; 
filing of the petition, the names of employees supporting the petition are supposed to  The Attendance Sheet of the meeting of December 10, 2003 bore the signature of  
be held secret to the opposite party. Logically, any such withdrawal or retraction 127 members who ratified the union’s Constitution and By-Laws; and
shows voluntariness in the absence of proof to the contrary. Moreover, it becomes  The Signature Sheet bore 128 signatures of those who attended that meeting.  
apparent that such employees had not given consent to the filing of the petition, hence
the subscription requirement has not been met.
Petitioner alleged that the misrepresentation was evidenced by the discrepancy in the number of
union members appearing in the application and the list as well as the number of signatories to
When the withdrawal or retraction is made after the petition is filed, the employees the attendance and signature sheets. The minutes reported only 90 employees attended the
who are supporting the petition become known to the opposite party since their names meeting. Petitioner also alleged that 33 members of PIGLAS were members of HHE, which is in
are attached to the petition at the time of filing. Therefore, it would not be unexpected violation of the policy against dual unionism. 
that the opposite party would use foul means for the subject employees to withdraw  
their support. DOLE-NCR: denied petitioner’s petition to cancel PIGLAS’ registration because the
discrepancies in the number of members in the application’s supporting documents were not
The affidavits of recantation were executed after the identities of the union members material and did not constitute misrepresentation. The dual unionism is also not a ground for
became public, i.e., after the union filed a petition for certification election on May 23, canceling registration, since the members of HHE simply exercised their right to self-
2005, since the names of the members were attached to the petition. The purported organization and freedom of association when they joined PIGLAS. The Bureau of Labor
withdrawal of support for the registration of the union was made after the documents Relations affirmed the DOLE-NCR, by reasoning that PIGLAS’ organization meeting lasted for
were submitted to the DOLE, Region IV-A. The logical conclusion, therefore, following 12 hours. Thus, it was possible for the number of attendees to have increased as the meeting
jurisprudence, is that the employees were not totally free from the employer's progressed. Besides, the union only needed 50 members of the total of 250 employees in the
pressure, and so the voluntariness of the employees' execution of the affidavits bargaining unit to comply with the 20% membership requirement. Petitioner filed a petition for
becomes suspect. certiorari with the Court of Appeals but it was denied as well as petitioner’s motion for
reconsideration. Hence, this petition for review under Rule 45.
The Heritage Hotel Manila v. Pinag-isang Galing at Lakas ng mga Manggagawa sa  
Heritage Manila (PIGLASHERITAGE), G.R. No. 177024, October 30, 2009. ISSUES:
1. Whether or not the union made fatal misrepresentation in its application for union registration
–NO.
FACTS: 2. Whether or not dual unionism is a ground for canceling a union’s registration –NO.
The Heritage Hotel Employees Union (HHE) was formed in 2000 by certain rank and file  
employees of herein petitioner Heritage Hotel Manila, to which the Department of Labor and HELD: 
Employment-National Capital Region issued a certificate of registration.  HHE filed a petition for
19
1. Petitioner has no evidence of the alleged misrepresentation. The discrepancies alone cannot RD. Ventures went to the CA. The CA dismissed Ventures’ petition as well as the MR. Hence,
be taken as indication that PIGLAS misrepresented the information contained in these this petition for review
documents. Charges of fraud and misrepresentation should be clearly established by evidence
and surrounding circumstances because once it is proved, the labor union acquires none of the
rights accorded to registered organizations. 
ISSUE:
 
The discrepancies can be explained. While it appears that in the minutes of the December 10, Whether the registration of the Union must be cancelled.
2003 organizational meeting, only 90 employees responded to the roll call at the beginning, it
cannot be assumed that such number could not grow to 128 as reflected on the signature sheet
for attendance. The meeting lasted 12 hours from 11:00am to 11:00pm. There is no evidence RULING:
that the meeting hall was locked up to exclude late attendees. As to the fact that only 127 NO. The right to form, join, or assist a union is specifically protected by Art. XIII, Section 3 of the
members ratified the union’s constitution and by-laws when 128 signed the attendance sheet, it Constitution and such right, according to Art. III, Sec. 8 of the Constitution and Art. 246 of the
cannot be assumed that all those who attended approved of such. Any member had the right to Labor Code, shall not be abridged. Once registered with the DOLE, a union is considered a
hold out and refrain from ratifying those documents or to simply ignore the process.  The Labor
legitimate labor organization endowed with the right and privileges granted by law to such
Code and its implementing rules do not require that the number of members appearing on the
documents in question should completely dovetail.  For as long as the documents and organization. While a certificate of registration confers a union with legitimacy with the
signatures are shown to be genuine and regular and the constitution and by-laws democratically concomitant right to participate in or ask for certification election in a bargaining unit, the
ratified, the union is deemed to have complied with registration requirements.     registration may be canceled or the union may be decertified as the bargaining unit, in which
  case the union is divested of the status of a legitimate labor organization. Among the grounds for
The discrepancy in the list of members (showing only 100 members) and the signature and cancellation is the commission of any of the acts enumerated in Art. 239(a) of the Labor Code,
attendance sheets (showing 127 or 128 members) submitted is immaterial. A comparison of the such as fraud and misrepresentation in connection with the adoption or ratification of the union’s
documents shows that except for six members, the names found in the list are also in the
constitution and like documents. The Court, has in previous cases, said that to decertify a union,
attendance and signature sheets. PIGLAS more than complied with the 20% requirement since
only 50 employees out of 250 employees in the bargaining unit were required to unionize.  it is not enough to show that the union includes ineligible employees in its membership. It must
  also be shown that there was misrepresentation, false statement, or fraud in connection with the
Labor laws are liberally construed in favor of labor especially if doing so would affirm its application for registration and the supporting documents, such as the adoption or ratification of
constitutionally guaranteed right to self-organization. PIGLAS union’s supporting documents the constitution and by-laws or amendments thereto and the minutes of ratification of the
reveal the unmistakable yearning of petitioner company’s rank and file employees to organize.   constitution or by-laws, among other documents.
This yearning should not be frustrated by inconsequential technicalities. The evidence presented by Ventures consist mostly of separate hand-written statements of 82
2. NO. The right of any person to join an organization also includes the right to leave that
organization and join another one.HHE union ceased to exist, its certificate of registration being employees who alleged that they were unwilling or harassed signatories to the attendance sheet
already cancelled.  of the organizational meeting. However these evidence was presented seven months after the
union filed its petition for cancellation of registration. Hence these statements partake of the
S.S. Ventures International, Inc. v. S.S. Ventures Labor Union (SSVLU), G.R. No. 161690, nature of withdrawal of union membership executed after the Union’s filing of a petition for
July 23, 2008. certification election on March 21, 2000. We have said that the employees’ withdrawal from a
FACTS: labor union made before the filing of the petition for certification election is presumed voluntary,
Petitioner S.S. Ventures International, Inc. (Ventures), a PEZA- registered export firm with while withdrawal after the filing of such petition is considered to be involuntary and does not
principal place of business at Phase I-PEZA- Bataan Export Zone, Mariveles, Bataan, is in the affect the same. Now then, if a withdrawal from union membership done after a petition for
business of manufacturing sports shoes. Respondent S.S. Ventures Labor Union (Union) is a certification election has been filed does not vitiate such petition, it is but logical to assume that
labor organization registered with the DOLE. such withdrawal cannot work to nullify the registration of the union. The Court is inclined to
agree with the CA that the BLR did not abuse its discretion nor gravely err when it concluded
that the affidavits of retraction of the 82 members had no evidentiary weight.
March 21, 2000, the Union filed with DOLE-Region III a petition for certification election in behalf
of the rank-and-file employees. August 21, 2000, Ventures filed a Petition to cancel the Union’s
certificate of registration alleging that the Union deliberately and maliciously included the names The registration or the recognition of a labor union after it has submitted the corresponding
of more or less 82 former employees no longer connected with Ventures in its list of members papers is not ministerial on the part of the BLR. It becomes mandatory for the BLR to check if
who attended the organizational meeting and in the adoption/ratification of its constitution and the requirements under Art. 234 of the Labor Code have been sedulously complied with. If the
by-laws; that No organizational meeting and ratification actually took place; and the Union’s union’s application is infected by falsification and like serious irregularities, especially those
application for registration was not supported by at least 20% of the rank-and-file employees of appearing on the face of the application and its attachments, a union should be denied
Ventures. recognition as a legitimate labor organization. The issuance to the Union of Certificate of
Registration, in the case at bar, necessarily implies that its application for registration and the
supporting documents thereof are prima facie free from any vitiating irregularities.
Regional Director of DOLE- Region III favored Ventures and resolved to Cancel the Certificate of
the union. On appeal, the BLR Director granted the Union’s appeal and reversing the decision of

20
The relevance of the 82 individuals’ active participation in the Union’s organizational meeting APC filed a Motion for Reconsideration/Appeal regarding this Decision of the DOLE-NCR. In a
and the signing ceremonies thereafter comes in only for purposes of determining whether or not Resolution dated 18 July 2001, the Bureau of Labor Relations (BLR) denied the appeal,
the Union, even without the 82, would still meet what Art. 234(c) of the Labor Code requires to affirming the rationale of the DOLE-NCR.4
be submitted, requiring that the union applicant must file the names of all its members
comprising at least twenty percent (20%) of all the employees in the bargaining unit where it APC then immediately filed a Petition for Certiorari with the Court of Appeals, imputing grave
seeks to operate. abuse of discretion on the part of the BLR in denying its appeal. However, the petition was
dismissed outright by the Court of Appeals on the ground that APC had "failed to avail of the
remedy of a prior Motion for Reconsideration" before the filing of the certiorari petition, which
In its union records on file with this Bureau, respondent union submitted the names of 542 step, it stressed, is a "condition sine qua non to the filing of a petition for certiorari."5
members. This number easily complied with the 20% requirement, be it 1,928 or 2,202
employees in the establishment. Even subtracting the 82 employees from 542 leaves 460 union APC filed a Motion for Reconsideration dated 5 February 2002, but this too was denied by the
members, still within 440 or 20% of the maximum total of 2,202 rank-and-file employees of the Court of Appeals in a Resolution dated 13 September 2002. This time, the appellate court ruled
employer Venture. that the Motion for Reconsideration was "totally defective," for failing to contain the proof of
service or registry return receipts to the respondents. The Court of Appeals even noted that the
Affidavit of Service attached to the Motion for Reconsideration "failed to indicate the registry
Whatever misgivings the petitioner may have with regard to the 82 dismissed employees is
return receipts of the registered mails to the respondents."6
better addressed in the inclusion-exclusion proceedings during a pre-election conference. The
issue surrounding the involvement of the 82 employees is a matter of membership or voter
Hence, the present petition.
eligibility. It is not a ground to cancel union registration.

APC argues that its petition before the Court of Appeals involved mere questions of law, among
For fraud and misrepresentation to be grounds for cancellation of union registration under Article which is whether APFLAA’s union registration may be cancelled considering that the union is
239, the nature of the fraud and misrepresentation must be grave and compelling enough to allegedly composed of a mixture of supervisory and rank-and-file employees. It is posited that
vitiate the consent of a majority of union members. questions of law may be raised directly in a petition for certiorari without need of a prior motion
for reconsideration.

Air Philippines Corporation v. Bureau of Labor relations, G.R. No. 155395, June 22, 2006 However, it is clear from the petition filed by APC before the Court of Appeals that the issues
involved do not consist of questions of law only. It is insisted therein that employees holding the
position of Lead Cabin Attendants are supervisory employees and hence disallowed from joining
FACTS: a union of rank-and-file employees.8 On the other hand, APFLAA countered before the DOLE-
NCR and the BLR that only rank-and-file flight attendants comprised its membership. 9 Thus, the
very question of whether Lead Cabin Attendants are indeed supervisory employees appears to
The case initially centered on the union registration of respondent Air Philippines Flight be factual in nature, the proper resolution of which necessitates a factual determination of the
Attendants Association (APFLAA). APFLAA filed on 17 March 1999 a petition for certification actual duties of Lead Cabin Attendants. Indeed, APC made reference therein to such documents
election as the collective bargaining representative of the flight attendants of APC. After the as an employee’s manual in support of its argument, 10 documents that would evidently require
Med-Arbiter rendered a ruling ordering the holding of a certification election, such election was factual evaluation before accorded proper evidentiary value.
held on 5 August 1999, with majority of the votes cast in favor of APFLAA.
There is admittedly some leeway for the Court of Appeals if it was so minded to give due course
On 25 November 1999, APC filed a Petition for De-Certification and Cancellation of Union to APC’s petition, notwithstanding the failure to file a motion for reconsideration. Yet ultimately,
Registration against APFLAA with the DOLE. APC alleged that APFLAA could not be registered the determination of whether or not to admit a petition attended with such defect falls within the
as a labor organization, as its composition consisted of "a mixture of supervisory and rank-and- sound discretion of the Court of Appeals.
file flight attendants." Particularly, APC alleged that flight attendants holding the position of "Lead
Cabin Attendant," which according to it is supervisory in character, were among those who
Should the Court of Appeals decide, as it did, to dismiss the petition outright on such ground, it
comprised APFLAA.
would commit no reversible error of law nor any grave abuse of discretion, considering that the
rule requiring the filing of a motion for reconsideration before resorting to the special civil action
On 18 July 2001, the DOLE-National Capital Region (NCR) Regional Director Alex E. Maraan of certiorari is well entrenched in jurisprudence.
rendered a Decision dismissing the petition. The DOLE-NCR held that Article 245 of the Labor
Code, which states that supervisory employees are not eligible for membership in labor
It also does not escape the attention of the Court that the Motion for Reconsideration filed by
organizations of rank-and-file employees, does not provide a ground for cancellation of union
APC before the Court of Appeals was itself fatally defective, allowing the appellate court to deny
registration, which is instead governed by Article 239 of the Labor Code. 3

21
the same without having to evaluate its substantial arguments. The action of the appellate court
relative to APC’s missteps is consistent with procedural rules. c. Reportorial Requirements
ART. 252. [242-A] Reportorial Requirements.194 The following are documents required to be
Still, the Court has deigned to give a close look at the substantial arguments raised in APC’s submitted to the Bureau by the legitimate labor organization concerned:
petition before the Court of Appeals. a) Its constitution and by-laws, or amendments thereto, the minutes of ratification, and the list of
members who took part in the ratification of the constitution and by-laws within thirty (30) days
ISSUE: Whether or not the APFLAA could be registered from adoption or ratification of the constitution and by-laws or amendments thereto;
(b) Its list of officers, minutes of the election of officers, and list of voters within thirty (30) days
from election;
HELD: Yes.
(c) Its annual financial report within thirty (30) days after the close of every fiscal year; and
The DOLE-NCR Regional Director, in dismissing the petition for cancellation, cited our minute
(d) Its list of members at least once a year or whenever required by the Bureau.
resolution in SPI Technologies Incorporated v. DOLE wherein the Court observed that Article
Failure to comply with the above requirements shall not be a ground for cancellation of union
245 of the Labor Code, the legal basis for the petition for cancellation, merely prescribed the
registration but shall subject the erring officers or members to suspension, expulsion from
requirements for eligibility in joining a union and did not prescribe the grounds for cancellation of
membership, or any appropriate penalty.
union registration.13 Since the filing of this petition, the Court has had occasion to rule, in
Tagaytay Highlands International Golf Club v. Tagaytay Highlands Employees Union-PGTWO,14
6. Foreign Activities
that "[t]he inclusion in a union of disqualified employees is not among the grounds for
• Articles 284; 285; and 286, Labor Code.
cancellation, unless such inclusion is due to misrepresentation, false statement or fraud under
the circumstances enumerated in Sections (a) and (c) of Article 239 15 of the Labor Code."
ART. 284. [269] Prohibition Against Aliens; Exceptions.219 All aliens, natural or juridical, as well
as foreign organizations are strictly prohibited from engaging directly or indirectly in all forms of
Clearly then, for the purpose of de-certifying a union, it is not enough to establish that the rank-
trade union activities without prejudice to normal contacts between Philippine labor unions and
and-file union includes ineligible employees in its membership. Pursuant to Article 239 (a) and
recognized international labor centers: Provided, however, That aliens working in the country
(c) of the Labor Code, it must be shown that there was misrepresentation, false statement or
with valid permits issued by the Department of Labor and Employment, may exercise the right to
fraud in connection with the adoption or ratification of the constitution and by-laws or
self- organization and join or assist labor organizations of their own choosing for purposes of
amendments thereto, the minutes of ratification, or in connection with the election of officers,
collective bargaining: Provided, further, That said aliens are nationals of a country which grants
minutes of the election of officers, the list of voters, or failure to submit these documents
the same or similar rights to Filipino workers.
together with the list of the newly elected-appointed officers and their postal addresses to the
BLR.17
ART. 285. [270] Regulations of Foreign Assistance. (a) No foreign individual, organization or
entity may give any donations, grants or other forms of assistance, in cash or in kind, directly or
In its Petition for De-certification and Cancellation of Union Registration, APC did not impute on indirectly, to any labor organization, group of workers or any auxiliary thereof, such as
APFLAA such misrepresentation of the character necessitated under Article 239 (a) and (c) of cooperatives, credit unions and institutions engaged in research, education or communication, in
the Labor Code. APC merely argued that APFLAA was not qualified to become a legitimate relation to trade union activities, without prior permission by the Secretary of Labor.
labor organization by reason of its mixed composition of rank-and-file and supervisory Trade union activities" shall mean:
employees; and that APFLAA committed misrepresentation by making it appear that its (1) organization, formation and administration of labor organization;
composition was composed purely of rank-and-file employees. Such misrepresentation (if it can (2) negotiation and administration of collective bargaining agreements;
be called as such) as alleged by APC, is not conformable to Article 239 (a) and (c) of the Labor (3) all forms of concerted union action;
Code. Indeed, it appears from the record that APC instead devoted the bulk of its arguments in (4) organizing, managing, or assisting union conventions, meetings, rallies, referenda, teach-ins,
establishing that supervisory employees comprised part of the membership of APFLAA, a seminars, conferences and institutes;
ground which is not sufficient to cause the cancellation of union registration. And this is of (5) any form of participation or involvement in representation proceedings, representation
course all under the assumption that Lead Cabin Attendants are indeed supervisory employees, elections, consent elections, union elections; and
a claim consistently denied by APFLAA and which was not confirmed by either the DOLE-NCR (6) other activities or actions analogous to the foregoing.
or the BLR. (b) This prohibition shall equally apply to foreign donations, grants or other forms of assistance,
in cash or in kind, given directly or indirectly to any employer or
There may be remedies available to enforce the proscription set forth in Article 245 of the Labor (c) The Secretary of Labor shall promulgate rules and regulations to regulate and control the
Code on supervisory employees joining the union of rank-and-file employees. But consistent giving and receiving of such donations, grants, or other forms of assistance, including the
with jurisprudence, the rule under Article 245 barring supervisory employees from joining the mandatory reporting of the amounts of the donations or grants, the specific recipients thereof,
union of rank-and-file employees is not a ground for cancellation of union registration. the projects or activities proposed to be supported, and their duration.
Accordingly, we see no error on the part of the DOLE-NCR and the BLR in having dismissed
APC’s petition, and thus no cause to compel the Court of Appeals to disregard APC’s procedural ART. 286. [271] Applicability to Farm Tenants and Rural Workers. The provisions of this Title
errors and accept the petition for certiorari. pertaining to foreign organizations and activities shall be deemed applicable likewise to all
22
organizations of farm tenants, rural workers and the like: Provided, That in appropriate cases, Any action involving the funds of the organization shall prescribe after three (3) years from the
the Secretary of Agrarian Reform shall exercise the powers and responsibilities vested by this date of submission of the annual financial report to the Department of Labor and Employment or
Title in the Secretary of Labor. from the date the same should have been submitted as required by law, whichever comes
earlier: Provided, That this provision shall apply only to a legitimate labor organization which has
submitted the financial report requirements under this Code: Provided, further, That failure of
7. Union-Member Relations any labor organization to comply with the periodic financial reports required by law and such
• Articles 250 and 260(a), Labor Code. rules and regulations promulgated thereunder six (6) months after the effectivity of this Act shall
automatically result in the cancellation of union registration of such labor organization;
ART. 250. [241] Rights and Conditions of Membership in a Labor Organization.192 The (k) The officers of any labor organization shall not be paid any compensation other than the
following are the rights and conditions of membership in a labor organization: salaries and expenses due to their positions as specifically provided for in its constitution and
(a) No arbitrary or excessive initiation fees shall be required of the members of a legitimate labor by-laws, or in a written resolution duly authorized by a majority of all the members at a general
organization nor shall arbitrary, excessive or oppressive fine and forfeiture be imposed; membership meeting duly called for the purpose. The minutes of the meeting and the list of
(b) The members shall be entitled to full and detailed reports from their officers and participants and ballots cast shall be subject to inspection by the Secretary of Labor or his duly
representatives of all financial transactions as provided for in the constitution and by-laws of the authorized representatives. Any irregularities in the approval of the resolutions shall be a ground
organization; for impeachment or expulsion from the organization;
(c) The members shall directly elect their officers in the local union, as well as their national (l) The treasurer of any labor organization and every officer thereof who is responsible for the
officers in the national union or federation to which they or their local union is affiliated, by secret account of such organization or for the collection, management, disbursement, custody or
ballot at intervals of five (5) years. No qualification requirement for candidacy to any position control of the funds, moneys and other properties of the organization, shall render to the
shall be imposed other than membership in good standing in subject labor organization. The organization and to its members a true and correct account of all moneys received and paid by
secretary or any other responsible union officer shall furnish the Secretary of Labor and him since he assumed office or since the last day on which he rendered such account, and of all
Employment with a list of the newly-elected officers, together with the appointive officers or bonds, securities and other properties of the organization entrusted to his custody or under his
agents who are entrusted with the handling of funds within thirty (30) calendar days after the control. The rendering of such account shall be made:
election of officers or from the occurrence of any change in the list of officers of the labor (1) At least once a year within thirty (30) days after the close of its fiscal year;
organization; (2) At such other times as may be required by a resolution of the majority of the members of the
(d) The members shall determine by secret ballot, after due deliberation, any question of major organization; and
policy affecting the entire membership of the organization, unless the nature of the organization (3) Upon vacating his office.
or force majeure renders such secret ballot impractical, in which case, the board of directors of The account shall be duly audited and verified by affidavit and a copy thereof shall be furnished
the organization may make the decision in behalf of the general membership; the Secretary of Labor.
(e) No labor organization shall knowingly admit as members or continue in membership any (m) The books of accounts and other records of the financial activities of any labor organization
individual who belongs to a subversive organization or who is engaged directly or indirectly in shall be open to inspection by any officer or member thereof during office hours;
any subversive activity; (n) No special assessment or other extraordinary fees may be levied upon the members of a
(f) No person who has been convicted of a crime involving moral turpitude shall be eligible for labor organization unless authorized by a written resolution of a majority of all the members in a
election as a union officer or for appointment to any position in the union; general membership meeting duly called for the purpose. The secretary of the organization shall
(g) No officer, agent or member of a labor organization shall collect any fees, dues, or other record the minutes of the meeting including the list of all members present, the votes cast, the
contributions in its behalf or make any disbursement of its money or funds unless he is duly purpose of the special assessment or fees and the recipient of such assessment or fees. The
authorized pursuant to its constitution and by-laws; record shall be attested to by the president.
192 Article 241, pars. (c) and (j) are shown here as amended by Sec. 16 of R.A. No. 6715 (o) Other than for mandatory activities under the Code, no special assessments,
(1989). from any amount due to an employee without an individual written authorization duly signed by
(h) Every payment of fees, dues or other contributions by a member shall be evidenced by a the employee. The authorization should specifically state the amount, purpose and beneficiary of
receipt signed by the officer or agent making the collection and entered into the record of the the deduction; and
organization to be kept and maintained for the purpose; (p) It shall be the duty of any labor organization and its officers to inform its members on the
(i) The funds of the organization shall not be applied for any purpose or object other than those provisions of its constitution and by-laws, collective bargaining agreement, the prevailing labor
expressly provided by its constitution and by-laws or those expressly authorized by written relations system and all their rights and obligations under existing labor laws.
resolution adopted by the majority of the members at a general meeting duly called for the For this purpose, registered labor organizations may assess reasonable dues to finance labor
purpose; relations seminars and other labor education activities.
(j) Every income or revenue of the organization shall be evidenced by a record showing its Any violation of the above rights and conditions of membership shall be a ground for
source, and every expenditure of its funds shall be evidenced by a receipt from the person to cancellation of union registration or expulsion of officers from office, whichever is appropriate. At
whom the payment is made, which shall state the date, place and purpose of such payment. least thirty percent (30%) of the members of a union or any member or members specially
Such record or receipt shall form part of the financial records of the organization. concerned may report such violation to the Bureau. The Bureau shall have the power to hear
and decide any reported violation to mete the appropriate penalty.
23
Criminal and civil liabilities arising from violations of above rights and conditions of membership ● As a result, Union declared a deadlock and filed a notice of strike with the National
shall continue to be under the jurisdiction of ordinary courts. Conciliation and Mediation Board -National Capital Region (NCMB-NCR).
● Conciliation and mediation proved to be futile, such that in January 2002, majority of
ART. 260. [249] Unfair Labor Practices of Labor Organizations.205 It shall be unfair labor Union’s members voted to stage a strike.
practice for a labor organization, its officers, agents or representatives: ● However, the DOLE Secretary (SOLE) timely assumed jurisdiction over the dispute, and
(a) To restrain or coerce employees in the exercise of their right to self- organization. However, the parties were summoned and heard on their respective claims, and were required to
a labor organization shall have the right to prescribe its own rules with respect to the acquisition submit their respective position papers.
or retention of membership; ● May 31, 2002: the SOLE issued an Order:
- Given all the foregoing, we cannot follow the Union's formula and in effect disregard
University of Santo Tomas v. Samahang Manggagawa ng UST (SM-UST), G.R. No. 169940, the School's two other bargaining units; to do so is a distortion of economic reality that
September 14, 2009. will not bring about long term industrial peace. We cannot simply adopt the School's
proposal in light of the parties' bargaining history, particularly the pattern of increases
Petitioner: University of Santo Tomas in the last cycle.
Respondent: Samahang Manggagawa ng UST (SM-UST)
- Considering all these, we believe the following to be a fair and reasonable resolution
FACTS: of the wage issue.
● Respondent Union was the authorized bargaining agent of the non-academic/non-teaching 1st Year - P1,000.00/month
rank-and-file daily- and monthly-paid employees (about 619) of Petitioner UST. 2nd Year - P2,000.00/month
● October 2001: During formal negotiations for a new collective bargaining agreement (CBA) 3rd Year - P2,200.00/month
for the academic year 2001 through 2006, UST submitted its "2001-2006 CBA Proposals.” - These increases, at a three-year total of P68,337,600, are less than the three (3)- year
● December 2001: UST submitted its final offer on the economic proposals. The proposals of increases in the last CBA cycle to accommodate the School's proven lack of capacity
Union and UST are compared below: to afford a higher increase, but are still substantial enough to accommodate the
workers' needs while taking into account the symmetry that must be maintained with
UNION UST the wages of the other bargaining units.
- On a straight line aggregate of P5,200.00, the non-academic personnel will receive
A. ACADEMIC YEAR 2001-2002 P498.48 less than an Instructor I (member of the faculty union) who received an
aggregate of P5,698.48, thus maintaining the gap between the teaching and non-
teaching personnel. The salary difference will as well be maintained over the three (3)-
1. Salary increase per month P800.00 P1,000.00
year period of the CBA.
2. Signing bonus P10,000.00 P10,000.00 - An RFI employee (member of the union's bargaining unit) will receive a monthly salary
of P21,695.95 while an Instructor I (faculty union member) will have a salary of
3. Additional Christmas bonus P2,000.00 P2,000.00 P22,948.00; while an RF5-5/A (member of the union's bargaining unit) will receive a
salary of P23,462.97 compared to an Asst. Prof. 1 (faculty) who will receive
B. ACADEMIC YEAR 2002-2003 P29,250.96.
- From a total cost of salary increases for the first year at P7,428,000, these costs will
escalate to P22,284,000 in the second year, and to P38,625,000 at the third year.
1. Salary increase per month P1,500.00 P1,700.00
Given these figures, the amounts available for distribution and the member of groups
2. Additional Christmas bonus P2,000.00 P2,000.00 sharing these amounts, these increases are by no means minimal.
- A review of the past bargaining history of the parties shows that the School as a
3. Salary restructuring P6 million P6.19 million matter of course grants a signing bonus. This ranged from P8,000.00 during the first
three (3) years of the last CBA to P10,000.00 during the remaining two (2) years of the
C. ACADEMIC YEAR 2003-2004 re-negotiated term. In this instance, the School's offer of P10,000.00 signing bonus is
already reasonable considering that the School could have taken the position that no
signing bonus is due on compulsory arbitration in line with the ruling in Meralco v.
1. Salary increase per month P1,700.00 P2,000.00
Quisumbing et al., G.R. No. 127598, 27 January 1999.
2. Additional Christmas bonus P2,000.00 P2,000.00 ● Union filed a motion for reconsideration but it was denied by the SOLE.
● Thus, Union filed an original petition for certiorari with the Court of Appeals, claiming that
the awards made by the SOLE are not supported by the evidence on record and are
● On the other hand, Union reduced its demands for the first year from P8,000.00 monthly contrary to law and jurisprudence.
salary increase per employee to P7,000.00, and from P75,000.00 signing bonus to ● The CA ruled:
P60,000.00 for each employee, but UST insisted on its final offer.

24
- Based on UST Chief Accountant Antonio J. Dayag's Certification, the tuition fee other benefits pursuant to the SOLE Order; and that to date, 572 out of the 619 members
increment for the SY 2001-2002 amounted to P101,036,330.37. From this amount, the have been paid. UST argues that by their acceptance of the award and the resulting
tuition fee adjustment amounting to P2,785,143.00 was deducted leaving a net tuition payments made to them, the said Union members have ratified its offer and thus rendered
fee increment of P98,251,189.36. moot the case before the CA.
- Pursuant to Section 5 (2) RA 6728, 70% of P98,251,187.36 or P68,775,831.15 is the - Union echoes the CA ruling that they are not precluded from receiving additional
amount UST has to allocate for salaries, wages, allowances and other benefits of its benefits that the courts may award later on, bearing in mind that – “the employer and
2,290 employees, categorized as follows: the employee do not stand on the same footing. Considering the country's prevailing
619 non-teaching personnel represented by UNION; economic conditions, the employee oftentimes finds himself in no position to resist
1,452 faculty members represented by UST-Faculty Union (UST-FU); and money proffered, thus, his case becomes one of adherence and not of choice.
219 academic/administrative officials. The last group of employees is - Union’s members were merely constrained to accept payment at the time. Christmas
excluded from the coverage of the two bargaining units. was then just around the corner, and the Union members were in no position to resist
- Based on the SOLE’s arbitral award for the first year (AY 2001-2002), UNION would the temptation to accept much-needed cash for use during the most auspicious
get P15,475,000.00 or 22.5% of P68,775,831.00 (or from the 70% of the tuition fee occasion of the year. Time and again, we have held that necessitous men are not,
increment for AY 2001-2002) by approximating UST's expense on the increment of truly speaking, free men; but to answer a present emergency, will submit to any terms
salaries/wages, allowances and benefits of the non-teaching personnel. xxx that the crafty may impose upon them.
- The allocation for salary increases, 13th month pay, signing bonus and Christmas - Besides, as individual components of a Union possessed of a distinct and separate
bonus for UST's teaching and non-teaching employees, as well as the school officials, corporate personality, Union’s members should realize that in joining the organization,
amount to P68.475 million. This represents almost 70% of the UST incremental tuition they have surrendered a portion of their individual freedom for the benefit of all the
fee proceeds for AY 2001-2002. Considering the fringe benefits being extended to other members; they submit to the will of the majority of the members in order that
UST employees, it is safe to assume that the fringe benefits being extended to UST they may derive the advantages to be gained from the concerted action of all. Since
employees, it is safe to assume that the funds for such benefits need to be sourced the will of the members is personified by its board of directors or trustees, the
from UST's other revenues. xxx We examined UST audited financial statements from decisions it makes should accordingly bind them. Precisely, a labor Union exists in
1997 to 2001 and found that UST's "other incomes" come from parking fees, rent whole or in part for the purpose of collective bargaining or of dealing with employers
income and interest income. It, likewise, derives income from school operations. xxx concerning terms and conditions of employment. What the individual employee may
- Even if the other benefits of the faculty members were to be charged from the not do alone, as for example obtain more favorable terms and conditions of work, the
remaining balance of the Accumulated Excess of Revenues Over Expenses, there labor organization, through persuasive and coercive power gained as a group, can
would still be sufficient amount to fund the other benefits of the non-teaching accomplish better.
personnel. xxx 2. The SC need not rule on this issue . UST argues that the CA erred in ordering it to source
- However, while We subscribe to UST's position on "salary distortion", Our earlier part of its judgment award from the school's other income, claiming that RA 6728 does not
findings support UST's contention that the UST has substantial accumulated income compel or require schools to allocate more than 70% of the incremental tuition fee increase
and thus, We deem it proper to award an increase, not in salary, to prevent any salary for the salaries and benefits of its employees. Citing an authority in education law
distortion, but in signing bonus. The arbitral award of P10,000 signing bonus per (Sarmiento III), it stresses that: "...if schools use any part of the 20% reserved for the
employee awarded by public Union is hereby increased to P18,000.00. upgrading of school facilities to supplement the salaries of their academic and non-
● UST filed an MR, which was denied. Hence, this petition. academic personnel, they would not only be violating the students' constitutional right to
quality education through "improvement and modernization" but also committing a serious
ISSUES: infraction of the mandatory provisions of RA 6728."
- Union argues that RA 6728 does not provide that the increase or improvement of the
1. WON the CA gravely abused its discretion when it ruled that the members of Union salaries and fringe benefits of the employees should be exclusively funded from the
did not voluntarily and knowingly accept the arbitral award of the SOLE. - NO income of UST which is derived from the increase in tuition fees. In fact, the statute
2. WON the CA has completely ignored the clear mandate and intention of RA 6728 has no application with respect to the manner of disposition of the other incomes (as
otherwise known as the Government Assistance to Students and Teachers in Private distinguished from income derived from tuition fee increases) of UST, nor does it
Education Act. - The SC need not rule on this issue. preclude or exempt the latter from using its other income or part thereof to fund the
3. WON the CA gravely abused its discretion when it increased the signing bonus from cost of increases or improvements in the salaries and benefits of its employees.
P10,000.00 to P18,000.00. - YES - The SC finds it unnecessary to rule on this matter. These fringe benefits are included
in the DOLE Secretary's award - an award which UST seeks to affirm in toto; this
HELD: being so, it cannot now argue otherwise. Since it abides by the DOLE Secretary's
award, which it finds "fair and equitable," it must raise the said amount through
sources other than incremental tuition fee proceeds.
1. NO, the CA did not. UST alleges that, as of December 11, 2002, 526 regular non-academic
3. YES, the CA did. UST contends that the CA’s award of additional signing bonus is contrary
employees - out of a total of 619 members - have decided to unconditionally abide by the
to the nature and principle behind the grant of such benefit, which is one given as a matter
May 31, 2002 Order of SOLE. UST claims that it began paying the wage adjustment and
25
of discretion and cannot be demanded by right, a consideration paid for the goodwill that - Denominated as a general faculty assembly, the convocation was supposed to
existed in the negotiations, which culminate in the signing of a CBA. UST claims that since discuss the "state of the unratified USTUSTFU CBA" and "status and election of
this condition is absent in the parties' case, it was erroneous to have rewarded UNION with USTFU officers"
an increased signing bonus. ● The convocation was held as scheduled (Oct. 4, 1996)
- Union argues that since no strike or any untoward incident occurred, goodwill between - The general assembly was attended by members of the USTFU and, as admitted by
the parties remained, which entitles UNION's members to receive their signing bonus. the appellants, also by "nonUSTFU members who are members in good standing of
Besides, UNION asserts that since UST did not appeal the DOLE Secretary's award, it the UST Academic Community Collective Bargaining Unit"
may not now asserts that since UST did not appeal the DOLE Secretary's award, it - On this occasion, appellants were elected as USTFU's new set of officers by
may not now argue against its grant, the issue remaining being the propriety of the acclamation and clapping of hands
awarded amount; that is, whether or not it was proper for the CA to have raised it from - Upon a motion of one Atty. Lopez, admittedly not a member of USTFU, that the
P10,000.00 to P18,000.00. USTFU Constitution and By-Laws (CBL) and "the rules of the election be suspended
- A signing bonus is a grant motivated by the goodwill generated when a CBA is and that the election be held [on] that day"
successfully negotiated and signed between the employer and the Union. Here, no ● Respondents filed the instant petition seeking injunctive reliefs and the nullification of the
CBA was successfully negotiated by the parties. It is only because UST prays for this results
Court to affirm in toto the SOLE’s Order that we shall allow an award of signing bonus. ● Subsequently, petitioners and UST allegedly entered into another CBA covering the period
There would have been no other basis to grant it if UST had not so prayed. We shall from 01 June 1996 to 31 May 2001
take it as a manifestation of UST's liberality, which we cannot now allow it to withdraw. ● The med-arbiter issued a TRO against petitioners
A bonus is a gratuity or act of liberality of the giver; when UST filed the instant petition ● Public respondent, director of the BLR agreed with the med-arbiter that the Oct. 4 election
seeking the affirmance of the SOLE’s Order in its entirety, assailing only the increased was void
amount of the signing bonus awarded, it is considered to have unqualifiedly agreed to
grant the original award to UNION's members. ISSUES:
1. WON the October 4 election was valid. – NO
2. WoN the suspension of the CBL was valid. – NO
WHEREFORE, the petition is PARTIALLY GRANTED. The signing bonus of EIGHTEEN
THOUSAND PESOS (P18,000.00) per member of respondent Samahang Manggagawa ng
HELD:
U.S.T. as awarded by the Court of Appeals is REDUCED to TEN THOUSAND PESOS
1. The election was not valid
(P10,000.00). All other findings and dispositions made by the Court of Appeals in its January 31, ● The right to vote in the union is enjoyed only by union members
2005 Decision and September 23, 2005 Resolution in CA-G.R. SP No. 72965 are AFFIRMED. - A union election should be distinguished from a certification election, which is the
process of determining, through secret ballot, the sole and exclusive bargaining agent
UST Faculty Union (USTFU) v. Bitonio, G.R. No. 131235, November 16, 1999. of the employees in the appropriate bargaining unit, for purposes of collective
bargaining
Petitioners: UST Faculty Union (Ustfu), Gil Y. Gamilla, Corazon Qui, Norma Calaguas, Irma - Specifically, the purpose of a certification election is to ascertain whether or not a
Potenciano, Luz De Guzman, Remedios Garcia, Rene Arnejo, Editha Ocampo, Cesar Reyes, majority of the employees wish to be represented by a labor organization and, in the
Celso Nierra, Gliceria Baldres, Ma. Lourdes Medina, Hidelita Gabo, Mafel Ysrael, Laura Abara, affirmative case, by which particular labor organization
Natividad Santos, Ferdinand Limos, Carmelita Espina, Zenaida Famorca, Philip Aguinaldo, - In a certification election, all employees belonging to the appropriate bargaining unit
Benedicta Alava and Leoncio Casal can vote
Respondents: Dir. Benedicto Ernesto R. Bitonio Jr. of The Bureau of Labor Relations, Med- - A union member who likewise belongs to the appropriate bargaining unit is entitled to
Arbiter Tomas F. Falconitin of the National Capital Region, Department of Labor And vote in said election. However, the reverse is not always true; an employee belonging
Employment (DOLE), Eduardo J. Mariño Jr., Ma. Melvyn Alamis, Norma Collantes, Urbano to the appropriate bargaining unit but who is not a member of the union cannot vote in
Alabagia, Ronaldo Asuncion, Zenaida Burgos, Anthony Cura, Fulvio M. Guerrero, Myrna Hilario, the union election, unless otherwise authorized by the constitution and bylaws of the
Teresita Meer, Fernando Pedrosa, Nilda Redoblado, Rene Sison, Evelyn Tirol and Rosie union.
Alcantara - In this case, non-members participated in the assembly and in the voting
● The election did not follow the procedures imposed by the CBL
FACTS: - The union's CBL is the fundamental law that governs the relationship between
● Private Respondents are duly elected officers of the UST Faculty Union (USTFU) and among the members of the union. It is where the rights, duties and obligations,
● The secretary general of USTFU posted a notice addressed to all USTFU members powers, functions and authority of the officers as well as the members are defined
announcing a general assembly to be held on 05 October 1996 - First, the assembly was not called by the USTFU. It was merely a convocation of
- Among others, the general assembly was called to elect USTFU's next set of officers. faculty clubs, It was not convened in accordance with the provision on general
● On 02 October 1996, the secretary general of UST, upon the request of the various UST membership meetings. It was in fact a gathering that was called and participated in by
faculty club presidents, issued notices allowing all faculty members to hold a convocation management and nonunion members
on 04 October 1996
26
- Second, there was no commission on elections to oversee the election, as mandated ● Judge Tabigne, as the trial judge, ordered a hearing thereon on December 22, 1962, as a
by Sections 1 and 2 of Article IX of the USTFU's CBL condition precedent to execution of the judgment. Such Report was submitted for resolution
- Third, the purported election was not done by secret balloting, in violation of Section 6, and approval at the hearing of December 22, 1962, but the records before us fail to show
Article IX of the USTFU's CBL, as well as Article 241 (c) of the Labor Code that the trial judge ever acted on or approved the Report.
2. The suspension of the CBL was not valid ● Before and after the submittal of the Chief Examiner's Report of December 14, 1962, the
● Petitioners contend that the October 4, 1996 assembly "suspended" the union's CBL. They union pressed for execution of the final judgment in favor of its claimants-members.
aver that the suspension and the election that followed were in accordance with their - It filed, furthermore, on December 20, 1962, an Urgent Motion for Preliminary
"constituent and residual powers as members of the collective bargaining unit to choose Attachment, in view of the disposition by King Hong of its trucks and automotive
their representatives for purposes of collective bargaining." equipment and by virtue of the fact admitted by King Hong that it had stopped
● The general faculty assembly was not the proper forum to conduct the election of USTFU operations preparatory to liquidation, by reason of the alien nationality of most of its
officers. Not all who attended the assembly were members of the union; some, apparently, stockholders, under the provisions of Republic Act No. 3018 nationalizing the rice and
were even disqualified from becoming union members, since they represented corn industry.
management - In another motion of December 4, 1962, the union had asked that the CIR at least
● The person who moved for the suspension of USTFU's CBL was not a member of USTFU. order King Hong to put up a bond of P500,000.00 to answer for the payment of the
Allowing a nonunion member to initiate the suspension of a union's CBL, and nonunion judgment or to deposit said amount in Court.
members to participate in a union election on the premise that the union's CBL had been ● The 49 union members-petitioners assert that these motions were left hanging until the
suspended in the meantime, is incompatible with the freedom of association and protection union filed a mandamus petition with this Court (not the present case).
of the right to organize ● The trial judge issued and released on April 15, 1963 the Deposit Order dated March 30,
● The grievances of the petitioners could have been brought up and resolved in accordance 1963, ordering King Hong, within 10 days from its finality, to deposit in Court P100,000.00
with the procedure laid down by the union's CBL and to file a surety bond of equal amount, "to guarantee the payment of whatever amount
(a) due petitioner (union) and its members concerned after this Court shall have finally
WHEREFORE, the Petition is hereby DISMISSED and the assailed Resolutions AFFIRMED. decided the obligation of herein King Hongs under the judgment." This Order was affirmed
Costs against petitioners. by the CIR en banc, in its Resolution denying King Hong's motion for reconsideration.
● King Hong sought a review of the Deposit Order. The Supreme Court, in its Resolution of
Heirs of Teodolo M. Cruz v. Court of Industrial Relations, G.R. No. L-23331, December 27, September 20, 1963, dismissed for lack of merit the petition for review, and the dismissal
1969. became final on October 24, 1963.
● Earlier, on June 25, 1963: King Hong had offered the Union the maximum amount of
Petitioners: Heirs of Teodolo M. Cruz, Mary Concepcion and Edgardo Cruz (counsel of the P110,000.00 in full settlement of its obligations to the members-claimants of the Union
Union, not the 49 Union members – petitioners in a separate case consolidated with this one) under the judgment, but the union rejected the offer and counter-offered the minimum
King Hongs: Court of Industrial Relations (CIR), Santiago Rice Mill and King Hong and
Company amount of P200,000.00.
● The union meanwhile filed to no avail a series of urgent motions on May 8, July 1, August
FACTS: 29 and September 6, 1963 for approval of the Chief Examiner's Partial Report of December
● June 21, 1952: The Santiago Labor Union, composed of workers of the Santiago Rice Mill, 14, 1962 and for enforcement, through a writ of execution or contempt proceedings, of the
a business enterprise engaged in the buying and milling of palay at Santiago, Isabela, and Deposit Order.
owned and operated by King Hong Co., Inc. (King Hong), filed before the CIR a petition for ● Upon the finality of the Supreme Court’s Resolution dismissing King Hong’s petition for a
overtime pay, premium pay for night, Sunday and holiday work, and for reinstatement of review of the Deposit Order, the union again filed on October 29, 1963 still another Urgent
workers illegally laid off. Motion, advising the trial judge of the SC's action rejecting King Hong’s appeal and invoking
● The total sum claimed by the workers, P100,816.36 for overtime pay, P19,350.00 for the CIR’s ministerial duty of enforcing its said Order. The trial judge took no action on this
premium pay and P3,360.00 for differential pay under the Minimum Wage Law - amounted latest Urgent Motion of the union.
to P123,526.36. ● Instead, an unscheduled conference was called and held on October 31, 1963 in the
● September 19, 1958: The CIR, thru Judge Emiliano G. Tabigne, dismissed the petition of chambers of the trial judge, and attended by representatives of King Hong, including their
the union for lack of merit and want of jurisdiction. counsels of record, on one hand and Segundino S. Maylem, president of the union and
● Upon a motion for reconsideration, the CIR en banc, by a split decision of 3-2 vote, eight directors of the union, on the other. Four of these nine union representatives,
reversed the decision of the trial judge. including the union president himself, had no claims or awards whatever under the
● The Supreme Court affirmed the CIR en banc decision and remanded the records for judgment. Said union officials were not assisted by counsel, as petitioner Mary Concepcion,
enforcement by the CIR. counsel of record of the union, was not present, not having been notified of the conference.
● The Chief Examiner filed with the CIR his Partial Report of December 14, 1962, wherein At this conference, King Hong made again the same offer to settle and quitclaim the
the judgment award in favor of the workers was determined and computed as totaling judgment in favor of the union members for P110,000.00. But this time, as appears from
PHP423,756.74. Both the union members and King Hong opposed the report. the transcript of the conference, King Hong and the directors of the union decided to settle
the case amicably with the payment by King Hong of the same amount of P110,000.00
27
which was deposited with the Court's disbursing officer immediately upon the signing of the ● Petitioners Magalpo, Bulos and Batoon, likewise moved CIR en banc to reconsider and set
settlement which will be prepared by King Hong through its counsel. aside the trial judge's approval of the "Settlement", in disregard of their objection and
● Petitioner Natividad Magalpo, a director of the union, together with petitioners Lydia Bulos pending motions of November 5, 1963 to shelve the proposed settlement and to enforce
and Paciencia Batoon, both union members-claimants, filed on November 5, 1963, their the Deposit Order.
verified "Manifestation and Objection with Ex- Parte Urgent Motion", relating what ● December 26, 1963: They were joined in their plea for reconsideration by 47 other union
transpired at the conference, charging the union president, Maylem, with bad faith in that members-claimants, Co-petitioners at bar.
he never previously advised the union representatives that the conference of October 31, ● King Hong filed its opposition to the motions for reconsideration, questioning the personality
1963 was to discuss a compromise settlement nor that the Supreme Court’s resolution and interest of petitioners-movants Magalpo and her 2 other co-movants and asserting that
dismissing King Hong’s appeal from the Deposit Order had already become final, and they were bound by the "Settlement" entered into by their union's board of directors. It
asking the trial judge to shelve the proposed settlement until King Hong shall have alleged that it had deposited with the CIR the sum of P110,000.00 stipulated in the
complied with the said deposit order. “Settlement" on the same day of its approval by the trial judge. It filed with the CIR on
● These petitioners further filed on November 4, 1963 an urgent Ex-parte Motion for the November 21, 1963 a letter of ratification dated November 10, 1963 addressed to the trial
issuance of a writ of execution for the enforcement of the Deposit Order against King Hong, judge and purportedly signed by some 79 union members-claimants confirming and
and asked the trial judge to act on their two urgent motions upon receipt thereof. accepting the settlement executed by the union board.
● Both urgent motions were totally ignored by both the trial judge as well as by King Hong, ● Petitioners list 21 of these signatures as questionable, asserting that they are at variance
despite due notice on the latter. with other corresponding signatures in the Payroll dated November 8, 1963 submitted to
● The request of the union president, Maylem, at the October 31, 1963 conference that the the CIR on November 21, 1963, such that "either one or the other signature is a forgery."
trial judge have the union counsel present during the proposed signing of the settlement ● King Hong counters that there is "absolutely no truth to the claim" and that the signers of
agreement set for November 8, 1963, as expressly noted by the trial judge, was likewise the ratification letter “have all received their individual shares of the P110,000.00 settlement
ignored. paid by [King Hong] and this in itself is a ratification on their part of said settlement."
● Notwithstanding that notice of the conference set for November 8, 1963 at 2:30 p.m. was ● Nothing appears in the record, however, as to whether and in what manner King Hong
served on November 5, 1963 on the union counsel, petitioner Mary Concepcion, the Court determined the authenticity of the signatures.
scheduled conference was never held. ● King Hong further filed on December 18, 1963 a motion for reconsideration of the trial
● Unexplainedly, Maylem, the union president and nine other members of the union's board judge's Order approving payment of P33,000.00 to the petitioners-attorneys by way of
of directors (out of 13 board members) had earlier executed a "Settlement" on November 8, attorneys' fees.
without the knowledge, advice, and conformity of the union counsel, with King Hong's ● On August 1, 1964, and August 4, 1964, they were served with copies of the CIR’s en banc
attorney-in-fact, who was duly assisted by King Hong’s two counsels, who likewise Resolution dated March 9, 1964, penned by the trial judge, “finding no sufficient
executed the "Settlement." In this "Settlement", the said union officials claiming to act “with justifications to set aside, disturb or modify the Order issued in these cases on November 8
the authorization of the Board of Directors and its members," in consideration of the sum of and 9, 1963" and denying all three motions for reconsideration. Judges Amando C.
P110,000.00, or one-fourth of the estimated P423,756.74-judgment liability of King Hong, Bugayong and Ansberto F. Paredes concurred under date of July 29, 1964 with the
“waived and quitclaimed…any and all claims it (the union) may have against King Hong as Resolution, while Judge Arsenio Martinez took no part. No statement of the material
well as the claim of each and every one of the members of the…union against the…firm.” allegations of, and issues raised in, the pertinent pleadings set out in detail hereinabove nor
The union further "warranted" in said “Settlement" "that aside from the petitioner (union) reasons for the conclusion of insufficient justification reached by the majority resolution are
itself and the members thereof, there are no other persons who have any interest over the given therein.
judgment debt and that if it should happen that other persons shall make a claim against ● Then Presiding Judge Jose S. Bautista dissented. "Taking into account the precipitate
[King Hong] and/or said judgment debt, that [King Hong], nevertheless, shall no longer be approval of settlement over the objection of some union members concerned and without
liable therefor." hearing them, on the strength simply of the manifestation of the petitioner's Board of
● The “Settlement" was immediately submitted to the trial judge who forthwith on the same Directors that it had authority to compromise when previously said union members
day, November 8, 1963, issued his Order, approving the same, and entered into CIR’s concerned had already manifested in Annex "E" that there was no such authority," he voted
records at 1:45 p.m. of the same day. "that the case be restored to the status quo as of October 30, 1963, but the payment
● Petitioners-lawyers Mary Concepcion et al. upon learning of the "Settlement" and King already made to the union members be considered as partial payments on account, subject
Hong’s deposit with the CIR of the sum of P110,000.00 in pursuance thereof filed in the to final liquidation and adjustment; that an order of execution of the judgment in cases Nos.
afternoon of November 8, 1963 a motion for withdrawal of the sum of P33,000.00 G.R. L-21758 and L-21759 of the Supreme Court be issued (upholding the Order of March
equivalent to their 30% contingent fee, without prejudice to such action as they may take 30, 1963 for deposit of P200,000.00 in cash and surety bond) be issued and that the
for enforcing their lien to its full extent. The trial judge granted such motion in its Order of Hearing Officer shall resume the hearing of the Examiner's Report."
November 9, 1963. ● Hence, the appeals of petitioners.
● In due course, petitioners-lawyers moved for reconsideration and setting aside of the trial ● The Santiago Labor Union filed its Answer “putting its weight behind the prayers of the
judge's Order of November 8, 1963 approving the "Settlement" and prayed the CIR en banc petitioners." The Answer reveals that the union members, feeling betrayed, had
to reinstate the judgment against King Hong and to enforce the Deposit Order disauthorized and removed from office Maylem, the union president and his board of

28
directors who had executed the "Settlement" with King Hong and disclaimed the documents P110,000.00 would conceivably just about cover the 30% attorneys' fees payable to the
of ratification that they had signed at the behest of Maylem. petitioners-counsels under the contract, if they were so minded to enforce it and bad faith
on the union's part were shown, with the union members left holding an empty bag. Such
ISSUE: WON the Supreme Court can sanction the CIR’s majority resolution upholding the trial onerous terms of the settlement could not then properly be approved by the trial judge as
judge's approval of the union board's settlement for P110,000.00 of the estimated P423,756.74 - "not contrary to law, morals and public policy."
judgment liability of King Hong in favor of the individual union member. ● All these underscore the failure of due process when petitioners were deprived of the
formal conference on the proposed settlement scheduled for November 8, 1963 and of their
HELD: NO. right to be assisted by the union counsel as expressly requested, so that a fair hearing
● The precipitate approval of the purported settlement under the circumstances goes against could be accorded petitioners and an opportunity afforded them to air their serious charges
the grain of fundamental considerations of justice, equity and due process. of bad faith and lack of authority against the union leadership.
● Petitioners were not accorded due process of law, when, for reasons unexplained in the ● Another vital reason for striking down the settlement is the lack of any express or specific
record, the conference set for November 8, 1963 at 2:30 p.m. to take up formally the authority of the president and majority of the union board of directors to execute the same
proposed settlement was cancelled and never held. and scale down the estimated P423,756.74 - judgment liability of King Hong in favor of the
● The lack of due deliberation and caution in the trial judge's instant approval of the individuals union members to P110,000.00. On the contrary, petitioner board member
settlement is seen from the stipulations therein that the union thereby waived and Magalpo timely challenged the authority of the union board to execute any such settlement,
quitclaimed any and all claims which it may have against King Hong, as well as the claim of expressly informing the trial judge that the union had specifically appointed an entity in
each and every one of the members of the union against King Hong, when precisely the Manila, the "CREAM, Inc.", formerly Credit Research and Intelligence, as its attorney-in-fact
authority of the union board members to enter into any such compromise or settlement was and "exclusive authorized representative for the evaluation, adjustment and liquidation of its
under express challenge by petitioner Magalpo, a board member herself in her Objection claim against King Hong." Forty-seven other union members-claimants joined petitioner
and Urgent Motion to shelve the settlement filed on November 5, 1963, which the trial judge Magalpo in their denunciation of the union board's unauthorized action, and in their plea for
completely disregarded. reconsideration with the CIR. Forty-nine union members-claimants entitled to the bulk of
- Petitioner Magalpo further made serious charges that Maylem, the union president, the judgment award have filed this appeal from the adverse rulings of the Court below.
had misled the board members into attending the unscheduled conference held on These union members have repudiated the former union president, Maylem and his board
October 31, 1963 before the trial judge, and had deliberately concealed from them the of directors, for having betrayed the union members, and the new union leadership, in its
fact of entry on October 24, 1963 of the Order of the Supreme Court upholding the Answer filed with the Court, has joined petitioners in their prayer for redress, categorically
Deposit Order and the effect thereof of making mandatory upon the trial judge, in asserting that the union records do not show any grant by the members to the former union
accordance with the terms of his own order, the issuance of a writ for execution or board under Maylem to "negotiate the claim or subsequent ratification of the settlement for
enforcement to compel King Hong to so deposit P100,000.00 in cash and an equal P110,000.00" which is “unthinkable and ridiculous." Under such circumstances, the letter of
amount in surety bond to guarantee satisfaction of the union members' judgment ratification of the settlement purportedly signed by some 79 members, many of whose
against King Hong. signatures thereon are denounced as forgeries and which ratification was not authenticated
- In point of fact, the union's own Urgent Motion of October 29, 1963, emphasizing that in the proceedings below and has been expressly disowned by petitioners herein, cannot
the CIR no longer had any excuse for not complying with the deposit order, as well as be given any legal significance or effect.
petitioner Magalpo et al's Urgent ex-parte motion of November 4, 1963 to the same ● The judgment award was for the payment of overtime, premium and differential pay to the
effect were pending before the trial judge, unresolved and unacted upon. individual union members as claimants and premium and differential pay to the individual
- Petitioners Magalpo et al. had reason, therefore, to assail the proposed settlement for union members as claimants and for the reinstatement of the individual union members
P110,000.00 as unconscionable, when at the very least the union members could be who testified and proved their having been illegally laid-off, which represent a personal
assured of P200,000.00 under the deposit order to satisfy their judgment credit, while material interest directly in favor of the individual union members, as against the lack of
the report of King Hong court's examiner showed that King Hong had sufficient assets material interest on the part of the union as such, the union's lack of authority to execute
and considering that their partial judgment credit, as estimated by King Hong court's the settlement, in the absence of express or specific authorization by the union members,
examiner, amounted to more than P400,000.00. becomes patent. The authority of the union as such, to execute a settlement of the
● The trial judge's rush approval of the settlement disregarded the grave adverse judgment award in favor of the individual union members, cannot be presumed but must be
consequences thereof to the union members. The settlement, as prepared by King Hong’s expressly granted.
counsel, provided for a union warranty that aside from the union itself and the members ● In La Campana Food Products, Inc. v. Employees Ass'n. vs. Court of Industrial Relations,
thereof, "there are no other persons who have any interest over the judgment debt and that et al., the Supreme Court ruled that the union's loss of interest in the case was no ground
if it should happen that other persons shall make a claim against King Hong and/or said for dismissing the case, since “the labor union as a body in reality has not so great a
judgment debt, that King Hong, nevertheless, shall no longer be liable therefor." Such material interest in the controversy as would prejudice it in the event of dismissal. It is the
warranty was against the very facts of record, which showed that as early as June 21, twenty-one (21) members for whose benefit the ULP case was prosecuted who stand to
1963, petitioners-counsels had duly recorded their attorneys' lien of "30% of whatever take tremendous losses" and suffer injustice.
amount may finally be awarded in favor of the petitioner." Thus, technically, since the award ● Just as this Court has stricken down unjust exploitation of laborers by oppressive
in favor of the union members amounted to more than P400,000.00 the settlement for employers, so will it strike down their unfair treatment by their own unworthy leaders. The
29
Constitution enjoins the State to afford protection to labor. Fair dealing is equally demanded WHEREFORE, the respondent Court's Orders of November 8, 1963 and March 9, 1964 are
of unions as well as of employers in their dealings with employees. The union has been hereby declared null and void and set aside. The respondent court is directed to proceed
evolved as an organization of collective strength for the protection of labor against the immediately with the execution of the judgment rendered by it against respondent firm in Cases
unjust exactions of capital, but equally important is the requirement of fair dealing between Nos. 709-V and V-1 as affirmed by this Court's decision of August 31, 1962, in accordance with
the union and its members, which is fiduciary in nature, and arises out of two factors: "one the directives set forth in the next preceding paragraph, which is incorporated by reference as an
is the degree of dependence of the individual employee on the union organization; the integral portion of the dispositive part of this decision. With costs against private respondent in
other, a corollary of the first, is the comprehensive power vested in the union with respect both cases herein decided.
to the individual." The union may be considered but the agent of its members for the
purpose of securing for them fair and just wages and good working conditions and is a. Admission and Discipline of Members
subject to the obligation of giving the members as its principals all information relevant to • Articles 250(a) and (e); 260(a), and 292(c), Labor Code.
union and labor matters entrusted to it.
● King Hong never filed with the CIR any denial or responsive pleading traversing the factual ART. 250. [241] Rights and Conditions of Membership in a Labor Organization.192 The
allegations in petitioner Magalpo's Manifestation and Objection. Such failure on the part of following are the rights and conditions of membership in a labor organization:
King Hong constitutes an implied admission of the material averments. King Hong’s (a) No arbitrary or excessive initiation fees shall be required of the members of a legitimate labor
justification now that it did not file any responsive pleading or denial because Magalpo and organization nor shall arbitrary, excessive or oppressive fine and forfeiture be imposed;
her co-petitioners had no personality to file their pleadings as they were not parties to the
cases in the CIR is of no avail, for they were actually the awardees and beneficiaries under (e) No labor organization shall knowingly admit as members or continue in membership any
the judgment against King Hong and the union was but their agent. individual who belongs to a subversive organization or who is engaged directly or indirectly in
● The cases of Jesalva et al. vs. Bautista, and Diomela et al. vs. CIR cited by King Hong any subversive activity;
clearly have no application. In Jesalva, there was no question that the union had acted with
the authority of the union membership. No deceit or concealment or misrepresentation ART. 260. [249] Unfair Labor Practices of Labor Organizations.205 It shall be unfair labor
tainted the settlement. Neither was the amount of the settlement denounced as practice for a labor organization, its officers, agents or representatives:
unconscionable. In Diomela, there was no question of the authority of the union president (a) To restrain or coerce employees in the exercise of their right to self- organization. However,
to withdraw the unfair labor practice charge, as the three employees directly affected had a labor organization shall have the right to prescribe its own rules with respect to the acquisition
co-signed the withdrawal motion with him. or retention of membership;
● We therefore sustain the minority opinion of then Presiding Judge Bautista of the CIR.
● It is directed that an order for the enforcement of the Deposit Order, be forthwith issued, ART. 292. [277] Miscellaneous Provisions.
and that hearings on the Chief Examiner's Report of December 14, 1962 be resumed (c) Any employee, whether employed for a definite period or not, shall, beginning on his first day
immediately and without interruption so that the amounts due under the judgment to the of service, be considered as an employee for purposes of membership in any labor union.
individual union members may be finally determined without further delay.
● It is unfortunate that pending these proceedings, no application for preliminary injunction Villar v. Inciong, G.R. Nos. 50283-84, April 20, 1983.
restraining King Hong from disposing of its assets was made, since King Hong had stopped
operations in 1962 preparatory to liquidation, by virtue of the provisions of Republic Act No. Petitioners: Dolores Villar, Romeo Pequito, Dionisio Ramos, Benigno Mamaraldo, Orlando
3018 nationalizing the rice and corn industry. King Hong's stockholders are, however, Acosta, Recitacion Bernus, Anselma Andan, Rolando De Guzman and Rita Llagas
charged with notice of King Hong's liability by virtue of the pendency of these appeals, and Respondents: The Hon. Amado G. Inciong, as Deputy Minister of The Ministry of Labor, Amigo
should any liquidating dividends have been distributed and paid to them in the meantime, Manufacturing Incorporated and Philippine Association of Free Labor Unions (PAFLU)
they shall stand liable for the satisfaction of the union workers' judgment against King Hong
to the extent of such dividends respectively paid to and received by them. Similarly, any FACTS:
outstanding unpaid subscriptions or balances of subscriptions to King Hong's capital stock, ● Petitioners were members of the Amigo Employees Union-PAFLU, a duly registered labor
estimated at P20,000.00 shall be subject to garnishment and execution in satisfaction of organization which, was the existing bargaining agent of the employees in private
the judgment. respondent Amigo Manufacturing, Inc. (Company).
● As to the contingent 30% attorneys' fees of petitioners-lawyers, the Court deems it proper ● The Company and the Amigo Employees Union-PAFLU had a CBA governing their labor
at this stage, to direct in the exercise of its authority to control the amount of such fees, that relations, which agreement was then about to expire on February 28, 1977. Within the last
petitioners-lawyers may collect their stipulated contingent 30% attorneys' fees to the extent 60 days of the CBA, upon written authority of at least 30% of the employees in the
that additional amounts may be realized on the union workers' judgment up to the sum of company, including the petitioners, the Federation of Unions of Rizal (FUR) filed a petition
P150,000.00, including the initial payment of P110,000.00, (on which they have already for certification election with MOLE. The petition was opposed by the PAFLU with whom
collected their corresponding fee), such that any further amounts collected beyond the Amigo Employees Union was at that time affiliated. The same employees who had
P150,000.00 shall no longer be subject to said contingent fee. signed the petition filed by FUR signed a joint resolution disaffiliating from PAFLU.
● Dolores Villar, representing herself to be the authorized representative of the Amigo
Employees Union, filed a petition for certification election in the Company. The Amigo
30
Employees Union-PAFLU intervened and moved for the dismissal of the petition for corrective and remedial measures, in keeping with its laws and regulations, for its
certification election filed by Villar, on the ground, among others that Villar had no legal preservation and continued existence; lest by its folly and inaction, the labor union crumble
personality to sign the petition since she was not an officer of the union nor is there factual and fall.
or legal basis for her claim that she was the authorized representative of the local union.
● Med-Arbiter dismissed the petition filed by Villar, which dismissal is still pending appeal WHEREFORE, IN VIEW OF ALL THE FOREGOING, the Order appealed from affirming the joint
before BLR. Amigo Employees Union-PAFLU called a special meeting of its general decision of the OIC of Regional Office No. 4 in RO4-Case No. T-IV-3549-T and RO4 Case No.
membership. A Resolution was thereby unanimously approved which called for the RD-4-4088-77-T granting clearance to terminate petitioners as well as dismissing their complaint
investigation by the PAFLU national president, of all of the petitioners and one Felipe with application for preliminary injunction, is hereby AFFIRMED. No costs.
Manlapao, for continuously maligning the union spreading false propaganda that the union
officers were merely appointees of the management; and for causing divisiveness in the Salunga v. Court of Industrial Relations, G.R. No. L-22456, September 27, 1967.
union. PAFLU formed a Trial Committee to investigate the local union's charges against the Facts:
petitioners for acts of disloyalty.
● PAFLU and the Company concluded a new CBA which also reincorporated the same Francisco Salunga (Salunga) had been an employee of San Miguel Brewery, Inc since 1948.
provisions of the existing CBA, including the union security clause. PAFLU President And Salunga was also a member of a Union in San Miguel since 1953.
rendered a decision finding the petitioners guilty of the charges. PAFLU demanded the
Company to terminate the employment of the petitioners pursuant to the security clause of San Miguel Agrees that as a condition of employment of those employees who are members of
the CBA. Acting on PAFLU's demand, the Company informed PAFLU that it will first secure the UNION on the date of the signing of this agreement, or may join the UNION during the
the necessary clearances to terminate petitioners. PAFLU requested the Company to put effectivity of this agreement that if such employees retracts its membership from the union,
petitioners under preventive suspension pending the application for said clearances to he/she shall be automatically removed as an employee in San Miguel.
terminate the petitioners. The Company filed the request for clearance to terminate the
petitioners before DOLE which was granted. DOLE Secretary Inciong denied the appeal, On August 18, 1961 Salunga tendered his resignation from the Union, which accepted it on
hence, this petition for review. August 26, 1961, and transmitted it to the Company on August 29, 1961, with a request for the
immediate implementation of section 3, Article II.
ISSUE: WON the DOLE Secretary erred in affirming the grant of clearance of termination of
petitioners. San Miguel informed Salunga that his aforementioned resignation would result in the
termination of his employment, in view of Section 3. AS such Salunga wrote to the Union a letter
HELD: NO. withdrawing or revoking his resignation and advising the Union to continue deducting his
● It is true that disaffiliation from a labor union is not open to legal objection. It is implicit in the monthly union dues. Salunga also furnished a copy of this communication to the Company. San
freedom of association ordained by the Constitution. But the Court has laid down the ruling Miguel in turn, notified the Union of the receipt of said copy and that San
that a closed shop is a valid form of union security, and such provision in a CBA is not a
restriction of the right of freedom of association guaranteed by the Constitution. Miguel shall not take any action on this case and shall consider Salunga still a member of the
● In the case at bench, the Company and the Amigo Employees Union-PAFLU entered into a union and continue deducting his union dues,
CBA with a union security clause which is a reiteration of the old CBA. The quoted
stipulation for closed-shop is clear and unequivocal. Petitioners’ theory that their expulsion On September 8, 1961, the Union told the San Miguel that Salunga’s membership could not be
was not valid upon the grounds is untenable. PAFLU had the authority to investigate reinstated and insisted on his separation from the service.San Miguel told The Union that they
petitioners on the charges filed by their co-employees in the local union and after finding asked Salunga if he realized that by resigning from the Union he would in effect be forfeiting his
them guilty as charged, to expel them from the roll of membership of the Amigo Employees position in the company. When he answered in the negative, San Miguel showed him a copy of
Union-PAFLU is clear under the constitution of the PAFLU to which the local union was our Collective Bargaining Agreement and called his attention to Sec. 3, Art. II. Salunga told San
affiliated. And pursuant to the security clause of the new CBA, reiterating the same clause Miguel that he did not realize that he would be losing his job if he were to resign from the Union.
in the old CBA, PAFLU was justified in applying said security clause.
● Recognized and salutary is the principle that when a labor union affiliates with a mother San Miguel clarified that it did not at any time ask or urge him to withdraw his resignation;
union, it becomes bound by the laws and regulations of the parent organization. It is neither that it is asking or insisting that the Union readmit him into the Union. San Miguel
undisputable that oppositors were members of the Amigo Employees Union at the time that asserted that informing him of the consequences of his resignation from the Union, was the only
said union affiliated with PAFLU; hence, oppositors are bound by the laws and regulations humane thing to do under the circumstances. Nevertheless, if notwithstanding the foregoing
of PAFLU. clarification the Union still consider him as having actually resigned from the organization, and
● Inherent in every labor union, or any organization for that matter, is the right of self- The Union insist that San Miguel dismiss him from the service in accordance with Sec. 3, Article
preservation. When members of a labor union seek the disintegration and destruction of the II of the CBA, Then San Miguel will have no alternative but to do so.
very union to which they belong; they thereby forfeit their rights to remain as members of
the union which they seek to destroy. Prudence and equity, as well as the dictates of law Salunga appealed to his complaint to PAFLU National Convention but his dismissal was
and justice, therefore, compelling mandate the adoption by the labor union of such maintained.
31
a complaint for unfair labor practice, against the Union, its president, respondent John de considerations, springing from the belief that petitioner had resigned from the Union
Castillo, respondent Cipriano Cid, as PAFLU president, the Company, and its aforementioned without realizing its effect upon his employment. And, as the Union reiterated its
Vice-President Miguel Noel with the CIR. demand, the Company notified petitioner that it had no other alternative but to
terminate his employment, and dismissed him from the service, although with "regret".
CIR: ruled for Salunga and held that San Miguel, Miguel Noel, National Brewery & Allied
Industrial Labor Union of the Philippines (PAFLU), John de Castillo, and Cipriano Cid, are
hereby declared guilty of unfair labor practices as charged, and ordered to cease and desist Bugay v. Kapisanan ng mga Manggagawa sa Manila Railroad Company, G.R. No. L-13093,
from further committing such unfair labor practice acts complained. February 28, 1962.

On appeal, CIR en banc was reversed. FACTS:


Bugay, auditor of the Kapisanan Ng Mga Manggagawa Sa Manila Railroad Company
Issue/s: (Kapisanan) and a payroll clerk of Manila Railroad, was requested by the secretary-treasurer of
the company to deliver certain documents belonging to the union and in compliance therewith he
Whether or not Salunga cannot be re-admitted delivered them without consulting the officers of the union. Making use of these documents, the
management of the company filed with the City Fiscal of Manila against Vicente K. Olazo,
Whether or not San Miguel was guilty of unfair labor practice. president of the union a charge for falsification of commercial document (such was dismissed by
the city fiscal).
Ruling: Subsequently, charges for disloyalty and conduct unbecoming a union member were preferred
against Bugay, which caused his explusion. Bugay filed a charge for ULP against the union
1. He may be re-admitted as a member, although he voiced out concerns and before the CIR which, after due hearing, rendered decision holding that Bugay’s expulsion was
grievances against The Union, because it was stated in the constitution and by-laws of illegal it appearing that the same has not been approved by the majority of the chapters of the
the Union. union as required by its constitution and by-laws. Bugay did not have sufficient opportunity to
defend himself.
Although as a general rule, A state may not compel ordinary voluntary associations to
admit thereto any given individual, because membership therein may be accorded or Issue: Whether or not Bugay expulsion as a union member is proper.
withheld as a matter of privilege; the rule is qualified in respect of labor unions holding
a monopoly in the supply of labor, either in a given locality, or as regards a particular RULING:
employer with which it has a closed-shop agreement. No. Bugay’s affiliation with the Kapisanan was never terminated. That being the case, Bugay is
entitled to all the rights and obligations appertaining to every member of the Kapisanan Union
Consequently, it is well settled that such unions are not entitled to arbitrarily exclude actions, whether favorable or otherwise, must be taken by the chapters within a period of ten
qualified applicants for membership, and a closed-shop provision would not justify the days from the time they receive the resolution. Even under the assumption that the proceedings
employer in discharging, or a union in insisting upon the discharge of, an employee against Bugay were not irregular, the resolution in question never had any valid effect on his
whom the union thus refuses to admit to membership, without any reasonable ground union membership. Considering that he has been unduly and discriminatorily deprived of such
therefor. Needless to say, if said unions may be compelled to admit new members, rights and obligations, the Court finds, and so holds, that the Kapisanan, by their act and
who have the requisite qualifications, with more reason may the law and the courts conduct, have engaged in and are engaging in ULP.
exercise the coercive power when the employee involved is a long standing union It should be observed that the main basis of Bugay's action is his claim that because of the ULP
member, who, owing to provocations of union officers, was impelled to tender his committed by the officers of defendant union as found by the CIR and the SC, he has suffered
resignation, which he forthwith withdrew or revoked. Surely, he may, at least, invoke moral damages. It is true that the decisions both of the CIR and SC do not contain any
the rights of those who seek admission for the first time, and can not arbitrarily he statement that the charges preferred by the officers of the union against him which resulted in
denied readmission. his expulsion were "trumped up" or fabricated, or that said officers acted maliciously or in bad
faith, but the fact remains that the two courts have found that his expulsion was illegal because
of the irregularities committed in his investigation. In effect, it was found that not only has he not
2. S.C ruled that there was no Unfair Lanor practice The Company was reluctant — if not
been given an opportunity to defend himself but his expulsion was not submitted to the different
unwilling — to discharge the petitioner. When the Union first informed the Company of
chapters of the union as required by its constitution and by-laws.
petitioner's resignation and urged implementation of section 3 of the bargaining
Issue: WON Bugay was afforded due process.
contract, the Company advised petitioner of the provision thereof, thereby intimating
SC: No. In the investigation held and in the board meeting where the committee’s report
that he had to withdraw his resignation in order to keep his employment. Besides, the
recommending Bugay’s expulsion was approved, Bugay was not present (he was in Lucena).
Company notified the Union that it (the Company) would not take any action on the
The committee assigned to summon him failed to serve notice upon him because he was in
case and would consider the petitioner, "still a member" of the Union. When the latter,
Lucena. However, proceedings still continued anent the absence of Bugay.
thereafter, insisted on petitioner's discharge, the Company still demurred and
explained it was not taking sides and that its stand was prompted merely by "humane"
32
Besides, the contention that majority of the chapters voted in favor of Bugay's expulsion is not before the holding of the election in the Convention. On June 18, 2001, Atty. Verceles sent a
borne by the evidence. An examination of the chapters to the Kapisanan board of directors follow‐up letter to the President of FFW requesting for immediate action on his protest.
shows that all of the votes, except those of the Hondagua Chapters and Engineering Manila
Yard Chapter were not validly cast. On July 13, 2001, Atty. Verceles, as President of UEEA‐FFW and officer of the Governing Board
of FFW, filed before the BLR a petition13 for the nullification of the election of Atty. Montaño as
b. Election of Officers FFW National Vice‐ President.
• Article 250(c); (f); and (k), Labor Code.
Atty. Montaño filed his Comment with Motion to Dismiss on the grounds that the Regional
ART. 250. [241] Rights and Conditions of Membership in a Labor Organization.192 The Director of the Department of Labor and Employment (DOLE) and not the BLR has jurisdiction
following are the rights and conditions of membership in a labor organization: over the case.
(c) The members shall directly elect their officers in the local union, as well as their national
officers in the national union or federation to which they or their local union is affiliated, by secret DECISION OF LOWER FFW COMELEC: Montano not qualified.
ballot at intervals of five (5) years. No qualification requirement for candidacy to any position
shall be imposed other than membership in good standing in subject labor organization. The Bureau of Labor Relations (BLR):
secretary or any other responsible union officer shall furnish the Secretary of Labor and Montano qualified. it upheld its jurisdiction over the intra‐union dispute case.
Employment with a list of the newly-elected officers, together with the appointive officers or
agents who are entrusted with the handling of funds within thirty (30) calendar days after the BLR likewise affirmed that Atty. Verceles' legal personality to institute the action as president of
election of officers or from the occurrence of any change in the list of officers of the labor an affiliate union of FFW.
organization;
BLR ruled that there were no grounds to hold Atty. Montaño unqualified to run for National Vice‐
(f) No person who has been convicted of a crime involving moral turpitude shall be eligible for President of FFW.
election as a union officer or for appointment to any position in the union;
BLR denied motion for reconsiderarion.
(k) The officers of any labor organization shall not be paid any compensation other than the
salaries and expenses due to their positions as specifically provided for in its constitution and CA: Reversed BLR, montano not qualified.
by-laws, or in a written resolution duly authorized by a majority of all the members at a general Atty. Montaño did not possess the qualification requirement under paragraph (d) of Section 26
membership meeting duly called for the purpose. The minutes of the meeting and the list of that candidates must be an officer or member of a legitimate labor organization. According to the
participants and ballots cast shall be subject to inspection by the Secretary of Labor or his duly CA, since Atty. Montaño, as legal assistant employed by FFW, is considered as confidential
authorized representatives. Any irregularities in the approval of the resolutions shall be a ground employee, consequently, he is ineligible to join FFW Staff Association, the rank‐and‐file union of
for impeachment or expulsion from the organization; FFW. * CA (motion for reconsideration): denied.

Montaño v. Verceles, G.R. No. 168583, July 26, 2010. ISSUE/S:


1.Whether or not the CA was correct in upholding the jurisdiction of the BLR;
FACTS:
2.Whether or not was correct in not declaring as premature the petition in view of the pending
Atty. Montaño worked as legal assistant of FFW Legal Center on October 1, 1994. protest before FFW COMELEC
Subsequently, he joined the union of rank‐and‐file employees, the FFW Staff Association, and
eventually became the employees' union president in July 1997. In November 1998, he was 3. Whether or not the CA was correct in not finding that the petition violated the rule on non ‐
likewise designated officer‐in‐charge of FFW Legal Center. forum shopping

During the 21st National Convention and Election of National Officers of FFW, Atty. Montaño 4. Whether or not the CA was correct in not dismissing the case for being moot in view of the
was nominated and elected for the position of National Vice‐President despite the finding of appointment of Atty. Verceles as NLRC Commissioner;
FFW COMELEC that Atty. Montaño is not qualified to run for the position because Section 76 of
Article XIX of the FFW Constitution and By‐Laws prohibits federation employees from sitting in 5. Whether or not Montano is qualified to run as National Vice ‐ President
its Governing Board and strong opposition and protest of respondent Atty. Ernesto C. Verceles
(Atty. Verceles), a delegate to the convention and president of University of the East Employees' 6. Whether or not the CA was correct in granting the petition to annul Montano's election as
Association (UEEA‐FFW) which is an affiliate union of FFW. FFW National Vice‐ President on the ground that FFW Staff Association is not a legitimate labor
organization.
On May 28, 2001, through a letter to the Chairman of FFW COMELEC, Atty. Verceles reiterated
his protest over Atty. Montaño's candidacy which he manifested during the plenary session Ruling:
33
By‐laws in order to ensure credible future elections in the interest and welfare of affiliate unions
1.YES. The BLR has jurisdiction over intra‐union disputes involving a federation. of FFW.

Section 226 of the Labor Code28 clearly provides that the BLR and the Regional Directors of 5. NO, the decision of FFW COMELEC is final and should have been given credence.
DOLE have concurrent jurisdiction over inter‐union and intra‐union disputes. Such disputes
include the conduct or nullification of election of union and workers' association officers. There FFW COMELEC, undeniably, has sufficient authority to adopt its own interpretation of the
is, thus, no doubt as to the BLR's jurisdiction over the instant dispute involving member‐unions explicit provisions of the federation's constitution and by‐laws and unless it is shown to have
of a federation arising from disagreement over the provisions of the federation's constitution and committed grave abuse of discretion, its decision and ruling will not be interfered with. The FFW
by‐laws. Constitution and By‐laws are clear that no member of the Governing Board shall at the same
time perform functions of the rank‐and‐file staff. The BLR erred in disregarding this clear
Rule XVI lays down the decentralized intra‐union dispute settlement mechanism. Section 1 provision. The FFW COMELEC's ruling which considered Atty. Montaño's candidacy in violation
states that any complaint in this regard ‘shall be filed in the Regional Office where the union is of the FFW Constitution is therefore correct.
domiciled.' The concept of domicile in labor relations regulation is equivalent to the place where
the union seeks to operate or has established a geographical presence for purposes of 6. NO. the CA's declaration of the illegitimate status of FFW Staff Association is proscribed by
collective bargaining or for dealing with employers concerning terms and conditions of law, owing to the preclusion of collateral attack.
employment.

The matter of venue becomes problematic when the intra‐union dispute involves a federation, Tancinco v. Ferrer-Calleja, G.R. No. 78131, January 20, 1988.
because the geographical presence of a federation may encompass more than one FACTS:
administrative region. Pursuant to its authority under Article 226, this Bureau exercises original Private respondents are the prime organizers of ITM-MEA. While said respondents were
jurisdiction over intra‐union disputes involving federations. It is well‐settled that FFW, having preparing to file a petition for direct certification of the Union as the sole and exclusive
local unions all over the country, operates in more than one administrative region. Therefore, bargaining agent of ITM's bargaining unit, the union's Vice- President, was promoted to the
this Bureau maintains original and exclusive jurisdiction over disputes arising from any violation position of Department Head, thereby disqualifying him for union membership. Said incident, led
of or disagreement over any provision of its constitution and by‐laws. to a strike spearheaded by Lacanilao group, respondents. Another group however, led by
petitioners staged a strike inside the company premises. After 4 days the strike was settled. On
2.YES. The petition to annul Atty. Montaño's election as VP was not prematurely filed. May 10, 1986 an agreement was entered into by the representatives of the management,
Lacanilao group and the Tancinco group the relevant terms of which states that all monthly
It is true that under the Implementing Rules, redress must first be sought within the organization employees shall be united under one union, the ITM Month Employees Association (ITM-MEA)
itself in accordance with its constitution and by‐laws. However, this requirement is not absolute to be affiliated with ANGLO. The management of ITM recognizes ANGLO as the sole and
but yields to exception under varying circumstances. the FFW COMELEC failed to timely act exclusive bargaining agent of all the monthly-paid employees;
thereon. Thus, Atty. Verceles had no other recourse but to take the next available remedy to However, during the pre-election conference attended by MOLE officers, ANGLO through its
protect the interest of the union he represents as well as the whole federation, especially so that National Secretary, made a unilateral ruling excluding some 56 employees consisting of the
Atty. Montaño, immediately after being proclaimed, already assumed and started to perform the Manila office employees, members of Iglesia ni Kristo, non-time card employees, drivers of Mrs.
duties of the position. Consequently, Atty. Verceles properly sought redress from the BLR so Salazar and the cooperative employees of Mrs. Salazar. The election of officers was conducted,
that the right to due process will not be violated. the 56 employees in question participated but their votes were segregated without being
counted. Lacanilao's group won. Lacanilao garnered 119 votes with a margin of 3 votes over
3.Montano is estopped from raising this issue since he only raised this during this motion for Tancinco prompting petitioners to make a protest.
reconsideration with the CA. The allegation regarding certification against forum shopping was BLR ruled holding the exclusion of the 56 employees as arbitrary, whimsical, and wanting in
belatedly raised. It is settled that new issues cannot be raised for the first time on appeal or on legal basis but set aside the challenged order on the ground that 51 ** of 56 challenged voters
motion for reconsideration. were not yet union members at the time of the election per April 24, 1986 list submitted before
the Bureau.
4. The CA is correct. There is necessity to resolve the case despite the issues having become
moot. As manifested by Atty. Verceles, Atty. Montaño ran and won as FFW National President ISSUE:
after his challenged term as FFW National Vice‐President had expired. It must be stated at this WON the 56 employees have the right to vote even though some of them are not included in the
juncture that the legitimacy of Atty. Montaño's leadership as National President is beyond our list of union members submitted to the Bureau. YES
jurisdiction and is not in issue in the instant case. The only issue for our resolution is petitioner's
qualification to run as FFW National Vice‐President during the May 26‐ 27, 2001 elections. We RATIO: Submission of the employees names with the BLR as qualified members of the union is
find it necessary and imperative to resolve this issue not only to prevent further repetition but not a condition sine qua non to enable said members to vote in the election of union's officers. It
also to clear any doubtful interpretation and application of the provisions of FFW Constitution & finds no support in fact and in law. Per public respondent's findings, the April 24, 1986 list

34
consists of 158 union members only wherein 51 of the 56 challenged voters' names do not Petitioners appealed the order to respondent Trajano of BLR the disallowed expenditures of
appear. P1,278.00 were made in good faith and not used for the personal benefit of herein union officers
It is true that under article 242(c) of the Labor Code, as amended, only members of the union but, instead, contributed to the benefit of the members they were elected in 1980 only and,
can participate in the election of union officers. The question however of eligibility to vote may be therefore, they could not be made responsible for the omissions of their predecessors who failed
determined through the use of the applicable payroll period and employee's status during the to turn over union records for the questioned period there would be a general election on Oct. 4,
applicable payroll period. The payroll of the month next preceding the labor dispute in case of 1982, at which time, both the election and the desired referendum could be undertaken to
regular employees and the payroll period at or near the peak of operations in case of employees determine the membership at minimum expense they prayed that resolution on the issue be held
in seasonal industries. in abeyance
It can also be shown that their act of joining the election by casting their votes is a clear
manifestation of their intention to join the union. They must therefore be considered ipso facto Respondents claimed that Med-Arbiter erred in calling a referendum to decide issue; the
members. Said employees having exercised their right to unionism by joining ITM-MEA their appropriate action should be the expulsion of union officers
decision is paramount. Their names could not have been included in the list of employee
submitted on April 24, 1986 to the Bureau of Labor for the agreement to join the union was Trajano dismissed both appeals and affirmed in toto the order of Med-Arbiter
entered into only on May 10, 1986. Indeed the election was supervised by the Department of
Labor where said 56 members were allowed to vote. Private respondents never challenged their ISSUE:
right to vote then. Whether or not union officers were guilty of the alleged acts imputed against them thus
expulsion was proper
Kapisanan ng Manggagawang Pinagyakap v. Trajano, G.R. No. 62306, January 21, 1985.
Ruling: The Supreme Court ruled that were not guilty of the alleged acts
FACTS:
If herein union officers (also petitioners) were guilty of the alleged acts imputed against them,
On June 30, 1981, a written request for accounts examination of the financial status of KMP said public respondent pursuant to Article 242 of the New Labor Code and in the light of Our
Labor Union, the existing labor union at Franklin Baker Company in San Pablo City, was filed by ruling in Duyag vs. Inciong, 98 SCRA 522, should have meted out the appropriate penalty on
private respondent Silvestre and 13 other employees, who are members of the union them, i.e., to expel them from the Union, as prayed for, and not call for a referendum to decide
the issue;
Acting on said request, Union Account Examiner Vicedo of the MOLE conducted the
investigation and thereafter submitted a report The alleged falsification and misrepresentation of herein union officers were not supported by
substantial evidence. The fact that they disbursed the amount of P1,278.00 from Union funds
Based on the revelations, private respondents filed with the Regional Office QC, MOLE, a and later on was disallowed for failure to attach supporting papers thereon did not of itself
petition for the expulsion of the union officers constitute falsification and/or misrepresentation. The expenditures appeared to have been made
in good faith and the amount spent for the purpose mentioned in the report, if concurred in or
They committed gross violation of the Labor Code, specifically pars. (a), (b), (g), (h), (j), and (k) accepted by the members, are reasonable; and
of Article 242; and, the constitution and by-laws of the union, Sections 6 and 7
The repudiation of both private respondents to the highly sensitive position of auditor at the
The Union Officers Denied imputation and argued that the disallowed expenditures were made October 4, 1982 election, is a convincing manifestation and demonstration of the union
in good faith; that the same conduced the benefit of members membership's faith in the herein officers' leadership on one hand and a clear condonation of an
act they had allegedly committed.
That They are willing to reimburse the same from their own personal funds and They should not
be held accountable for the non-production of books of accounts of the Union for years 1977, By and large, the holding of the referendum in question has become moot and academic. This is
1978, and 1979 because they were not the officers then and not one of the former officers of the in line with Our ruling in Pascual vs. Provincial Board of Nueva Ecija, 106 Phil. 471, which We
Union had turned over to them the records quote:

And the Union officers asserted asserted Non-ratification of the constitution and by-laws of the The Court should never remove a public officer for acts done prior to his present term of office.
Union and the non-segregation of the Union funds occurred before they became officers and To do otherwise would be to deprive the people of their right to elect their officers. When the
that they have already been correcting the same people have elected a man to office, it must be assumed that they did this with knowledge of his
life and character, and that they disregarded or forgave misfaults or misconduct, if he had been
Med-Arbiter Cabibihan ordered the holding of a referendum, to be conducted under the guilty of any. It is not for the court, by reason of such faults or misconduct to practically overrule
supervision of BLR the will of the people.

35
c. Major Policy Matter 3. The contract which retroactively took effect on January 1, 1966, was executed when Atty.
• Article 250(d), Labor Code. Espinas was still handling the appeal of Halili Transit in the main case before the Supreme
Art. 250. Procedure in collective bargaining. The following procedures shall be observed in Court.
collective bargaining 4. When Atty. Pineda filed his motion for approval of his attorney's lien with Arbiter Valenzuela
on February 8, 1983, he did not attach the retainer's contract.
d. During the conciliation proceedings in the Board, the parties are prohibited from doing any act 5. The retainer's contract was not even notarized.
which may disrupt or impede the early settlement of the disputes;
A prospective buyer, the Manila Memorial Park Cemetery, Inc. objected in view of PD 1529
Halili v. Court of Industrial Relations, G.R. No. L-24864, April 30, 1985. which requires no less than an order from a court of competent jurisdiction as authority to sell
Facts: property in trust.
The cases involve disputes regarding claims for overtime of more than five hundred bus drivers
and conductors of Halili Transit. Litigation initially commenced with the filing of a complaint for Atty. Pineda, without authority from the Supreme Court but relying on the earlier authority given
overtime with the CIR. The disputes were eventually settled when the contending parties him by the Ministry of Labor, filed another urgent motion, praying that the Union be authorized to
reached an Agreement where the Administratrix would transfer to the employees the title to a sell the lot. The sale was finally consummated, resulting in the execution of an escrow
tract of land in Caloocan, Rizal. The parcel of land was eventually registered in the name of the agreement.
Union.
When Atty. Jose C. Espinas (herein movant and alleged original counsel for the Union) learned
The Union, through Atty. Benjamin C. Pineda, filed an urgent motion with the Ministry of Labor of the sale and apportionment of the proceeds from past Union president Amado Lopez, he
and Employment (MOLE) requesting for authority to sell and dispose of the property. Union requested Labor Arbiter Raymundo Valenzuela to allow him to look into the records of the case.
President Amado Lopez, in a letter, informed J.C. Espinas and Associates that the general The latter, however, told him that the records of the case were missing. Thereupon, Atty.
membership of the said Union had authorized a 20% contingent fee for the law firm based on Espinas requested Director Pascual Reyes of the NLRC to locate the records.
whatever amount would be awarded the Union.
Atty. Jose C. Espinas, (the original counsel) established the award of 897 workers' claim. When Issue:
Atty. Pineda appeared for the Union in these cases, still an associate of the law firm, his
appearance carried the firm name B.C. Pineda and Associates," giving the impression that he RULING:
was the principal lawyer in these cases.
d. Union Funds
Atty. Pineda joined the law firm of Atty. Espinas in 1965 when these cases were pending • Articles 228(b); 250; and 289, Labor Code.
resolution. He always held office in the firm's place at Puyat Building, except in 1966 to 1967
when he transferred to the Lakas ng Manggagawa Offices. During this one-year stint at the latter ART. 228. [222] Appearances and Fees.172 (a) Non-lawyers may appear before the
office, Atty. Pineda continued handling the case with the arrangement that he would report the Commission or any Labor Arbiter only:
developments to the Espinas firm. When he rejoined the law firm in 1968, he continued working 1. If they represent themselves; or
on these cases and using the Puyat Building office as his address in the pleadings. 2. If they represent their organization or members thereof.

When Atty. Pineda rejoined the Espinas firm in 1968, he did not reveal to his partners (he was (b) No attorney’s fees, negotiation fees or similar charges of any kind arising from any collective
made the most senior partner) that he had a retainer's contract. He stayed with the law firm until bargaining agreement shall be imposed on any individual member of the contracting union:
1974 and still did not divulge the 1967 retainer's contract. Only the officers of the Union knew of Provided, However, that attorney’s fees may be charged against union funds in an amount to be
the contract. agreed upon by the parties. Any contract, agreement or arrangement of any sort to the contrary
shall be null and void.
The alleged retainer's contract between Atty. Pineda and the Union appears anomalous and
even illegal as well as unethical considering that- ART. 289. [274] Visitorial Power.223 The Secretary of Labor and Employment or his duly
authorized representative is hereby empowered to inquire into financial activities of legitimate
1. The contract was executed only between Atty. Pineda and the officers of the Union chosen by labor organizations upon the filing of a complaint under oath and duly supported by the written
about 125 members only. It was not a contract with the general membership. consent of at least twenty percent (20%) of the total membership of the labor organization
2. The contingent fee of 30% for those who were still working with Halili Transit and the 45% fee concerned and to examine their books of accounts and other records to determine compliance
for those who were no longer working worked to the prejudice of the latter group who should and or non-compliance with the law and to prosecute any violations of the law and the union
were entitled to more benefits. Thus, too, when the alleged retainer's contract was executed in constitution and by- laws: Provided, That such inquiry or examination shall not be conducted
1967, the Halili Transit had already stopped operations in Metro Manila. By then, Atty. Pineda during the sixty (60) days freedom period nor within the thirty (30) days immediately preceding
knew that all the workers would be out of work which would mean that the 45% contingent fee the date of election of union officials.
would apply to all.
36
Peninsula Employees Union (PEU) v. Esquivel, G.R. No. 218454, December 1, 2016. secretary’s record of the minutes of the meeting, which shall include the list of all
members present, the votes cast, the purpose of the special assessment or fees and
FACTS: the recipient of such assessment or fees; and (c) individual written authorizations for
● On December 13, 2007, Peninsula Employees Union’ (PEU) Board of Directors check-off duly signed by the employees concemed.
passed Local Board Resolution No. 12, series of 20078 authorizing, among others, ● In the present case, however, PEU-NUWHRAIN failed to show compliance with the
the affiliation of PEU with NUWHRAIN, and the direct membership of its individual foregoing requirements. It attempted to remedy the “inadvertent omission” of the
members thereto. matter of the approval of the deduction of two percent (2%) union dues from the
● On the same day, the said act was submitted to the general membership, and was monthly basic salary of each union member.
duly ratified by 223 PEU members. Beginning January 1, 2009, PEU-NUWHRAIN ● While the matter of implementing the two percent (2%) union dues was taken up
sought to increase the union dues/agency fees from one percent (1 % ) to two percent during the PEU-NUWHRAIN’s 8th General Membership Meeting on October 28, 2008,
(2%) of the rank and file employees’ monthly salaries, brought about by PEU’s there was no sufficient showing that the same had been duly deliberated and
affiliation with NUWHRAIN, which supposedly requires its affiliates to remit to it two approved.
percent (2%) of their monthly salaries. ● The minutes of the Assembly itself belie PEU-NUWHRAIN’s claim that the increase in
● The non-PEU members objected to the assessment of increased agency fees arguing union dues and the corresponding check-off were duly approved since it merely stated
that: (a) the new CBA is unenforceable since no written CBA has been formally signed that “the [two percent (2%)] Union dues will have to be implemented,” meaning, it
and executed by PEU-NUWHRAIN and the Hotel; (b) the 2% agency fee is exorbitant would still require the submission of such matter to the Assembly for deliberation and
and unreasonable; and (c) PEU-NUWHRAIN failed to comply with the mandatory approval.
requirements for such increase. ● Having failed to establish due deliberation and approval of the increase in union dues
● The OSEC upheld PEU-NUWHRAIN's right to collect agency fees from the non-PEU from one percent ( 1 % ) to two percent (2% ), as well as the deduction of the two
members but only 1% and denied its bid to increase the agency fees to two percent percent (2%) union dues during PEU-NUWHRAIN’s 8th General Membership Meeting
(2%) for failure to show that its general membership approved the same. on October 28, 2008, there was nothing to confirm, affirm, or ratify through the July 1,
● On MR, OSEC issued an Order partially granting PEU-NUWHRAIN's motion for 2010 GMR.
reconsideration, and declaring it entitled to collect two percent (2%) agency fees from ● Corollarily, no individual check-off authorizations can proceed therefrom, and the
the non-PEU members. submission of the November 2008 check-off authorizations becomes inconsequential.
● The CA set aside the OSEC's March 6, 2012 Order, and reinstated the June 2, 2010 Jurisprudence states that the express consent of the employee to any deduction in his
Decision. It ruled that PEU-NUWHRAIN failed to prove compliance with the requisites compensation is required to be obtained in accordance with the steps outlined by the
for a valid check-off since the October 28, 2008 minutes do not show that the increase law, which must be followed to the letter; however, PEU-NUWHRAIN failed to comply.
in union dues was duly approved by its general membership. It also found the July 1, ● Thus, the CA correctly ruled that there is no legal basis to impose union dues and
2010 GMR suspicious considering that it surfaced only after PEU received the agency fees more than that allowed in the expired CBA, .e., at one percent (1 %) of
OSEC's June 2, 2010 Decision disallowing the collection of increased agency fees. the employee’s monthly basic salary.

ISSUE: Whether or not PEU-NUWHRAIN has right to collect the increased agency fees. YES. Kaisahan at Kapatiran ng mga Manggagawa at Kawani sa MWC-East Zone Union v. Manila
Water Company, Inc., G.R. no. 174179, November 16, 2011.
HELD:
● The recognized collective bargaining union which successfully negotiated the CBA FACTS:
with the employer is given the right to collect a reasonable fee called “agency fee”
from non-union members who are employees of the appropriate bargaining unit, in an ● The Union is the duly-recognized bargaining agent of the rank-and-file employees of
amount equivalent to the dues and other fees paid by union members, in case they the respondent Manila Water Company, Inc. (Company) while Borela is the Union
accept the benefits under the CBA. President. Metropolitan Waterworks and Sewerage System (MWSS) entered into a
● While the collection of agency fees is recognized by Article 259 (formerly Article 248) Concession Agreement (Agreement) with the Company to privatize the operations of
of the Labor Code, as amended, the legal basis of the union’s right to agency fees is the MWSS.
neither contractual nor statutory, but quasi-contractual, deriving from the established ● Article 6.1.3 of the Agreement provides that the Concessionaire shall grant its
principle that non-union employees may not unjustly enrich themselves by benefiting employees benefits no less favorable than those granted to MWSS employees at the
from employment conditions negotiated by the bargaining union. In the present case, time of their separation from MWSS such as amelioration allowance (AA) and the
PEU-NUWHRAIN’s right to collect agency fees is not disputed. cost-of-living allowance (COLA).
● Case law interpreting Article 250 (n) and ( o ) of the Labor Code mandates the ● The payment of the AA and the COLA was discontinued pursuant to Republic Act No.
submission of three (3) documentary requisites in order to justify a valid levy of 6758, otherwise known as the Salary Standardization Law, which integrated the
increased union dues. allowances into the standardized salary. Nonetheless, the Union demanded from the
● These are: (a) an authorization by a written resolution of the majority of all the Company the payment of the AA and the COLA during the renegotiation of the parties
members at the general membership meeting duly called for the purpose; (b) the Collective Bargaining Agreement (CBA).
37
● The company subsequently agreed for the payment of the AA and COLA. However, bearing at all to the attorney's fees awarded by the NLRC under Article 111 of the
did not pay said benefits because COA disapproved their payment because the Labor Code.
company had no funds to cover this benefit. ● The Company's argument that the attorney's fees are unconscionable as they
● A complaint against the Company for payment of the AA, COLA, moral and exemplary represent 20% of the amount due or about P21.4 million is more apparent than real.
damages, legal interest, and attorney's fees before the National Labor Relations Since the attorney's fees awarded by the LA pertained to the Union's members as
Commission (NLRC). indemnity for damages, it was totally within their right to waive the amount and give it
● LA ruled in favor of the petitioners and ordered the payment of their AA and COLA, six to their counsel as part of their contingent fee agreement.
percent (6%) interest of the total amount awarded, and ten percent (10%) attorney's ● Beyond the limit fixed by Article 111 of the Labor Code, such as between the lawyer
fees. NLRC confirmed. and the client, the attorney's fees may exceed ten percent (10%) on the basis of
● The Company pointed out that the award of ten percent (10%) attorney's fees to the quantum meruit, as in the present case. The SC reversed CA decision and reinstated
petitioners is already provided for in their Memorandum of Agreement (MOA) which LA’s award.
mandated that attorney's fees shall be deducted from the AA and CBA receivables.
● The petitioners argued that the MOA only covered the payment of their share in the Mariño v. Gamilla, G.R. No. 149763, July 7, 2009.
contracted attorney's fees, but did not include the attorney's fees awarded by the
NLRC. FACTS:
● The CA modified the assailed NLRC rulings by deleting the order for respondent ● Petitioners were among the executive officers and directors (collectively called the
MWCI to pay attorney's fees equivalent to 10% of the total judgment awards. Mariño Group) of the University of Sto. Tomas Faculty Union (USTFU), a labor union
duly organized and registered under the laws of the Republic of the Philippines and
ISSUE: Whether or not the NLRC gravely abused its discretion in awarding ten percent (10%) the bargaining representative of the faculty members of the University of Santo Tomas
attorney's fees to the petitioners. NO. (UST). Respondents were UST professors and USTFU members.
● The 1986 CBA between UST and USTFU expired on 31 May 1988. Thereafter,
HELD: bargaining negotiations ensued between UST and the Mariño Group, which
● There are two commonly accepted concepts of attorney's fees. In its ordinary represented USTFU. They were not able to reach an agreement and a bargaining
concept, an attorney's fee is the reasonable compensation paid to a lawyer by his deadlock was declared.
client for the legal services the former renders; compensation is paid for the cost ● On 1992, UST and USTFU executed a (MOA), whereby UST faculty members
and/or results of legal services per agreement or as may be assessed. In its belonging to the collective bargaining unit were granted additional economic benefits
extraordinary concept, attorney's fees are deemed indemnity for damages ordered for the fourth and fifth years of the 1988-1993 CBA, specifically, the period from 1
by the court to be paid by the losing party to the winning party. June 1992 up to 31 May 1993.
● Article 111 of the Labor Code, as amended, contemplates the extraordinary concept
of attorney's fees and that Article 111 is an exception to the declared policy of strict MEMORANDUM OF AGREEMENT
construction in the award of attorney's fees. Although an express finding of facts and 1.0. The University hereby grants additional benefits to Faculty Members belonging to the
law is still necessary to prove the merit of the award, there need not be any showing collective bargaining unit xxx, which additional benefits shall amount in the aggregate to
that the employer acted maliciously or in bad faith when it withheld the wages. P42,000,000.00[.]
● Settled is the rule that in actions for recovery of wages, or where an employee was 7.0.It is clearly understood and agreed upon that the
forced to litigate and, thus, incur expenses to protect his rights and interests, a aggregate sum of P42 million is chargeable against the share of the faculty members in the
monetary award by way of attorney's fees is justifiable under Article 111 of the Labor incremental proceeds of tuition fees collected and still to be collected; xxx and incremental
Code; Section 8, Rule VIII, Book III of its Implementing Rules; and paragraph 7, Article proceeds are, by law and pertinent (DECS) regulations, required to be allotted for the payment
2208 of the Civil Code. The award of attorney's fees is proper, and there need not be of salaries, wages, allowances and other benefits of teaching and non-teaching personnel for
any showing that the employer acted maliciously or in bad faith when it withheld the the UNIVERSITY.
wages. There need only be a showing that the lawful wages were not paid
accordingly. ● On 12purportedly signified their consent to the economic benefits granted under the
● In the present case, we find it undisputed that the union members are entitled to their MOA. USTFU, through its President, petitioner Atty. Mariño, wrote a letter to the UST
AA benefits and that these benefits were not paid by the Company. That the Treasurer requesting the release to the union of the sum ofP4.2 million, which was
Company had no funds is not a defense as this was not an insuperable cause that 10% of the P42 million economic benefits package granted by the MOA to faculty
was cited and properly invoked. members belonging to the collective bargaining unit. UST remitted the sum of P4.2
● As a consequence, the union members represented by the Union were compelled to million to USTFU. After deducting from the P42 million economic benefits package
litigate and incur legal expenses.On these bases, we find no difficulty in upholding the several expenses, a net amount of P6, 389,145.04 remained and was distributed to
NLRC's award of ten percent (10%) attorney's fees. the faculty members.
● The attorney's fees contracted under the MOA do not refer to the amount of attorney's ● On 15 December 1994, respondents filed with the Med-Arbiter, DOLE- National
fees awarded by the NLRC; the MOA provision on attorney's fees does not have any Capital Region (NCR), a Complaint for the expulsion of the Mariño Group as USTFU
38
officers and directors. Alleged in their Complaint that the Mariño Group violated the benefits package was derived from sources other than the 70% allotment from tuition
rights and conditions of membership in USTFU, particularly by: 1) investing the fee increases of UST.
unspent balance of the P42 million economic benefits package given by UST without ● Given the lack of evidence to the contrary, it can be conclusively presumed that the
prior approval of the general membership; 2) simultaneously holding elections viva entire P42 million economic benefits package extended to USTFU came from the 70%
voce; 3) ratifying the CBA involving the P42 million economic benefits package; and 4) allotment from tuition fee increases of UST. Preceding from this presumption, any
approving the attorney’s/agency fees worth P4.2 million in the form of check-off. deduction from the P42 million economic benefits package, such as the P4.2 million
● On 16 December 1994, UST and USTFU, represented by the Mariño Group, entered claimed by the Mariño Group as attorney’s/agency fees, should not be allowed,
into a new CBA, effective 1 June 1993 to 31 May 1998 (1993-1998 CBA). because it would ultimately result in the reduction of the statutorily mandated 70%
● 24 September 1996, petitioner Collantes, as USTFU Secretary- General, posted allotment from the tuition fee increases of UST.
notices in some faculty rooms at UST, informing the union members of a general ● General rule is that attorney’s fees, negotiation fees, and other similar charges may
assembly. The agenda was the election of new On October 4 1996, the UST only be collected from union funds, not from the amounts that pertain to individual
Secretary General headed a general faculty assembly. Respondents were among the union members. As an exception, special assessments or other extraordinary fees
elected officers of USTFU (collectively referred to as the Gamilla Group). Med-Arbiter may be levied upon or checked off from any amount due an employee for as long as
DOLE-NCR, nullified the election of the Gamilla Group as USTFU officers. Affirmed on there is proper authorization by the employee.
appeal by the (BLR). ● A check-off is a process or device whereby the employer, on agreement with the
● Respondents filed before the Med-Arbiter, DOLE-NCR, a complaint against the Mariño Union, recognized as the proper bargaining representative, or on prior authorization
Group, as well as the Philippine Foundation for the Advancement of the Teaching from the employees, deducts union dues or agency fees from the latter's wages and
Profession, Inc., Security Bank Corporation, and Bank of the Philippine Islands remits them directly to the Union.
claiming that they were the legitimate USTFU officers, having been elected on 4 ● The Court finds that, in the instant case, the P42 million economic benefits package
October 1996. They prayed for an order directing the Mariño Group to cease and granted by UST did not constitute union funds from whence the P4.2 million could
desist from using the name of USTFU and from performing acts for and on behalf of have been validly deducted as attorney’s fees. The P42 million economic benefits
the USTFU and the rest of the members of the collective bargaining unit. package was not intended for the USTFU coffers, but for all the members of the
● The DOLE-NCR Reg. Dir. Held that the Mariño Group, as the executive officers of bargaining unit USTFU represented, whether members or non-members of the union.
USTFU, guilty of violating the provisions of the USTFU Constitution and By-laws. Also, ● A close reading of the terms of the MOA reveals that after the satisfaction of the
they violated Article 241(c) and (l) of the Labor Code when among others, they outstanding obligations of UST under the 1986 CBA, the balance of the P42 million
invested in a bank, without prior consent of USTFU members, the sum of was to be distributed to the covered faculty members of the collective bargaining unit
P9,766,570.01, which formed part of the P42 million economic benefits package. in the form of salary increases, returns on paycheck deductions; and increases in
Additionally, the check-off of P4.2 million collected by the Mariño Group, as hospitalization, educational, and retirement benefits, and other economic benefits.
negotiation fees, was invalid. Under Republic Act No. 6728, 70% of the tuition fee ● The deduction of the P4.2 million, as alleged attorney’s/agency fees, from the P42
increases should be allotted to academic and non-academic personnel. million economic benefits package effectively decreased the share from said package
● Given that the records were silent as to how much of the P42 million economic accruing to each member of the collective bargaining unit.
benefits package was obtained through negotiations and how much was from the ● The Court further determines that the requisites for a valid levy and check-off of
statutory allotment of 70% of the tuition fee increases, the DOLE-NCR Regional special assessments, laid down by Article 241(n) and (o), respectively, of the
Director held that the entire amount was within the statutory allotment, which could not Labor Code, as amended, have not been complied with in the case at bar. To recall,
be the subject of negotiation and, thus, could not be burdened by negotiation fees. these requisites are: (1) an authorization by a written resolution of the majority of all
● Bureau of Labor Relations agreed with DOLE-NCR Regional Director. The CA the union members at the general membership meeting duly called for the purpose;
affirmed the BLR decision. (2) secretary's record of the minutes of the meeting; and (3) individual written
authorization for check-off duly signed by the employee concerned.
ISSUE: Whether or not the 10% check-off collected by the Mariño Group from the P42 million ● In an attempt to comply with the foregoing requirements, the Mariño Group caused the
economic benefits package is valid. NO. majority of the general membership of USTFU to individually sign a document, which
embodied the ratification of the MOA between UST and USTFU, dated 10 September
HELD: 1992, as well as the authorization for the check-off of P4.2 million, from the P42
million economic benefits package, as payment for attorney’s fees.
● As held by the Court of Appeals, however, the said documents constitute
● UST and USTFU stipulated in their 10 September 1992 MOA that the P42 million
unsatisfactory compliance with the requisites set forth in the Labor Code, as
economic benefits package granted by UST to the members of the collective
amended, and in the USTFU Constitution and By-Laws, even though individually
bargaining unit represented by USTFU, was chargeable against the 70% allotment
signed by a majority of USTFU members.
from the proceeds of the tuition fee increases collected and still to be collected by
● The inclusion of the authorization for a check-off of union dues and special
UST.
assessments for the Labor Education Fund and attorney’s fees, in the same document
● As observed by the DOLE-NCR Regional Director, and affirmed by both the BLR and
the Court of Appeals, there is no showing that any portion of the P42 million economic
39
for the ratification of the 10 September 1992 MOA granting the P42 million economic organization. As this Court has acknowledged, the system of check-off is primarily for
benefits package, necessarily vitiated the consent ofUSTFU members. the benefit of the union and only indirectly for the individual employees. The pertinent
● The failure of the Mariño Group to strictly comply with the requirements set forth by legal provisions on check-offs are found in Article 222 (b) and Article 241 (o) of the
the Labor Code, as amended, and the USTFU Constitution and By- Laws, invalidates Labor Code.
the questioned special assessment. Substantial compliance is not enough in view of ● Art. 241 has three (3) requisites for the validity of the special assessment for union’s
the fact that the special assessment will diminish the compensation of the union incidental expenses, attorney’s fees and representation expenses. These are: (1)
members. Their express consent is required, and this consent must be obtained in authorization by a written resolution of the majority of all the members at the general
accordance with the steps outlined by law, which must be followed to the letter. No membership meeting called for the purpose; (2) secretary’s record of the minutes of
shortcuts are allowed. the meeting; and (3) individual written authorization for check off duly signed by the
employees concerned.
Gabriel v. The Honorable Secretary of Labor and Employment, G.R. No. 115949, March 16, ● Clearly, attorney’s fees may not be deducted or checked off from any amount due to
2000. an employee without his written consent.
FACTS: ● The Court finds that the General Membership Resolution of October 19, 1991 of the
● Petitioners and 5 others compromise the Executive Board of Solidbank union, the SolidBank Union did not satisfy the requirements laid down by law and jurisprudence
collective bargaining agent for the Solidbank Corporation. Private respondents are for the validity of the ten percent (10%) special assessment for union’s incidental
members of said union. expenses, attorney’s fees and representation expenses. There were no individual
● The union’s Executive Board decided to retain the services of their counsel, Atty. written check off authorizations by the employees concerned and so the assessment
Lacsina, in connection with negotiations for a new CBA. cannot be legally deducted by their employer.
● A general membership meeting was called where majority of union members ● Article 222 (b) of the Labor Code prohibits the payment of attorney's fees only when it
approved a resolution confirming the decision to engage the services of the union’s is effected through forced contributions from workers from their own funds as
counsel. The resolution provided that 10% of the total economic benefits that may be distinguished from the union funds.
secured be given to the counsel at attorney’s fees. Also, it contained an authorization ● The purpose of the provision is to prevent imposition on the workers of the duty to
for Solidbank Corporation to check-off said attorney’s fees the first lump sum of individually contribute their respective shares in the fee to be paid the attorney for his
payment of benefits to the employees under the new CBA as well as turnover said services on behalf of the union in its negotiations with management. The obligation to
amount to Atty. Lacsina or his representative. pay the attorney's fees belongs to the union and cannot be shunted to the workers as
● Private respondents instituted a complaint against the petitioners and the union their direct responsibility. Neither the lawyer nor the union itself may require the
counsel before the Department of Labor and Employment (DOLE) for illegal deduction individual worker to assume the obligation to pay attorney's fees from their own
of attorney’s fees as well as for quantification of the benefits in the 1992 CBA. pockets.
Petitioners moved for dismissal. ● Thus the SC held that the SOLE is correct in ruling that the workers through their
● Med-arbiter granted the complaint; Secretary of Labor partially granted and the Order union should be made to shoulder the expenses incurred for the services of a lawyer.
of the Med-Arbiter dated 22 April 1993 is hereby modified as follows: (1) that the And accordingly, the reimbursement should be charged to the union's general fund or
ordered refund shall be limited to those union members who have not signified their account. No deduction can be made from the salaries of the concerned employees
conformity to the check-off of attorney’s fees; and (2) the directive on the payment of other than those mandated by law.
5% attorney’s fees should be deleted for lack of basis.
● On MR: affirmed resolution with following modification: (1) the union’s counsel be Vengco v. Trajano, G.R. No. 74453, May 5, 1989.
dropped as party litigant; (2) the workers through their union should be made to FACTS:
shoulder the expenses incurred for the attorney's services. Accordingly, the ● The Management of the Anglo-American Tobacco Corporation and the Kapisanan ng
reimbursement should be charged to the union's general fund/account. Manggagawa sa Anglo-American Tobacco Corporation (FOITAF) entered into a
compromise agreement.
ISSUE: Whether or not the SOLE erred in ruling that the workers through their union should be ● The company is to pay the union members the sum of P150k for their claims arising
made to shoulder the expenses incurred for the attorney's services (for the CB negotiations) and from the unpaid emergency cost of living allowance (ECOLA) and other benefits which
that the reimbursement should be charged to the union's general fund/account. NO. were the subject of their complaint before the Ministry of Labor.
● Respondent Emmanuel Timbungco (union president) received the money paid in
HELD: installments and distributed the amount among the union members.
● Private respondent’s contention: claim that the check-off provision in question is illegal ● Petitioners Vengco, et al., who are union members noted that Timbungco was not
because it was never submitted for approval at a general membership meeting called authorized by the union workers to get the money; and that ten percent (10%) of the
for the purpose and that it failed to meet the formalities mandated by the Labor Code. P150k had been deducted to pay for attorney's fees without their written authorization
● In check-off, the employer, on agreement with the Union, or on prior authorization in violation of Article 242(o) of the Labor Code.
from employees, deducts union dues or agency fees from the latter’s wages and
remits them directly to the union. It assures continuous funding; for the labor
40
● They demanded from Timbungco an accounting of how the P150k was distributed to "CLAIM" bukod pa sa marami kaming naghati-hati sa nasabing halaga ipinapasiya
the members. Timbungco did not give in to their demand. Thus Vengco, et al. filed a naming na kusang-loob na kunin ang aming bahagi sa aming kapisanan sa unang
complaint with the Ministry of Labor. linggo ng Disyembre, 1981 at ito'y ipinaalam namin sa Pangulo ng Kapisanan na si
● Timbungco alleged among others, that he was authorized by a resolution signed by Ginoong Emmanuel Timbungco.
the majority of the union members to receive and distribute the P150k among the ● The above-quoted statement merely indicated the intention of the workers to get their
workers; that the (10%) attorney's fees was in relation to the claim of the local union claim on the first week of December, 1981 and to inform Timbungco of their intention.
for payment of emergency cost of living allowance before the Ministry of Labor which Clearly, this statement cannot be construed to confer upon Timbungco the authority to
is totally distinct and separate from the negotiation of the CBA; and that the (10%) receive the fringe benefits for the workers.
deduction was in accordance with Section II, Rule No. VIII, Book No. III of the Rules ● Absent such authority, Timbungco should not have kept the money to himself but
and Regulations implementing the Labor Code and therefore, no authorization from should have turned it over to the Union Treasurer. He, therefore, exceeded his
the union members is required. authority as President of the Union.
● Med-arbiter dismissed the complaint. Vengco et al appealed to the BLR. Director of
the Bureau of Labor Relations granted the appeal and ordered Timbungco to render a Galvadores v. Trajano, G.R. No. 70067, September 15, 1986.
full accounting of the 150k and to publish in the union’s bulletin board the list of all
recipient union members and the respective amounts they have received, within ten Pacific Banking Corporation v. Clave, G.R. No. L-56965, March 7, 1984.
(10) days from receipt. Doctrine: Attorney’s fees may be charged against union funds in an amount to be agreed upon
● OIC Calaycay set aside the decision and ordered an audit examination of the books of by the parties. Any contract, agreement or arrangement of any sort to the contrary shall be null
account of the union. and void.
● Vengco, et al, sought reconsideration of the order and contended that the examination
of the books of accounts of the union is irrelevant considering that the issue involved Facts:
in the case does not consist of union funds but back pay received by the union · This case is about the legality of deducting from the monetary benefits
members from the company. awarded in a collective bargaining agreement the attorney’s fees of the
● Dir. Trajano denied the MR and affirmed the audit of the books of account. lawyer who assisted the union president in negotiating the agreement.

ISSUE: Whether or not Timbungco is guilty of illegally deducting 10% attorneys' fees from
petitioners' backwages. YES · 1979, there had been negotiations between the Pacific Banking
Corporation and the Pacific Banking Corporation Employees
HELD: Organization (PABECO) for a collective bargaining agreement for 1979
● Timbungco maintains that the "Kapasiyahan" gave him the authority to make the to 1981. Because of a deadlock, the Minister of Labor assumed
deduction. This contention is unfounded. Contrary to his claim, the undated jurisdiction over the controversy. The Deputy Minister rendered a
"Kapasiyahan" or resolution did not confer upon him the power to deduct 10% of the decision directing the parties to execute a CBA in accordance with the
P150,000.00 despite the alleged approval of the majority of the union workers. Article terms and conditions set forth in his decision
241 (o) of the Labor Code provides: · Lawyer Juanito M. Saavedra’s earliest recorded participation in the
● ART. 241. Rights and conditions of membership in a labor organization. — The case was on July 15 and 27, 1979 when he filed a motion for
following are the rights and conditions of membership in a labor organization. reconsideration and a supplemental motion in behalf of the union.
xxx (o) Other than for mandatory activities under the Code, no special assessment, · Even before the formalization of the CBA on June 3, 1980, Saavedra
attorney's fees, negotiation fees or any other extraordinary fees may be checked off on March 24, 1980 filed in the case his notice of attorney ‘s lien.
from any amount due an employee without an individual written authorization duly · The union officials requested the bank to withhold around P345,000
signed by an employee. The authorization should specifically state the amount, out of the total benefits as ten percent attorney’s fees of Saavedra. At
purpose and beneficiary of the deduction. first, the bank interposed no objection to the request in the interest of
● It is very clear from the provision that attorney's fees may not be deducted or checked harmonious labor-management relations.In theory, the actual ten
off from any amount due to an employee without his written consent except for percent attorney’s fees may amount to more than one million pesos
mandatory activities under the Code. A mandatory activity has been defined as a · For nearly a year, the Office of the President in four resolutions
judicial process of settling dispute laid down by the law. wrestled with the propriety of Saavedra’s ten percent attorney’s fees. In
● It requires the individual written authorization of each employee concerned, to make a resolution dated May 29, 1980, Presidential Executive Assistant
the deduction of attorney's fees valid. Jacobo C. Clave refused to intervene in the matter. He ruled that the
● Likewise, the court finds that the other "Kapasiyahan" dated September 18,1981 payment of attorney’s fees was a question that should be settled by the
submitted by Timbungco belied his claim that he was authorized by the union workers union and its lawyer themselves as provided in Artile111 of the Labor
to receive the sum of P150,000.00 on their behalf. The pertinent portion of the said Code: Art. 111. Attorney’s fees. — (a) In cases of unlawful withholding
"Kapasiyahan" provides: 3. Na sa dahilang hindi bigla ang pagbabayad sa nasabing
41
of wages the culpable party may be assessed attorney’s fees recipient of such assessment or fees. The record shall be attested to by the
equivalent to ten percent of the amount of wages recovered. president;.
"(b) It shall be unlawful for any person to demand or accept, in any judicial
or administrative proceedings for the recovery of wages, attorney’s "(o) Other than for mandatory activities under the Code, no
fees which exceed ten percent of the amount of wages special assessment, attorney’s fees, negotiation fees or any other
recovered."cralaw virtu extraordinary fees may be checked off from any amount due an employee
· The last resolution dated April 13, 1981, Deputy Presidential Executive without an individual written authorization duly signed by the employee. The
Assistant Joaquin T. Venus, Jr. ordered the bank to pay the union authorization should specifically state the amount, purpose and beneficiary
treasurer the said attorney’s fees less the amounts corresponding to of the deduction;
the protesting employees. He held that the following article 222 of the · There is no doubt that lawyer Saavedra is entitled to the payment of his fees but
Labor Code, as amended by Presidential Decree No. 1691, effective article 222 ordains that union funds should be used for that purpose. The amount
May 1, 1980 (before the formalization of the CBA award) had no of P345,000 does not constitute union funds. It is money of the employees. The
retroactive effect to the case:jgc:chanrobles.com.ph union, not the employees, is obligated to Saavedra.

"ART. 222. Appearances and Fees. — . . . (b) No attorney’s fees,


negotiation fees or similar charges of any kind arising from any e. Union Information
collective bargaining negotiations or conclusion of the collective
agreement shall be imposed on any individual member of the Art. 250. Procedure in collective bargaining. The following procedures shall be observed in
contracting union: Provided, however, that attorney’s fees may be collective bargaining:
charged against union funds in an amount to be agreed upon by the
parties. Any contract, agreement or arrangement of any sort to the 1. When a party desires to negotiate an agreement, it shall serve a written notice upon
contrary shall be null and void." the other party with a statement of its proposals. The other party shall make a reply
thereto not later than ten (10) calendar days from receipt of such notice;
Issue: Whether or not the attorney’s fees can be deducted from the monetary benefits
from the CBA? 2. Should differences arise on the basis of such notice and reply, either party may
request for a conference which shall begin not later than ten (10) calendar days from
the date of request.
Ruling: NO.
· Under the circumstances, the Office of the President had no jurisdiction to make 3. If the dispute is not settled, the Board shall intervene upon request of either or both
an adjudication on Saavedra’s attorney’s fees. The case was appealed with parties or at its own initiative and immediately call the parties to conciliation meetings.
respect to the CBA terms and conditions, not with respect to attorney’s fees. The Board shall have the power to issue subpoenas requiring the attendance of the
Although the fees were a mere incident, nevertheless, the jurisdiction to fix the parties to such meetings. It shall be the duty of the parties to participate fully and
promptly in the conciliation meetings the Board may call;
same and to order the payment thereof was outside the pale of Clave’s appellate
jurisdiction. He was right in adopting a hands-off attitude in his first resolution and
holding that the payment of the fees was a question between the lawyer and the 4. During the conciliation proceedings in the Board, the parties are prohibited from doing
any act which may disrupt or impede the early settlement of the disputes; and
union.
· the case is covered squarely by the mandatory and explicit prescription of article
222 which is another guarantee intended to protect the employee against 5. The Board shall exert all efforts to settle disputes amicably and encourage the parties
to submit their case to a voluntary arbitrator. (As amended by Section 20, Republic
unwarranted practices that would diminish his compensation without his Act No. 6715, March 21, 1989)
knowledge and consent as provided in Art 242 of the Labor Code:
ART. 242. Rights and conditions of membership in a labor
organization. — The following are the rights and conditions of membership
in a labor organization:jgc:chanrobles.com.ph
Continental Cement Corporation Labor Union (NLU) v. Continental Cement Corporation,
G.R. No. 51544, August 30, 1990.
"(n) No special assessment or other extraordinary fees may be
levied upon the members of a labor organization unless authorized by a
f. Enforcement and Remedies
written resolution of a majority of all the members at a general membership
• Articles 250, last paragraph;
meeting duly called for the purpose. The secretary of the organization shall
Art 252, LC
record the minutes of the meeting including the list of all members present,
Meaning of duty to bargain collectively. The duty to bargain collectively means the
the votes cast, the purpose of the special assessment or fees and the
performance of a mutual obligation to meet and convene promptly and expeditiously in good
42
faith for the purpose of negotiating an agreement with respect to wages, hours of work and all profit share through the union which the union failed to distribute completely. These actions was
other terms and conditions of employment including proposals for adjusting any grievances or lead by the union president, Rica Manalad. Manalad also allowed disbursements of retirement
questions arising under such agreement and executing a contract incorporating such funds to some members without the concurrence of the required number of members. While this
case was pending, Manalad and the union treasurer, Leano, presented to the court that their
agreements if requested by either party but such duty does not compel any party to agree to a
actions were already ratified by 90% of the members, the med arbiter however, though this case
proposal or to make any concession. is already moot still chose to look in the case since these are violations of the constitutional by
laws of the union.
Art 289,LC
Who are liable when committed by other than natural person. If the offense is committed by As the case progress, it was discovered that Manalad had other union memberships and offices
a corporation, trust, firm, partnership, association or any other entity, the penalty shall be assumed in other unions and labor organization which she had questionable transactions with
imposed upon the guilty officer or officers of such corporation, trust, firm, partnership, using their union. With the given reasons, the Med Arbiter ordered the removal of the private
respondents as officers of the union and directed them to reimburse to the members thereof the
association or entity.
amounts illegally collected from them.

Art 303,LC On appeal with the Director of Labor Relations, it was reversed. The Director held that resort to
ART. 303. [288] Penalties.229 Except as otherwise provided in this Code, or unless the acts intra-union remedies is not necessary and that the five complainants have the rights and
complained of hinge on a question of interpretation or implementation of ambiguous provisions personality to institute the proceedings for the removal of the respondents, to recover the
of an existing collective bargaining agreement, any violation of the provisions of this Code amount illegally collected or withheld from them and to question illegal disbursements and
declared to be unlawful or penal in nature shall be punished with a fine of not less than One expenditure of union funds.
Thousand Pesos (P1,000.00) nor more than Ten Thousand Pesos (P10,000.00), or
However, the Director ruled that the power to remove the union officers rests in the members
imprisonment of not less than three months nor more than three years, or both such fine and and that the Bureau of Labor Relations generally has nothing to do with the tenure of union
imprisonment at the discretion of the court. officers which "is a political question". The Director further ruled that his office has jurisdiction to
In addition to such penalty, any alien found guilty shall be summarily deported upon completion look into the charge of illegal disbursements of union funds. He directed the Labor Organization
of service of sentence. Division of the Bureau to examine the books of account and financial records of the union and to
Any provision of law to the contrary notwithstanding, any criminal offense punished in this Code submit a report on such examination. This decision was affirmed by the Under Secretary of
shall be under the concurrent jurisdiction of the Municipal or City Courts and the Courts of First Labor.
Instance.
ISSUE: Whether or not the Director of Labor Relations is correct in reversing the decision of the
Med Arbiter?
ART. 304. [289] Who are Liable When Committed by Other Than Natural Person. If the offense
is committed by a corporation, trust, firm, partnership, association or any other entity, the penalty
shall be imposed upon the guilty officer or officers of such corporation, trust, firm, partnership, HELD:
No, the SC held that the Labor Arbiter did not err in removing the respondents as union officers.
association or entity.
The membership of Manalad and Puerto in another union is a sufficient ground for their removal
under the constitution and by-laws of the union. In Manalad's case, his organization of a family-
Mendoza v. Officers of Manila Water Employees Union (MWEU), G.R. No. 201595, January owned corporation competing with. the union headed by him renders it untenable that he should
25, 2016. remain as union president.The petitioners are entitled to the refund of the union dues illegally
collected from them. The union should be the proper refund.bThe Director of Labor Relations
Diokno v. Cacdac, G.R. No. 168475, July 4, 2007. erred in holding that, as a matter of policy, the tenure of union office being a "political question
is, generally, a matter outside his Bureau's jurisdiction and should be pa upon by the union
members themselves.
Employees Union of Bayer Philippines, FFW v. Bayer Philippines, Inc., G.R. No. 162943,
June 15, 2005 WHEREFORE, (1) that portion of the decision of the med-arbiter, removing respondents
Manalad, Leano and Puerto as union officers, is affirmed. (Respondent Amparo is no longer an
Rodriguez v. Galvadores, G.R. Nos. 76579-82, August 31, 1988. officer of the union.)
(2) We also affirm that portion of the decision of the Director of Labor Relations, directing the
Duyag v. Inciong, G.R. No. L-47775, July 5, 1980. Bureau's Labor Organization Division to examine the books of accounts and records of the
FACTS: Associated Port Checkers and Workers Union and to submit a report on such examination within
The plaintiffs in this case are arrastre checkers at E. Razon, Inc. in the South Harbor, Port Area, a reasonable time.
Manila as well as bona fide members of the Associated Port Checkers and Workers Union. (3) We declare that the five petitioners are entitled to a refund of the union dues illegally
Meanwhile, the defendants are the officers of the union, who, respectively, are the president (for collected from them. The Director of Labor Relations is ordered to require the union to make the
more than twenty years), treasurer, vice-president and auditor of the union. refund within twenty days from notice to his counsel of the entry of judgment in this case. Costs
against the private respondents.
The reason for this case are the unauthorized increase in their monthly union contributions, and
deduction on their midyear bonus as contribution to the union. These increases and deduction
happened in the years 1972, 1973, 1975, and 1976. Also, E. Razon remitted the employees’ 8. Union Chartering and Affiliation
43
a. Nature and Purpose Policy considerations dictate that in weighing the claims of a local union as against those of a
national federation, those of the former must be preferred. Parenthetically though, the desires of
Philippine Skylanders, Inc. v. National Labor Relations Commission, G.R. No. 127374, the mother federation to protect its locals are not altogether to be shunned. It will however be to
err greatly against the Constitution if the desires of the federation would be favored over those of
January 31, 2002.
its members. If it were otherwise, instead of protection, there would be disregard and neglect of
FACTS: the lowly workingmen.
Nov 1993, the Philippine Skylanders Employees Association (PSEA), a local labor union Disposition Petition is granted.
affiliated with the Philippine Association of Free Labor Unions (PAFLU) September (PAFLU),
won in the certification election conducted among the rank and file employees of Philippine
Skylanders, Inc. (PSI). Its rival union, Philippine Skylanders Employees Association-WATU
(PSEA-WATU) immediately protested the result of the election before the Secretary of Labor. Filipino Pipe and Foundry Corporation v. National Labor Relations Commission, G.R. No.
115180, November 16, 1999.
Several months later, pending settlement of the controversy, PSEA sent PAFLU a notice of FACTS:
disaffiliation citing as reason PAFLU's supposed deliberate and habitual dereliction of duty - National Labor Union-Trade Union Congress of the Philippines (NLU-TUCP) filed w/ the
toward its members. PSEA subsequently affiliated itself with the National Congress of Workers Ministry of Labor and Employment, in behalf of its local chapter, the Filipino Pipe Workers Union-
(NCW), changed its name to Philippine Skylanders Employees Association - National Congress
National Labor Union (FPWU-NLU), a notice of strike signed by its national president, Atty.
of Workers (PSEANCW), and allowed the former officers of PSEA-PAFLU to continue occupying
their positions as elected officers PSEA-NCW. Lerum, against Filipino Pipe and Foundry Corporation (Fil Pipe), alleging union busting and non
On 17 March 1994 PSEA-NCW entered into a collective bargaining agreement with PSI which implementation of the Collective Bargaining Agreement. A conciliation conference was set, but
was immediately registered with DOLE. before the conciliation was done, the FPWU-NLU staged a strike.
Fil Pipe filed petition to declare the strike illegal and for damages against FPWU-NLU, NLU-
PAFLU Secretary General Serafin Ayroso wrote Mariles C. Romulo requesting a copy of PSI's TUCP and Lerum. Later, Fil Pipe moved to dismiss the complaint against FPWU-NLU.
audited financial statement. Ayroso explained that with the dismissal of PSEA-WATU's election Labor Arbiter declared the strike illegal, and ordered NLU-TUCP to pay damages. Lerum was
protestthe time was ripe for the parties to enter into a collective bargaining agreement. PSI
absolved. Appeals were filed. NLRC set aside decision for lack of merit in so far as NLU-TUCP
through its personnel manager Francisco Dakila denied the request citing as reason PSEA's
disaffiliation from PAFLU and its subsequent affiliation with NCW. PAFLU through Serafin and Lerum is concerned.
Ayroso filed a complaint for unfair labor practice against PSI, its president Mariles Romulo and Filipino Pipe: NLRC erred in declaring that NLU-TUCP and Lerum are not primarily responsible,
personnel manager Francisco Dakila. PAFLU amended its complaint by including the elected and that the company has lost its cause of action. They argued that the responsibility for the
officers of PSEA-PAFLU as additional party respondents. damages allegedly sustained by petitioner company on account of the illegal strike, should be
borne by NLU-TUCP and Lerum, for having directly participated in aiding and abetting the illegal
Labor Arbiter declared PSEA's disaffiliation from PAFLU invalid and held PSI, PSEA-PAFLU and strike. Also, that FPWUNLU is a mere agent of respondent NLU-TUCP, because FPWUNLU, a
their respective officers guilty of unfair labor practice.
local union, cannot act as the principal of respondent NLUTUCP, a mother federation, because it
NLRC upheld the Decision of the Labor Arbiter and conjectured that since an election protest is not a legitimate labor organization.
questioning PSEA-PAFLU's certification as the sole and exclusive bargaining agent was pending
resolution before the Secretary of Labor, PSEA could not validly separate from PAFLU, join ISSUES:
another national federation and subsequently enter into a collective bargaining agreement with 1. Whether or not the strike was illegal
its employer-company. 2. Whether or not NLU-TUCP and Lerum are primarily responsible

ISSUE: Whether or not PSEA, which is an independent and separate local union, may validly
HELD:
disaffiliate from PAFLU pending the settlement of an election protest questioning its status as
the sole and exclusive bargaining agent of PSI's rank and file employees. 1. Yes, 1st, the strike staged by FPWU-NLU was baseless. 2nd, the union failed to serve the
company a copy of the notice of strike (see Sec 3 Rule XXII, Book V, Implementing Rules). 3rd,
HELD: the strike blatantly disregarded the prohibition on the doing of any act which may impede or
YES. The pendency of an election protest involving both the mother federation and the local disrupt the conciliation proceedings (see Sec 6)
union did not constitute a bar to a valid disaffiliation. 2. No, The mother union, acting for and in behalf of its affiliate, had the status of an agent while
Reasoning In Liberty Cotton Mills Workers Union vs. Liberty Cotton Mills, Inc. the SC upheld the the local union remained the basic unit of the association, free to serve the common interest of
right of local unions to separate from their mother federation on the ground that as separate and
all its members subject only to the restraints imposed by the constitution and by-laws of the
voluntary associations, local unions do not owe their creation and existence to the national
federation to which they are affiliated but, instead, to the will of their members. The sole essence association. The same is true even if the local union is not a legitimate labor organization
of affiliation is to increase, by collective action, the common bargaining power of local unions for The direct and primary responsibility for the damages fall on the local union FPWU, being the
the effective enhancement and protection of their interests. principal, and not on respondent NLUTUCP, a mere agent of FPWU-NLU which assisted the
latter in filing the notice of strike. Being just an agent, the notice of strike filed by Lerum is
Yet the local unions remain the basic units of association, free to serve their own interests deemed to have been filed by its principal, the FPWU-NLU. Having thus dismissed the claim for
subject to the restraints imposed by the constitution and by-laws of the national federation, and damages against the principal, FPWU-NLU, the action for damages against its agent,
free also to renounce the affiliation upon the terms laid down in the agreement which brought
such affiliation into existence. respondent NLU-TUCP, and Atty. Lerum, has no more leg to stand on and should also be
dismissed.
44
Disposition Petition is dismissed.
EXCEPTION: Even before the onset of the freedom period, disaffiliation may be carried out
ALLIANCE OF NACIONALIST AND GENUINE LABOR ORGANIZATION (ANGLO-KMU) vs. when there is a shift of allegiance on the part of the majority of the members of the union.
SAMAHAN NG MGA MANGGAGAWANG NAGKAKAISA SA MANILA BAY SPINNING MILLS (Associated Workers Union-PTGWO vs. NLRC) In this case, there was a unanimous
AT J.P. COATS (SAMANA BAY), G.R. NO. 118562 July 5, 1996 confirmation by the members of SAMANA BAY of the disaffiliation - clearly a shift of allegiance
of the majority (if not all) of the union members.

Doctrine: 2.) NO.


Disaffiliation of a local union from its mother union is valid even if certain The mere act of disaffiliation does not divest the local union of its personality and neither does
procedural requirements for a valid disaffiliation was not complied with, provided the this give the mother federation any license to act independently of the local union. A local union
general membership of the local union ratified the disaffiliation action. does not owe its existence to the mother federation with which it is affiliated. There is only a
contract of agency wherein the mother federation acts in representation of its affiliate local
union. A local labor union is a separate and distinct unit, primarily designed to secure and
Facts: maintain an equality of bargaining power between the employer and their employee-members.
Petitioner Alliance of Nationalist and Genuine Labor Organization (ANGLO for By SAMANA BAY'S disaffiliation from ANGLO, the vinculum that previously bond the two entities
short) is a duly registered labor organization. Respondent union Samahan Ng mga was completely severed.
Manggagawang Nagkakaisa sa Manila Bay Spinning Mills and J.P. Coats (SAMANA BAY for
short) is its affiliate. In representation of SAMANA BAY, ANGLO entered and concluded a CBA Therefore, ANGLO was divested of any and all power to act in representation of SAMANA BAY.
with Manila Bay Spinning Mills and J.P. Coats Manila Bay Inc. (hereinafter referred to as the Hence, any act by ANGLO affecting SAMANA BAY, including the ouster of its officers, is without
corporations) on November 1, 1991. In December 1993, SAMANA BAY decided to disaffiliate force and effect.
from ANGLO because of the latter's dereliction of duty in promoting and advancing SAMANA
BAY's welfare. There were also alleged cases of corruption involving ANGLO officers. The Progressive Development Corporation v. The Honorable Secretary, Department of Labor
disaffiliation was confirmed by the members of SAMANA BAY unanimously. Apparently, ANGLO and Employment, G.R. No. 96425, February 4, 1992.
furnished the corporations copies of SAMANA BAY's FACTS:
resolution to disaffiliate. The corporations refused to honor the disaffiliation. In response to this, The controversy in this case centers on the requirements before a local or chapter of a
a petition to stop remittance of federation dues to ANGLO was filed by SAMANA BAY with the federation may file a petition for certification election and be certified as the sole and exclusive
Bureau of Labor Relations. ANGLO counter-acted by unseating all officers and board members bargaining agent of the petitioner’s employees.
of SAMANA Pambansang Kilusan ng Paggawa (KILUSAN) -TUCP (hereinafter referred to as Kilusan) filed
BAY and appointing in their stead new officers who were duly recognized by the corporations. with the Department of Labor and Employment (DOLE) a petition for certification election among
ANGLO submitted a position paper with the Med-Arbiter, contending that the disaffiliation was the rank-and-file employees of the petitioner alleging that it is a legitimate labor federation and
void considering that a CBA was still existing and the freedom period has not set in. The Med- its local chapter, Progressive Development Employees Union, was issued charter certificate.
Arbiter resolved that the disaffiliation was void, but upheld the illegality of the ousting by ANGLO Kilusan claimed that there was no existing collective bargaining agreement and that no other
of the SAMANA BAY officers. Both parties appealed to the DOLE. The DOLE reversed the Med- legitimate labor organization existed in the bargaining unit.
Arbiter's decision and ultimately ruled for SAMANA BAY. ANGLO's MR to the DOLE was denied Petitioner PDC filed its motion to dismiss contending that the local union failed to comply with
hence it filed a petition for certiorari with the Supreme Court. Rule II Section 3, Book V of the Rules Implementing the Labor Code, as amended, which
requires the submission of: (a) the constitution and by-laws; (b) names, addresses and list of
officers and/or members; and (c) books of accounts.
Issues:
ISSUE: Whether or not a local union need to be a Legitimate Labor Union on despite its
1.) WON the disaffiliation of SAMANA BAY from ANGLO was valid issuance of charter certificate.

2.) WON the unilateral ousting by ANGLO of officers and board members of RULING:
SAMANA BAY was valid Yes. But while Article 257 cited by the Solicitor General directs the automatic conduct of a
certification election in an unorganized establishment, it also requires that the petition for
certification election must be filed by a legitimate labor organization. Article 242 enumerates the
Ruling: exclusive rights of a legitimate labor organization among which is the right to be certified as the
exclusive representative of all the employees in an appropriate collective bargaining unit for
1.) YES. purposes of collective bargaining.
The Court will not bend the principle that all employees enjoy the right to self-organization and to Meanwhile, Article 212(h) defines a legitimate labor organization as “any labor organization duly
form and join labor organizations of their own choosing for the purpose of collective bargaining registered with the DOLE and includes any branch or local thereof.”
due to a mere procedural defect. The law is not without exceptions. Settled is the rule that a Rule I, Section 1(j), Book V of the Implementing Rules likewise defines a legitimate labor
local union has the right to disaffiliate from its mother union when circumstances warrant organization as “any labor organization duly registered with the DOLE and includes any branch,
(Volkshel local or affiliate thereof.”
Labor Union v. Bureau of Labor Relations)
GENERAL RULE: a labor union may disaffiliate from its mother union to form a local/ It is important to clarify the relationship between the mother union and the local union. In the
independent union only during the 60-day freedom period immediately preceding the expiration case of Liberty Cotton Mills Workers Union v. Liberty Cotton Mills, Inc., 66 SCRA 512 [1975]),
of a CBA. the Court held that the mother union, acting for and in behalf of its affiliate, had the status of an
45
agent while the local union remained the basic unit of the association, free to serve the common is no legal justification to support the conclusion that a trade union center is allowed to directly
interest of all its members subject only to the restraints imposed by the constitution and by-laws create a local or chapter through chartering.
of the association. Thus, whereas in this case the petition for certification election was filed by
the federation which is merely an agent, the petition is deemed to be filed by the chapter, the If the law’s intent were otherwise, the law could have so easily and conveniently
principal, which must be a legitimate labor organization. The chapter cannot merely rely on the included “trade union centers” in identifying the labor organizations allowed to charter a chapter
legitimate status of the mother union. or local. Since under the pertinent status and applicable implementing rules, the power granted
to labor organizations to directly create a chapter or local through chartering is given to a
federation or national union, then a trade union center is without authority to charter directly.
b. Registration
San Miguel Corporation Employees Union – Philippine Transport and General Workers Disposition: Petition GRANTED. RESPONDENT’s Certificate of Registration CANCELLED.
Organization (SMCEUPTGWO) v. San Miguel Packaging Products Employees Union –
Pambansang Diwa ng Manggagawang Pilipino (SMPEU-PDMP), G.R. No. 171153, I. Definition
September 12, 2007. A legitimate labor organization is defined as “any labor organization duly registered with the
Department of Labor and Employment, and includes any branch or local thereof.” Legitimate
FACTS: labor organizations have exclusive rights under the law which cannot be exercised by non-
Petitioner is the incumbent bargaining agent for the regular monthly-paid rank and file legitimate unions, one of which is the right to be certified as the exclusive representative of all
employees of the Three Divisions of San Miguel Corporation (SMC). It had been the the employees in an appropriate collective bargaining unit for purposes of collective bargaining.
certified bargaining agent for 20 years—from 1987 to 1997. II. Registration
Respondent is registered as a chapter of Pambansang Diwa ng Manggagawang Pilipino 1. The procedure for registration of a local or chapter of a labor organization is
(PDMP). PDMP issued a Charter Certificate to Respondent. In compliance with registration provided in the Implementing Rules, which enunciates a two-fold procedure for the
requirements, RESPONDENT submitted the requisite documents to the BLR for the creation of a chapter or a local.
purpose of acquiring legal personality. a. The first involves the affiliation of an independent union with a federation or
Upon submission of its charter certificate and other documents, RESPONDENT was issued national union or industry union.
by the BLR a Certificate of Creation of Local or Chapter. b. The second involves the direct creation of a local or a chapter through the
PETITIONER then filed with the DOLE-NCR a petition seeking the cancellation of process of chartering.
RESPONDENT’s registration and its dropping from the rolls of legitimate labor 2. The intent of the law in imposing less requirements in the case of a branch or local of
organizations. PETITIONER alleged: a registered federation or national union is to encourage the affiliation of a local union
a. That RESPONDENT committed fraud and falsification and non-compliance with a federation or national union in order to increase the local union’s bargaining
to registration requirement; and powers respecting terms and conditions of labor.
b. That PDMP is not a legitimate labor organization, but a trade union 3. Once a certificate of registration is issued to a union, its legal personality cannot be
center, hence, it cannot directly create a local or chapter. subject to collateral attack—it may be questioned only in an independent petition for
DOLE-NCR dismissed PETITIONER’s petition and ruled that RESPONDENT is allowed to cancellation.
directly create a local or chapter. However, it found that RESPONDENT did not comply with 4. There is no legal justification to support the conclusion that a trade union center is
the 20% membership requirement and, thus, ordered the cancellation of its certificate of allowed to directly create a local or chapter through chartering.
registration and removal from the rolls of legitimate labor organizations. 5. If the law’s intent were otherwise, the law could have so easily and conveniently
RESPONDENT then appealed to the BLR, which declared that, as a chartered local union, included “trade union centers” in identifying the labor organizations allowed to charter
PETITIONER is not required to submit the number of employees and names of all its a chapter or local; Since under the pertinent staPetitioners and applicable
members comprising at least 20% of the employees in the bargaining unit where it seeks to implementing rules, the power granted to labor organizations to directly create a
operate. chapter or local through chartering is given to a federation or national union, then a
Further, although PDMP is considered as a trade union center, it is a holder of Registration trade union center is without authority to charter directly.
Certificate issued by the BLR, giving it the status of a legitimate labor organization. Thus,
PDMP can charter or create a local, such as RESPONDENT.
PETITIONER filed a Petition for Certiorari with the CA, which affirmed the assailed decision Coastal Subic Bay Terminal, Inc. v. Department of Labor and Employment – Office of The
and resolution. Secretary, G.R. No. 157117, November 20, 2006.

ISSUE: Is RESPONDENT a legitimate Labor Organization, having been created via chartering FACTS: Herein private respondents Coastal Subic Bay Terminal, Inc. Rank and File Union
by PDMP, which is a Trade Union Center? (CSBTI-RFU) and Coastal Subic Bay Terminal, Inc. Supervisory Union (CSBTI-SU) filed
separate petitions for certification election before the Med-Arbiter of Regional Office No. III.
Held: CSBTI-RFU insists that it is a legitimate labor organization issued a charter certificate by the
No, it is not. The PDMP failed to comply with the registration requirements of Associated Labor Union (ALU) and CSBTI-SU by the Associated Professional, Supervisory,
the Respondent to become a local or chapter. Although PDMP as a trade union center is a Office and Technical Employees Union (APSOTEU). They also alleged that the establishment
legitimate labor organization, it has no power to directly create a local or chapter, such as they sought to operate in was unorganized. Petitioner Coastal Subic Bay Terminal, Inc. opposed
RESPONDENT. Thus, RESPONDENT cannot be created under the more lenient requirements both petitions alleging that CSBTI-RFU and CSBTI-SU were not legitimate labor organizations
for chartering, but must have complied with the more stringent rules for creation and and the proposed bargaining units were not particularly described.
registration of an independent union, including the 20% membership requirement. There

46
The Med-Arbiter dismissed both petitions, holding that ALU and APSOTEU are one and the partakes the nature of a collateral attack. Hence, in the absence of any independent action for
same federation having a common set of officers. Thus, the two respondent unions were in cancellation of registration against either APSOTEU or ALU, and unless and until their
effect affiliated with only one federation. Both parties appealed to the Secretary of Labor, who registrations are cancelled, each continues to possess a separate legal personality. The CSBTI-
reversed the said decision stating that they have separate legal personalities to file their RFU and CSBTI-SU are therefore affiliated with distinct and separate federations, despite the
separate petitions for certification election. APSOTEU is also a legitimate labor organization commonalities of APSOTEU and ALU.
because it was properly registered pursuant to the 1989 Revised Rules and Regulations
implementing Republic Act No. 6715. ALU and APSOTEU are also separate and distinct labor Under the Implementing Rules, a chartered local union acquires legal personality through the
unions having separate certificates of registrations from DOLE. They have different sets of charter certificate issued by a duly registered federation or national union, and reported to the
locals. The two respondent unions are legitimate labor organizations chartered respectively by Regional Office in accordance with the rules implementing the Labor Code. A local union does
ALU and APSOTEU. Thus, the Secretary ordered the holding of separate certification election, not owe its existence to the federation with which it is affiliated. It is a separate and distinct
with the following choices: voluntary association owing its creation to the will of its members. Mere affiliation does not
I. For all rank and file employees of CSBTI: divest the local union of its own personality, neither does it give the mother federation the
1. COASTAL SUBIC BAY TERMINAL, INC. RANK-AND-FILE UNION-ALU- license to act independently of the local union. It only gives rise to a contract of agency, where
TUCP; and the former acts in representation of the latter. Hence, local unions are considered principals
2. NO UNION. while the federation is deemed to be merely their agent. As such principals, the unions are
II. For all supervisory employees of CSBTI: entitled to exercise the rights and privileges of a legitimate labor organization, including the right
1. COASTAL SUBIC BAY TERMINAL, INC. SUPERVISORY EMPLOYEES to seek certification as the sole and exclusive bargaining agent in the appropriate employer unit.
UNION-APSOTEU; and
2. NO UNION. The purpose of affiliation of the local unions into a common enterprise is to increase the
The latest payroll of the employer, including its payrolls for the last three months immediately collective bargaining power in respect of the terms and conditions of labor. When there is
preceding the issuance of this decision, shall be the basis for determining the qualified list of commingling of officers of a rank-and-file union with a supervisory union, the
voters. constitutional policy on labor is circumvented. Labor organizations should ensure the
On appeal, the Court of Appeals affirmed the Secretary. Hence, this instant petition. freedom of employees to organize themselves for the purpose of leveling the bargaining process
but also to ensure the freedom of workingmen and to keep open the corridor of opportunity to
ISSUES: Can the supervisory and rank-and-file unions file separate petitions for certification enable them to do it for themselves.
election?
Under Article 245 of the Labor Code, supervisory employees are not eligible for membership in a
HELD: First, with respect to APSOTEU’s status: Petitioner argues that APSOTEU improperly labor union of rank-and-file employees. The supervisory employees are allowed to form their
secured its registration from DOLE Regional Director and not from the BLR. Under Article 235 of own union but they are not allowed to join the rank-and-file union because of potential conflicts
the Labor Code, “applications for registration shall be acted upon by the Bureau, which means of interest. Further, to avoid a situation where supervisors would merge with the rank-and-
the BLR and/or the Labor Relations Division in the Regional Offices of the Department of Labor.” file or where the supervisors’ labor union would represent conflicting interests, a local
Section 2, Rule II, Book V of the 1989 Revised Implementing Rules of the Labor Code supervisors’ union should not be allowed to affiliate with the national federation of
further provides that “any national labor organization or labor federation or local union may file unions of rank-and-file employees where that federation actively participates in the union
an application for registration with the Bureau or the Regional Office where the applicant’s activity within the company. Thus, the limitation is not confined to a case of supervisors
principal offices is located.” wanting to join a rank-and-file union. The prohibition extends to a supervisors’ local union
Section 1, Rule III of Book V of the Implementing Rules as amended by Department applying for membership in a national federation the members of which include local unions of
Order No. 9 also provides that “the application for registration of any federation, national or rank-and-file employees. In De La Salle University Medical Center and College of Medicine v.
industry union or trade union center shall be filed with the Bureau. Where the application is filed Laguesma, we reiterated the rule that for the prohibition to apply, it is not enough that the
with the Regional Office, the same shall be immediately forwarded to the Bureau…The supervisory union and the rank-and-file union are affiliated with a single federation. In addition,
application for registration of an independent union shall be filed with and acted upon by the the supervisors must have direct authority over the rank-and-file employees.
Regional Office where the applicant’s principal office is located.”
DOLE issued Department Order No. 40-03 further amending Book V of the above In the instant case, the national federations that exist as separate entities to which the rank-and-
implementing rules, which provides that “applications for registration of labor organizations shall file and supervisory unions are separately affiliated with, do have a common set of officers. In
be filed either with the Regional Office or with the BLR” addition, APSOTEU, the supervisory federation, actively participates in the CSBTI-SU while
Even after the amendments, the rules did not divest the Regional Office and the ALU, the rank-and-file federation, actively participates in the CSBTI-RFU, giving occasion to
BLR of their jurisdiction over applications for registration by labor organizations. The possible conflicts of interest among the common officers of the federation of rank-and-file and
amendments to the implementing rules merely specified that when the application was the federation of supervisory unions. For as long as they are affiliated with the APSOTEU and
filed with the Regional Office, the application would be acted upon by the BLR. ALU, the supervisory and rank-and-file unions both do not meet the criteria to attain the status of
In case at bar, APSOTEU’s principal office is in Diliman, Quezon City. Its registration was legitimate labor organizations, and thus could not separately petition for certification elections.
filed with the NCR Regional Office. At that time, the law applicable was Section 2, Rule Ii, Book Thus, petition is granted.
V of the Implementing Rules. Thus, APSOTEU’s certificate of registration is valid.
Sugbuanon Rural Bank, Inc. v. Laguesma, G.R. No. 116194, February 2, 2000.
Now with the issue: The answer is NO. Once a labor union attains the status of a legitimate FACTS:
labor organization, it continues as such until its certificate of registration is cancelled or Petitioner is a duly-registered banking institution in Cebu, while private respondent APSOTEU-
revoked in an independent action for cancellation. The legal personality of a labor TUCP was a labor organization duly-registered with the Labor Department. APSOTEU filed a
organization cannot be collaterally attacked. Thus, when the personality of the labor petition for certification election of the 5 supervisory employees. SRBI filed a motion to dismiss
organization is questioned in the same manner the veil of corporate fiction is pierced, the action the union's petition on two grounds. First, that the members of union were in fact managerial or
47
confidential employees and were to be disqualified from forming or joining unions. Second, the ART. 256. [245-A] Effect of Inclusion as Members of Employees Outside the Bargaining
Association of Labor Unions-TUCP was representing the union. Since ALU-TUCP also sought to Unit.198 The inclusion as union members of employees outside the bargaining unit shall not be
represent the rank-and-file employees of SRBI, there was a violation of the principle of a ground for the cancellation of the registration of the union. Said employees are automatically
separation of unions. The union argued that its members were not managerial employees but
deemed removed from the list of membership of said union.
merely supervisory employees.
The Med-Arbiter denied petitioner's motion to dismiss. The same was denied on appeal to the
Secretary of Labor and Employment. Thus, the certification election was ordered. Holy Child Catholic School v. Sto. Tomas, G.R. No. 179146, July 23, 2013.
Subsequently, the petition reached DOLE Regional Office seeking the cancellation of the De La Salle Unviersity Medical Center and College of Medicine v. Laguesma, G.R. No.
respondent union's registration. Respondent DOLE Undersecretary denied SRBI's appeal for 102084, August 12, 1998.
lack of merit. He ruled that APSOTEU-TUCP was a legitimate labor organization fully entitled to
all the rights and privileges granted by law including the right to file a petition for certification d. Local Union Disaffiliation
election.
National Union of Bank Employees (NUBE) v. Philnabank Employees Association (PEMA),
ISSUE: Should the petition for certification election be granted? G.R. No. 174287, August 12, 2013.

HELD: Philippine Skylanders, Inc. v. National Labor Relations Commission, G.R. No. 127374,
Yes. The petitioner bank failed to show that their Cashiers, Accountant, and Acting Chief of the January 31, 2002.
Loans Department possessed managerial powers and duties. At best they only had
recommendatory powers subject to evaluation, review, and final decision by the bank's Alliance of Nationalist and Genuine Labor Organization (ANGLO-KMU) v. Samahan ng
management. The job description forms submitted by petitioner clearly show that the union
mga Manggagawang Nagkakaisa sa Manila Bay Spinning Mills at J.P. Coats (SAMANA
members in question may not transfer, suspend, lay-off, recall, discharge, assign, or discipline
employees. Moreover, the forms also do not indicate that said officers could formulate and BAY), G.R. No. 118562, July 5, 1996.
execute management policies which are normally expected of management officers. Neither
could the union members be treated as confidential employees. Although the cashier serves the
bank's management, it could not be deemed to have access to confidential information
specifically relating to SRBI's labor relations policies.

As regards the issue on the violation of the principle of separation of unions, records show that
respondent union was initially assisted by ALU during its preliminary stages. A local union
maintains its separate personality despite affiliation with a larger national federation. APSOTEU-
TUCP had separate legal personality from ALU and TUCP.

The law frowns on a union where the membership is composed of both supervisors and rank-
and-file employees, for fear that conflicts of interest may arise in the areas of discipline,
collective bargaining, and strikes. However, in the present case, none of the members of the
respondent union came from the rank-and-file employees of the bank.

Filipino Pipe and Foundry Corporation v. National Labor Relations Commission, G.R. No.
115180, November 16, 1999. (supra, but different topic)

Progressive Development Corporation v. The Honorable Secretary, Department of Labor


and Employment, G.R. No. 96425, February 4, 1992 (supra. but different topic na )

c. Supervisory and Rank-and-File Union Affiliation


• Articles 255 and 256, Labor Code.

ART. 255. [245] Ineligibility of Managerial Employees to Join any Labor Organization; Right of
Supervisory Employees.197 Managerial employees are not eligible to join, assist or form any
labor organization. Supervisory employees shall not be eligible for membership in the collective
bargaining unit of the rank-and-file employees but may join, assist or form separate collective
bargaining units and/or legitimate labor organizations of their own. The rank and file union and
the supervisors' union operating within the same establishment may join the same federation or
national union.

48

Das könnte Ihnen auch gefallen